• 11.03 MB
  • 2021-05-14 发布

历年高考物理高考试题及答案

  • 314页
  • 当前文档由用户上传发布,收益归属用户
  1. 1、本文档由用户上传,淘文库整理发布,可阅读全部内容。
  2. 2、本文档内容版权归属内容提供方,所产生的收益全部归内容提供方所有。如果您对本文有版权争议,请立即联系网站客服。
  3. 3、本文档由用户上传,本站不保证质量和数量令人满意,可能有诸多瑕疵,付费之前,请仔细阅读内容确认后进行付费下载。
  4. 网站客服QQ:403074932
1952年试题 力学、物性及声学(共40分) (1)[3分] 某物体的速度在10分钟内从36千米/小时增到57.6千米/小时. 它的平均加速度是( )厘米/秒2. (2)[4分] 由手中铅直向上抛出一球.抛出2秒钟后,球回到手里.如果空 气的阻力可以忽略不计,那么抛出去时,球的速度是( )米 /秒,球上升的高度是( )米. (3)[3分] 一重200克的物体,放在和水平面成30°角的斜面上.如果这 物体和斜面之间没有摩擦力,那么沿斜面的方向要加( ) 克的力,才能使这物体静止在斜面上. (4)[3分] 载重汽车和公共汽车的轮胎比小汽车的轮胎宽得多,轮子也 不止四个.这是为了减少它们对地面的( ). (5)[2分] 木块因为受了浮力才能浮在水面,那么沉下水里的石头是没 有受到水的浮力.这个结论是错的还是对的?( ). (6)[3分] 砖墙如果建立在潮湿的地方,潮湿会沿着砖墙上升.这是由于 ( ). (7)[4分] 北京的重力加速度是980厘米/秒2,赤道的重力加速度是978 厘米/秒2.某物体的质量是100克.在北京,50克的力作用在这 物体上,将产生加速度( )厘米/秒2,而在赤道上,50克的 力作用在这物体上,将产生加速度( )厘米/秒2. (8)[3分] 火车重500公吨,在笔直的、水平的铁轨上,等速前进.如果车 轮与铁轨之间的摩擦系数是0.002,那么火车做等速运动时, 机车的牵引力是( )千克.(空气的阻力可以不计). (9)[4分] 某人体重60千克,用绳经定滑轮提起重40千克的物体.那么此 人对地面的总压力是( )千克. (10)[4分] 劳卫制锻炼中,爬绳及格的标准是5米.一个体重60千克的人 要达到这个标准,每爬一次需做功( )焦耳.(重力加速度 是980厘米/秒2. (11)[5分] 某物体的质量是20克,受( )达因的力的作用,由静止开始 运动,能在10秒钟末,得到1焦耳的动能. (12)[2分] 声音是由于物体( )而产生的. 热学及光学(共30分) (1)[2分] 水的温度由4℃开始上升,它的体积要( ).温度由4℃开始 下降,它的体积要( ). (2)[3分] 液体的沸腾温度,在外部压力减小时( ),在外部压力增大 时( ).因此,在高山上水的沸腾温度比地面上的( ). (3)[3分] 白金的线膨胀系数是0.000009.一块白金的温度由0℃升高到 ( )℃时,它的体积的增大是0℃时体积的1/100. (4)[4分] 热功当量是427千克米/千卡.从34.2米高的河堤上落下的水, 如果全部动能变成了热量,那么水的温度将升高( )℃ (5)[3分] 水的比热是0.5卡/克度,它的熔解热是80卡/克.使1千克— 30℃的冰变成40℃的水,需要( )千卡的热量. (6)[2分] 光线垂直投射到平面镜上.如果把镜面旋转30°角,那么入射 光线和反射光线的夹角是( ). (7)[3分] 用一个50烛光的电灯照射某物体.当灯和物体相距5米时,得 到某一定的照明.如果用另一个电灯放在4米处照射这物体而 得到同样的照明,那么后一个电灯的烛光数是( ). (8)[2分] 近视眼镜是( )透镜做成的,远视眼镜是( )透镜做成 的. (9)[4分] 灯与幕之间的距离是125厘米.当会聚透镜距灯的距离是25厘 米时,在幕上便可得到灯的清晰的像.那么会聚透镜的焦距是 ( )厘米. (10)[4分] 要增加望远镜的放大率,应该用焦距( )的目镜和焦距 ( )的物镜. 电磁学(共30分) (1)[4分] 用钢铁制的船身,大半都微微磁化.某船磁化的方向与船身垂 直,面向船头时,船右是N极.当这船向正南航行时,罗盘的N极 必定指( )偏( ). (2)[3分] 一个平行板空气介质的电容器,充电后,在板中间插入一片玻 璃.两板间的电位差必将( ). (3)[3分] 一个2欧姆的电阻和一个( )欧姆的电阻并联,可以得到 1.2欧姆的电阻. (4)[4分] 有一个60瓦、220伏特的电灯泡.它的灯丝的电阻在亮的时候, 是( )欧姆. (5)[4分] 用电解法提炼纯铜.所得纯铜的质量和电流通过的时间成 ( )比,和通过电流的强度成( )比. (6)[4分] 变压器原线圈和副线圈里的电流,与它们的( )成( ) 比. (7)[4分] 两根平行导线通过电流.两个电流的方向相反时,它们之间的 作用力是使两线相( ). (8)[4分] 一条竖直的铜棍,在空中自西向东移动.因地磁场所产生的感 应电动势,使铜棍上端的电压比铜棍下端的电压( ). 1952年答案 力学、物性及声学(共40分) 1.(1厘米/秒2) 2.(9.8米/秒),(4.9米) 3.(100克) 4.(压力) 5.(错) 6.(毛细管现象) 7.(490厘米/秒2),(489厘米/秒2) 8.(1000千克) 9.(20千克) 10.(2940焦耳) 11.(2000达因) 12.(振动) 热学及光学(共30分) 1.(增大),(增大) 2.(下降),(上升),(低) 3.(370℃) 4.(0.08℃) 5.(135千卡) 6.(60°) 7.(32) 8.(凹),(凸) 9.(20厘米) 10.(短),(长) 电磁学(共30分) 1.(北),(东) 2.(降低) 3.(3欧姆) 4.(807欧姆) 5.(正),(正) 6.(匝数),(反) 7.(排斥) 8.(低) _ 1953年试题 力学、物性学及声学(共24分) (1)[5分] 由手中铅直向上抛出一球,抛出5秒钟后,球回到手里,如果空 气的阻力可以略而不计,那么抛出去时,球的速度是( )米 /秒,球上升的最高度是( )米. (2)[4分] 某物体沿直线运动,它的初速度是72公里/小时,经过200米的 路程而静止,它的平均加速度是( )厘米/秒2. (3)[4分] 设有两物体,它们的质量各为M及M',原来相对静止,因互相作 用而开始运动,经若干秒后,它们的速度的大小各变为V及V', 则M,M',V,V'各量之间的关系是( ). (4)[4分] 在空气里称起来是等重的一块铁块和一块铝块,在水里称时 ( )比( )重,在真空里称时( )比( )重. (5)[4分] 有重100克的物体,放在长一米的斜面上,当这个斜面上端的 高度渐次增加到60厘米时,物体即开始下滑,因此,最大摩擦 是( )克,摩擦系数是( ). (6)[3分] 发声体遇有与自己( )相同的声波传来时能渐行振动而发声, 此种现象谓之( ). 热学及光学(共24分) (7)[4分] 在标准状况下,有一气泡由水底升至水面时,其体积增大一倍, 这水的深度是( )米. (8)[4分] 把质量为1000克的物体加热到80℃,然后投入温度为20℃质 量为100克的水中,混和后温度变为50℃,如并无热量散失,则 该物体的比热是( )卡/克度. (9)[4分] 站立在火炉近旁,觉得很热,热量传到我们身上主要是由于 ( )的方式,其次是由于( ). (10)[4分] 白色光束由空气通过玻璃的三棱镜后散为七色,按折射率排 列这七色是( ),此中红色的折射率比绿色( ),红色的 速度比绿色( ). (11)[4分] 平常看书,书上的照度是15米烛光就可以了,现在有60烛光的 灯应挂在离书( )米的地方. (12)[4分] 一个物体放在一个凹面镜的前面,它的位置在镜的焦点和曲 率中心之间,所成的像一定在( )和( )之间. 电磁学及近代物理(共占32分) (13)[4分] 在匀强磁场里有一个线圈作平行移动,线圈中是否产生感生 电流?答( ),因为( ). (14)[4分] 图示线路里,有二个完全相同的电池,它们的电动势各为1.5 伏 特, 内 阻各 为1欧 姆 , 它 们 并 联 着,通 过 电 阻 r1=1.3 欧 姆,r2=2欧姆,r3=3欧姆.那么,通过的电流是( )安培. (15)[3分] 图示电磁铁的右方置一小磁针.当电流沿箭头所示方向流过 时,磁针的N极向哪方向偏转.答( ). (16)[4分] 某洗衣店用电熨斗5个,并联于100伏特的电源上,每个电熨斗 的电阻是20欧姆,那么,电熨斗总共消耗的电功率为( )瓦 特. (17)[3分] 将一库仑的正电荷自电场内的甲点移至乙点需功1焦耳,那么, 甲点电位比乙点电位高( )伏特. (18)[4分] 三极真空管中,由( )极发射( ),穿过( )极,达到 ( )极而成电流. (19)[2分] 通电于硝酸银中,经2秒钟,把负极析出银0.001118克,所通过 的电量( )库仑,其电流是( )安培. (20)[4分] 设有一变压器,其原线圈和副线圈的匝数各为500及10,000, 通过原线圈里的电流是10安培,电压是2000伏特,如无电能耗 失,副线圈里的电流是( )安培,电压是( )伏特. (21)[4分] 原子序数由原子核的( )决定的,原子量是由原子核内的 ( )和( )的质量决定的,元素的化学性质是由原子的 ( )决定的. 混和题(共占20分) (22)在一场强为50个静电单位的匀强电场中,放置一个质量2克的单位正 电荷(见图).如果我们给这电荷一个与电场平行而方向相反的初速度 V0=75厘米/秒. 甲.[4分] 问这电荷所受的力若干?答( ). 乙.[8分] 问在4秒钟后这电荷距出发点多远?答( ). 丙.[8分] 问在4秒钟后这电荷的速度多少?方向如何?答方向 ( ),速度( ). 1953年答案 力学、物性学及声学(共24分) (1)[5分] 由手中铅直向上抛出一球,抛出5秒钟后,球回到手里,如果空 气的阻力可以略而不计,那么抛出去时,球的速度是(24.5)米 /秒,球上升的最高度是(30.625)米. (2)[4分] 某物体沿直线运动,它的初速度是72公里/小时,经过200米的 路程而静止,它的平均加速度是(-100)厘米/秒2. (3)[4分] 设有两物体,它们的质量各为M及M',原来相对静止,因互相作 用而开始运动,经若干秒后,它们的速度的大小各变为V及V', 则M,M',V,V'各量之间的关系是(MV=M'V'). (4)[4分] 在空气里称起来是等重的一块铁块和一块铝块,在水里称时 (铁)比(铝)重,在真空里称时(铝)比(铁)重. (5)[4分] 有重100克的物体,放在长一米的斜面上,当这个斜面上端的 高度渐次增加到60厘米时,物体即开始下滑,因此,最大摩擦 是(60)克,摩擦系数是(3/4). (6)[3分] 发声体遇有与自己(振数)相同的声波传来时能渐行振动而发 声,此种现象谓之(共鸣). 热学及光学(共24分) (7)[4分] 在标准状况下,有一气泡由水底升至水面时,其体积增大一倍, 这水的深度是(10.34)米. (8)[4分] 把质量为1000克的物体加热到80℃,然后投入温度为20℃质 量为100克的水中,混和后温度变为50℃,如并无热量散失,则 该物体的比热是(0.1)卡/克度. (9)[4分] 站立在火炉近旁,觉得很热,热量传到我们身上主要是由于 (辐射)的方式,其次是由于(对流) . (10)[4分] 白色光束由空气通过玻璃的三棱镜后散为七色,按折射率排 列这七色是(红,橙,黄,绿,蓝,靛,紫),此中红色的折射率比 绿色(小),红色的速度比绿色(大). (11)[4分] 平常看书,书上的照度是15米烛光就可以了,现在有60烛光的 灯应挂在离书(2)米的地方. (12)[4分] 一个物体在一个凹面镜的前面,它的位置在镜的焦点和曲率 中心之间,所成的像一定在(曲率中心)和(无穷远)之间. 电磁学及近代物理(共占32分) (13)[4分] 在匀强磁场里有一个线圈作平行移动,线圈中是否产生感生 电流?答(否),因为(通过线圈的磁力线无变化). (14)[4分] 图示线路里,有二个完全相同的电池,它们的电动势各为1.5 伏 特, 内 阻各 为1欧 姆 , 它 们 并 联 着,通 过 电 阻 r1=1.3 欧 姆,r2=2欧姆,r3=3欧姆.那么,通过的电流是(0.5)安培. (15)[3分] 图示电磁铁的右方置一小磁针.当电流沿箭头所示方向流过 时,磁针的N极向哪方偏转.答(反时针方向). (16)[4分] 某洗衣店用电熨斗5个,并联于100伏特的电源上,每个电熨斗 的电阻是20欧姆,那么电熨斗总共消耗的电功率为(2500)瓦 特. (17)[3分] 将一库仑的正电荷自电场内的甲点移至乙点需功1焦耳,那么, 甲点电位比乙点电位高(1)伏特. (18)[4分] 三极真空管中,由(丝)极发射(电子),穿过(栅)极,达到(板) 极而成电流. (19)[2分] 通电于硝酸银中,经2秒钟,把负极析出银0.001118克,所通过 的电量(1 )库仑,其电流是(0.5)安培. (20)[4分] 设有一变压器,其原线圈和副线圈的匝数各为500及10,000, 通过原线圈里的电流是10安培,电压是2000伏特,如无电能耗 失,副线圈里的电流是(0.5)安培,电压是(40,000)伏特. (21)[4分] 原子序数由原子核的(质子数)决定的,原子量是由原子核内 的(质子)和(中子)的质量决定的,元素的化学性质是由原子 的(外层电子)决定的. 混和题(共占20分) (22)在一场强为50个静电单位的匀强电场中,放置一个质量是2克的单位 正电荷(见图).如果我们给这电荷一个与电场平行而方向相反的初速 度V0=75厘米/秒. 甲.[4分] 问这电荷所受的力若干?答(50达因). 乙.[8分] 问在4秒钟后这电荷距出发点多远?答(100厘米). 丙.[8分] 问在4秒钟后这电荷的速度多少?方向如何?答方向(电场 方向),速度(25厘米/秒). 1954年试题 1."一个过氧化氢分子(H2O2)中,含有一个氢分子(H2)和一个氧分子(O2)".这句 话是否正确?为什么? (10分) 2.下列各对化合物能否进行反应?根据什么原则来判定? (甲)铜和硫酸锌溶液,(乙)硫酸钠溶液和氯化钡溶液, (丙)碳酸钾溶液和醋酸. (15分) 3.在高炉(鼓风炉)炼铁过程中:2Fe2O3+3C=4Fe+3CO2↑每生产一吨铁需要焦炭 (假定为纯碳)多少吨?同时产生二氧化碳多少升?(在标准状态下)(Fe=56,C=12) (15分) 4.门捷列夫周期表的第三周期中,有下列八种元素: Na,Mg,Al,Si,P,S,Cl,Ar. (甲)举出各元素的最高原子价.(乙)哪些元素是金属,哪些是非金属? (丙)说明它们成酸成碱性质的递变. (20分) 5.今有三种有机化合物,它们的分子式都是C3H8O,试写出它们的结构式和化学 名称来. (15分) 6.用锌和盐酸制氢气,在导管出口处,点燃时,为什么有时会发生爆炸?应如何在 燃氢时避免爆炸? (10分) 7.实验室中制氯的反应方程式如下:4HCl+MnO2=MnCl2+Cl2↑+2H2O (甲)上式中哪个元素被氧化了?哪个元素被还原了? (乙)二氯化锰在这一反应中起了什么作用? (丙)由氯酸钾制氧时,也用到二氧化锰,它是否起了同样的作用?为什么? (15分) 1955年试题 一、力学共28分 (1)[6分] 地上放着一个10千克重的物体,一个孩子把物体竖直向上提, 但他所能用的力只有7千克重.问物体共受哪几个力作用,各等于多少? (2)[10分] 从同一地点用相等的初速度V0=49米/秒先后竖直向上抛出两 块石头,第二块比第一块晚抛出2秒.问第一块石头抛出后,经过几秒钟,两块石 头在空中相碰?( ) (3)[6分] 重量是10,000达因的物体由1米高处自由下落,如空气的阻力 可以忽略不计,求物体到达地面时的动能.( ) (4)[6分] 一个单摆,摆动周期等于2秒.求下列情况下,摆的周期:摆长 缩短为原来的1/4时,周期=( 秒),摆球的质量减为原来的1/4时,周期 =( 秒),振幅减为原来的1/4时,周期=( 秒). 二、热学及分子物理共22分 (5)[6分] 原来在20℃的金属,温度减到0℃时,密度增加3千分之一, 求金属的线胀系数( ). (6)[8分] 在密闭容器里装一些0℃的水.当迅速地从容器里抽出空 气时,由于水的急剧蒸发,剩下的水就逐渐完全结成冰.问这样结冰的水, 能占水的原有总质量的几分之几?(≈ %),水在0℃时的汽化热等于 595卡/克;冰的熔解热等于80卡/克. (7)[8分] 在温度等于127℃,压强等于720毫米水银柱高时,空气的 密度等于( )克/(厘米)3.在标准状态下的空气密度等于0.00129克 /(厘米)3. 三、电学、光学、原子结构共40分 (8)[6分] 在右图所示的电路中,电池组的电动势ε=6.0伏特,R1=6.0 欧姆,R2=3.0欧姆,伏特计V指5.0伏特.那么,安培计A指 ( )安培,电池组的内阻等于( )欧姆. (9)[5分] 一个阴极射线管的阴极射线从南向北射出.把这管子夹在 马蹄形的磁铁中,N极在西,S极在东.那么,阴极射线要向 ( )偏转. (10)[6分] 按照图中所表示的条形磁铁的N极向线圈插入时,右方磁针 的S极向哪方偏转?( ) (11)[5分] 水的折射率等于4/3,玻璃的折射率等于3/2.那末,光在水 中的传播速度比在玻璃中的传播速度( );光在水中传 播速度与在玻璃中传播速度之比等于( ). (12)[6分] 一个幻灯镜头的焦距等于1尺,镜头距幕的距离等于21尺. 要幕上的像清楚,幻灯片距镜头应当等于( )尺. (13)[6分] 下图中F是薄透镜的一个主焦点,用作图法找出物AB的像的 位置.用直尺在图上画出来.像的长度是物的( )倍. (14)[6分] 两个氧原子核相距10-8厘米,它们相互吸引还是相互排 斥?( ),这是因为( );它们之间的作用力等于( ) 达因.氧的原子序数等于8,电子的电荷等于5×10-10静电系 单位. 四、混合题共10分 (15)[10分] 某一抽水站拟购置一架220伏特的直流电动抽水机,从井内 将水抽到地面.设井内水面离地面的距离是10米,对抽水机 要求每分钟能出水600升,但电动机所供给的电能实际上只 有7%产生抽水作用,问: 甲.[7分]电动机的功率最少该是多少瓦?( ) 乙.[3分]当反电动势可以略去时,通过电机内的总电流是 多少安培?( ) 1955年答案 一、力学共28分 (1)[6分] 物体受10千克的重力,小孩向上提的力7千克,地面的反作 用力3千克. (2)[10分] 6秒 (3)[6分] 106尔格 (4)[6分] 1秒,2秒,2秒 二、热学及分子物理共22分 (5)[6分] 5.56×10-6/度 (6)[8分] 88 (7)[8分] 0.00083 三、电学、光学、原子结构共40分 (8)[6分]2.5,0.4 (9)[3分]上 (10)[6分]向左偏转 (11)[5分]大,9/8 (12)[6分]1.05 (13)[6分]1/3 (14)[6分]相互排斥,原子核是带阳电的,0.16 四、混和题共10分 (15)[10分](1)1.4千瓦,(2)6.36安培 1956年试题 一、[8分]甲乙两物体,由同一地点,向同一方向,以同样的加速度作匀加速运动, 其初速度皆为零.甲物体运动了2秒后,乙物体才开始运动.问乙物体开始 运动后经过多长时间,甲乙两物体间的距离恰是乙物体开始运动时两物体 间距离的两倍? 答案: 二、[7分]有一根长5尺粗细不均匀的木棒.若在距一端2尺处把它支起,恰好能平 衡.若把支点移到距另一端2尺处,就必须在这一端挂上一个2千克重的物 体,才能保持平衡.求此木棒的重量. 答案: 三、[9分]在地平面上放着一块10千克重的金属块.现在用3千克重的水平力拉着 它由静止开始运动.如果它和地面间的摩擦系数为0.2,问10秒末这金属块 具有多大的动能? 答案: 四、[6分]如图所示,在广袤无边的均匀媒质中有一正弦波沿AB方向传播.设某一 时刻CD两点间的波形如图所示,画出经过1/4周期后,CD两点间的波形. 五、[6分]一个有活塞的密闭容器内盛有饱和水蒸气与少量同温度的水.若保持 温度不变,慢慢地推进活塞,使水蒸气的体积减少到原来体积的一半,问水 蒸气的压强如何改变?( )在压缩过程中容器内还发生了什么变 化?( ) 六、[9分]将500克在熔点的液态铅倒入1升22℃的水内.铅在水内凝成固体,同时 水的温度升高,并有一部分水变成了100℃的蒸汽.如果未汽化的水的最后 温度是27℃,那么变成蒸汽的水的质量是多少?(铅的熔点是327℃,熔解热 是5卡/克,比热是0.03卡/克·度,水在100℃时的汽化热是539卡/克) 答案: 七、[8分]有一个电容器由两片圆形金属板构成,电介质是空气.每板半径是2厘 米,板间距离是0.1厘米.设所带电量是0.1静电系单位,(1)试求两板间的电 势差;(2)若两板间的电场可看作是匀强电场,求其电场强度. 答案:(1) (2) 八、[10分]有一电源,其电动势225伏特,内阻是2.5欧姆,其外电路由数盏220伏 特、40瓦特的电灯组成.如果希望电灯正常发亮,应该用几个灯?(连接电灯 的导线的电阻可以忽略不计.) 答案: 九、[7分]下图中A表示发电厂中一架交流发电机,L1、L2、L3等表示用户的灯. 设发电厂和用户间的距离很远,试补画下图表明发电机到电灯的输电线路 的主要部分,以及电灯的联接法,并注明线路图中各部分的作用. 十、[8分]照相机镜头的焦距是10厘米.现在要拍摄一个物体,要求像高是原物体 的1/5,算出物体应离镜头几厘米,并画出光路图. 答案: 十一、[8分]一束光线从空气射入某液体,当人射角是45°时,折射角是 30°.如果要在空气与这液体的分界面上发生全反射,光线应从分界面的 哪一边射入?( )并计算入射光线和界面法线间的角度应在什么范围内. 答案: 十二、[4分]目前以裂变方式放出原子能的原料中,用得最多的是( ),它的原 子质量数是( ),这种原子的原子核中有( )个质子和( )个中子. 十三、[10分]平行板电容器两个极板间的电场可以看作是匀强电场.现在有一电 容器,两极板间的电势差为4伏特.一粒质点应该具有多大的初速度才能 由低电势的极板面达到高电势的极板面?(质子和中子的质量都是1.6× 10-24克,电子的电量是4.8×10-10静电系单位.) 答案: 1956年答案 一、1秒 评分标准:若方程式列错,不给分. 若方程式列对,但推算过程中有错(如移项等),给5分. 若公式推算都对,而单纯数字计算有错误,给7分. 二、4千克重 评分标准:若单纯数字计算有错误,给5分. 三、480焦耳(或4.8×109尔格,或49千克重米) 评分标准:若单位错,给4分. 若单纯数字计算错,给8分. 四、 五、……(不改变)……(汽的质量减少,水的质量增多)…… 评分标准:共计6分.前者4分,后者2分. 六、≈3.3克 评分标准:若方程式列错,不给分. 若方程式列对,而没有算出最后结果,给5分.若单纯数字计算错误, 给8分. 七、(1)0.01静电系电位. (2)0.1静电系电位. 评分标准:全题共8分.(1)4分,(2)4分. 若单纯数字计算错误,给6分. 八、11个灯 评分标准:若单纯数字计算错误,给9分. 九、 T:升压变压器.作用:把发电机的电压升高,以减少远距离送电时输电线上的 电能消耗. T':降压变压器.作用:将高压降低以适合电灯的需要.电灯是并联的. 评分标准:若只画了一个变压器或两个都没画,不给分. 若画对了但作用注明错了,不给分. 若变压器的圈数不对(如升压时的原线圈数反而比副线圈数多), 而又未注明其作用,不给分. 若线圈数画错,而作用注明对了,给5分. 若其他部分都对,而电灯画为串联,给4分. 十、60厘米(5分) 图 (3分) 评分标准:全题共8分.计算部分5分,画图3分.若单纯数字计算错,计算部分 给3分. 十一、……(由液体那边射上去)…… 入射角应大于45°. 评分标准:全题共8分,每小题4分. 若单纯数字计算有错,计算部分给3分. 十二、……(铀)……(235)……(92)……(143)…… 评分标准:全题共4分,每一小题1分. 十三、2×106厘米/秒 评分标准:若单纯变换错误,给8分. 若单纯数字计算错误,给9分. 若a质点的质量和电量用错,而其余部分正确,给8分. 1957年试题 一、甲、乙二人用扁担抬水,扁担是均匀的,长1.5米,2千克重,水和水桶共18 千克重,挂在距甲0.5米的地方.求甲乙二人肩上各承担多大的力. 答案:甲: 乙: (6分) 二、把500克重的物体,从20米高的地方用10米/秒的速度竖直下抛.落地时的速 度是20米/秒.求物体所受的空气的平均阻力.(g=9.8米/秒2) 答案: (10分) 三、如图所示,一个大轮通过皮带拉着一个小轮转动,皮带和两个轮子之间没有 滑动.大轮的半径是小轮的2倍.大轮上有一点S离转动轴的距离是大轮半 径的1/3.当大轮边缘上P点的向心加速度是12厘米/秒2时,大轮上的S点 和小轮边缘上的Q点的向心加速度各为何值? 答案: (10分) 四、设有原长相同、横截面积也相同的铁丝和铜丝各一条.铁丝的一端固定在 天花板上,其下端挂一个18千克重的重物A.铜丝的一端固定在A的下面, 而在其另一端挂一重物B(如图).试问重物B的重量应该是多少才能使铁 丝与铜丝的伸长相等.(忽略铁丝与铜丝的重量).(铁的杨氏模量是20000 千克重/[毫米]2,铜的杨氏模量是11000千克重/[毫米]2). 答案: (8分) 五、设有一容器盛有一定质量的气体,当气体温度升高时,如果容器体积适当增 大,可以使气体压强保持不变.试用分子运动论说明,气体温度增高体积 增大时可以使压强保持不变的道理. 答案: (8分) 六、在应用量热器、天平、温度计、碎冰块和水测定冰的熔解热的实验中: (1)应该记录哪些温度?什么时候记录? (2)怎样测定冰的质量? (3)放入量热器的冰的质量如不合适:过多有什么不好?过少有什么不好? 答案:(1) (2) (3) (8分) 七、把两个电容器C1、C2联接如图,并使所带总电量是30静电系单位,如已知C1 的电容是7静电系单位,C2的电容是8静电系单位,求各个电容器所带的电 量: 答案: (10分) 八、有一电池,当外电路的电阻是2.9欧姆时,量得电流强度是0.5安培;当外电 路的电阻是1.4欧姆时,量得路端电压是1.4伏特.求电池的电动势和内电 阻. 答案: (10分) 九、水平放置一个有铁心的螺线管,在螺线管的一端有一悬挂着的轻小闭合线 圈,如图.问接通电流的瞬间轻小线圈将怎样运动?说明理由. 答案: (8分) 十、有一物体正立在凸透镜的主轴上,凸透镜的焦距等于4厘米,如果要得到一 个放大和正立的像,像高和物高的比等于2,求物体和透镜之间的距离.用 垂直于主轴的直线代表物体,画出光路图. 答案: (10分) 十一、试将下列问题的答案各填入右面括弧内: (2)炽热固体所发射的光线形成什么光谱?( ) (3)凸面镜对光线有什么作用?( ) (4)三棱镜对复色光有什么作用?( )(8分) 十二、平衡下列核反应方程式: 7N14+2He4→8O17+( ) 4Be9+2He4→6C12+( )(4分) 1957年答案 一、甲承担的力是13千克重. 乙承担的力是7千克重. 评分标准:全题共6分,若计算方法正确,单纯数字错误,给5分. 二、约117克重(或1.15×105达因). 评分标准:全题共10分.本题如果用运动学及动力学的方法,得出了加速 度而没有求出阻力,给4分.如果用功和能的方法,求出了克服 阻力所做的功而没有求出阻力,给6分.单纯数字计算错误,扣2 分,单位错误扣2分. 三、S点的向心加速度是4厘米/[秒]2. Q点的向心加速度是24厘米/[秒]2]. 评分标准:全题共10分,第1小题5分,第2小题5分. 四、22千克重. 评分标准:全题8分.解题方法若有任何错误,不给分. 如果有单纯数字错误,扣2分. 五、由于温度升高,分子运动的速度增大,因而使气体压强增大.但由于体积增 大,分子撞击器壁的次数减少,因而使气体压强减少.当适当增大体积时,这 两种作用互相抵消,故压强保持不变. 评分标准:全题8分.在解释中主要是看考生能否正确指出分子运动的速度 及撞击次数对压强的影响,上述二要点中任何一个如果回答正确 即可给4分. 六、(1)在将冰块放入量热器以前,记录其中水的温度;待冰完全熔解后,再记录 水的温度. (2)冰熔解后量热器及其中水的总质量减去放入冰以前量热器及其中水 的总质量,即得冰的质量. (3)冰过多则不能完全熔解;冰过少则水的温度降低太少,结果不易准确. 评分标准:全题8分.分配给各小题如下: (1)3分(初温1分,终温2分). (2)3分. (3)2分. 在第(1)问题,考生是否答出还要测量冰的温度不影响他得分 多少. 七、Q1=14静电系单位 Q2=16静电系单位 评分标准:全题共10分;单纯数字计算有错误时给8分;答案数值正确而单 位有错或漏写时,每一答案各扣1分. 八、E=1.5伏特.r=0.1欧姆. 评分标准:全题共10分;正确列出所有方程式而数字计算有错误,给8分;正 确列出所有方程式而没有进行计算,给6分;正确列出其中任一方 程式,给3分. 九、在接通电流的瞬间,轻小线圈向离开螺线管的方向运动;因在接通电流的瞬 间,在轻小线圈中产生与螺钱管电流方向相反的电流,所以受磁场的推斥而 向离开螺线管的方向运动. 评分标准:全题8分;只答出怎样运动而未说明理由时给2分. 十、2厘米 光路图(略) 评分标准:全题共10分,计算部分6分,做图部分4分. 计算式中正负号弄错,计算部分不给分.若方程式列对,单纯数字 计算有错误,扣2分.光路图画得不准,而原则上正确,不扣分. 十一、(1)2.25×1010厘米/秒. (2)连续光谱. (3)发散作用. (4)色散作用. 评分标准:全题共8分,每小题2分.第一小题不要求列算式.答案只要有 错,就不给分. 十二、 评分标准:全题4分,每个空格2分. 空格填写中文"质子"、"中子"或写成"P"、"n",不扣分. _ 1959年试题 一、一个带摆的时钟,在平原上计时准确.若把它移到高山上,问: (1)它将变快还是变慢?说明理由; (2)如何调节摆锤,才能使它恢复准确?说明理由. 答案: (6分) 二、一人在水平地面上移动一木箱.木箱的质量为60千克,木箱和地面间的滑动 摩擦系数为0.2,人施于木箱的力和水平方向成45°角.设木箱作匀速运动, 在下列两种情况下,求人施于木箱的力:(1)人推木箱;(2)人拉木箱. 答案:(1) (2) (12分) 三、利用传动皮带,把电动机和车床主轴联结起来.电动机的转速为1500转/分, 车床的效率为62.8%.现在要削制200毫米直径的金属工件,设传动速比为 1:5,所需的切削力为150千克重.求: (1)车床主轴的转速; (2)输入车床的功率是多少马力. 答案:(1) (2) (10分) 四、用内直径为0.30毫米的均匀细玻璃管制成酒精温度计,它的刻度范围是— 50℃到+50℃,在0℃时酒精的体积是157〔毫米〕3.这支温度计最高到最低 刻度间的长度是多少厘米?(酒精的体胀系数是0.00111/度,玻璃膨胀的影 响可忽略不计.) 答案: (8分) 五、一端封闭的均匀玻璃管,充满水银,倒立在一个深的水银槽中,然后注入一 些乙醚(如图).已知大气压强是75厘米高水银柱,在下列各种情形下,管内 汽体的压强各是多少厘米高水银柱? (1)温度为t,管内存有液体,h1=50厘米,h2=30厘米; (2)温度保持t',将管向上提高,管内仍有液体,h1=62厘米; (3)温度增加到t',管内液体已完全汽化,h1=62厘米,h2=26厘米; (4)温度保持t,再将管向上提高,h1=75厘米. 答案:(1) (2) (3) (4) (12分) 六、如图,在带电的平行板电容器间有一个质量是10-6克、带+9.8×10-6静电系 单位电量的尘粒P.已知电容器极板间的距离为2厘米,如果该尘粒所受的静 电力恰与重力平衡,求电容器极板间的电势差,并指出哪一个板带正电. 答案: ; (8分) 七、如下左图,A、B、C是三个具有相同电阻的小电灯,设电源的内电阻可以忽 略不计,求: (1)电键K打开时和闭合时A灯消耗功率之比; (2)电键K打开时和闭合时B灯消耗功率之比; (3)由此说明原来打开的电键K闭合后,A、B二灯的明暗情况各有何变化. 答案:(1) (2) (3) (10分) 八、在蹄形磁体的二极间,放置一个线圈ABCD(如上右图).当蹄形磁体绕OO'轴 线转动时,线圈将发生什么现象?为什么? 答案: (10分) 答案: (8 分) 十、一束红色光,在真空中的波长是6000埃,射入玻离中,计算它在玻璃中的波 长和频率.玻璃的折射率是1.5. 答案: ; (6分) 十一、原子堆有哪些主要组成部分?试列举名称并说明它们的作用. 答案: (10分) 1959年答案 一、(1)在高山上重力加速度减小,摆动周期加大,故变慢; (2)上移摆锤,减小摆长,可使摆动周期恢复原值,从而计时准确. 评分标准:全题6分. 第(1)小题4分.结果正确,未说理由,只给1分;结果正确,说明理由中 未提到g的变化,扣1分;理由正确,但结果写错,扣1分. 第(2)小题2分.结果正确,未说理由,给1分. 二、(1)21.2千克重; (2)14.1千克重. 评分标准:全题12分. 第(1)小题6分.正压力计算错误,扣3分;单纯运算错误,扣1分;单位 错误,扣1分. 第(2)小题6分.正压力计算错误,扣3分;单纯运算错误,扣1分;单位 错误,扣1分. 三、(1)5转/秒(或300转/分); (2)10马力. 评分标准:全题10分. 正确算出第(1)小题车床的转速,给2分. 第(2)小题8分.切削速度错误,扣2分;输出功率错误,扣4分;前两步 正确,但未算出输入功率,扣2分. 单纯运算错误,共扣1分;单位错误,共扣1分. 四、24.4厘米. 评分标准:全题8分. 只求出体积的增量,给5分. 单纯运算错误,扣1分,单位错误,扣1分. 五、(1)45厘米高水银柱; (2)45厘米高水银柱; (3)49厘米高水银柱; (4)42厘米高水银柱. 评分标准:全题12分. 第(1)小题2分. 第(2)小题4分. 第(3)小题2分. 第(4)小题4分.单纯运算错误,扣1分. 六、200伏特;下板带正电. 评分标准:全题8分. 电势差部分5分;另一部分3分. 单纯运算错误,扣1分;单位错误,扣1分. 七、(1)9:4; (2)9:16; (3)A灯变暗,B灯变亮. 评分标准:全题10分. 第(1)小题5分.正确列出总电流算式,但将总电流当作分路中的电流 计算,扣3分. 第(2)小题3分. 第(3)小题2分. 每小题单纯运算错误,各扣1分. 八、线圈将跟随磁体转动(或与磁体同方向转动).因为磁体的转动使线圈中产 生感生电流,根据楞次定律,磁场与感生电流的相互作用要阻止磁体与线圈 的相对运动,所以线圈就跟随磁体转动. 评分标准:全题10分. 只答跟随转动,未答原因或答错,不给分;答出跟随转动,在说原因时, 只提到产生感生电流,给4分;再提到磁场与感生电流的相互作用,给6 分;只答转动,未提跟随(或同方向),扣2分. 九、透镜应放在发光体与光屏之间,可以有两个位置: 评分标准:全题8分. 能正确列出算式,但未算下去,给3分;算出两个位置,但有单纯运算错 误,扣1分;算出两个位置,但答案只选一个,扣3分. 十、波长4000埃(或4×10-5厘米); 频率5×1014赫兹. 评分标准:全题6分. 只算出光在玻璃中的波长,给3分;只算出光在玻璃中的频率,给3分; 单纯运算错误,扣1分;单位错误,扣1分. 十一、至少包括下列五项: (1)铀条,其中的铀235核裂变时释放出原子能. (2)石墨(答重水亦可),是减速剂.使中子速度减低,以便被铀235俘获, 维持链式反应. (3)镉棒(答能大量吸收中子的物质亦可),能吸收中子,用来控制反应的 快慢. (4)水(答流通的液体亦可),吸取释放出来的能量,传输到原子堆的外 面. (5)保护层,吸收原子堆放出的强烈射线,借以保护工作人员. 评分标准:全题10分 不答铀条,不给分;不答石墨、镉棒、水、保护层各扣2分;每项未答 作用,各扣1分;答了这五项以外的,无论正确与否,不影响评分. 1960年试题 一、12.5吨重的电车,在水平道路上由静止开始运动,经过30秒钟速度达到6米/ 秒.把电车的运动当作匀加速运动,设阻力等于车重的0.02,求电车的牵 引力. 答案: (12分) 二、设人造地球卫星离地心的距离为r,以地心为圆心作匀速圆周运动.已知 答案: (6 分) 三、用焦距是5厘米的放大镜观察物体.为了使所成的虚像距放大镜25厘米,物 体应放在什么地方?画出成像的光路图. 答案: (10分) 四、在下列四题中任意选作三题:(如四题都作,必须划去一题;否则,按(1)、 (2)、(3)题给分.) (1)单色光从空气进入水中.它的波长、频率和传播速度,什么改变?什么保 持不变? 答案: (2)把红外线、伦琴射线、无线电波、紫外线和可见光线,按照频率由大到 小的次序排列出来. 答案: (3)下图是用来开动或停止电动机的光控替续器的示意图.试简要说明其 工作原理. 答案: (4)已知氢原子的核外电子在第二条轨道上运动时的能量是E2,在第三条 轨道上运动时的能量是E3,普朗克恒量是h.试求氢的核外电子从第三条 轨道跃入第二条轨道时所发出的光的频率. 答案: (9分) 五、试简要回答下列问题: (1)原子堆中为什么要用石墨和镉棒? 答案: (2)什么叫做热核反应? 答案: (8分) 六、用效率为80%的锅炉来生产过热蒸汽.送入锅炉的水的温度是100℃,所产生 的过热蒸汽的温度是290℃.水的比热取1卡/克·度,在这个锅炉中水的沸 点是190℃,汽化热是480卡/克,过热蒸汽的比热取0.5卡/克·度.问每生 产560千克过热蒸汽要消耗多少煤.(煤的燃烧值是7000千卡/千克.) 答案: (10分) 七、在下列二题中任意选作一题:(如二题都作)必须划去一题;否则,按(1)题给 分. (1)有一个气焊用的氧气筒,容积为100升.在温度为15℃时,筒上压强计所 指出的压强是96大气压,求筒里氧气的质量.(在标准状况时每升氧气的 质量是1.43克). (2)长江大桥的钢梁一端固定,另一端是自由的.这是为什么? 如果在-10℃时把两端都固定起来,当温度升高到40℃时,钢梁所承担的 胁强是多少? (钢的线胀系数为12×10-61/度,弹性模量为2.0×104/千克重/[毫 米]2). 答案: (10分) 八、绘出矿石收音机的线路图,并指出各主要部分的作用. 答案: (8分) 九、如何测定干电池的电动势和内电阻?在回答中应绘出线路图,标明图中各部 分器材的名称,简要说明实验步骤,并写出最后计算式. 答案: (12分) 十、一个直流电动机的输入电压是220伏特,电枢的电阻是0.3欧姆.正常 运转 时,通过电枢的电流强度是100安培.问: (1)如果起动时通过电枢的电流强度不能超过150安培,应使用多大的起动 电阻? (2)正常运转时电动机的反电动势是多少? (3)正常运转时电动机的输出功率是多少? (4)用这台电动机来带动效率为70%的起重机,以0.3米/秒的速度匀速地提 起货物,能提起的重量是多少? 答案:(1) , (2) , (3) , (4) .(15分) 1960年答案 一、约5000牛顿. 评分标准:全题12分;加速度4分,牵引力8分. 计算加速度中单位错误,扣1分. 计算牵引力中单位错误扣3分;单纯运算错误扣1分;只写出方程 F-f=ma,未继续运算下去的,给3分. 二、评分标准:全题6分. 三、约4.2厘米. 评分标准:全题10分;计算5分,作图5分. 计算中像距符号错误,不给分;单位错误,扣1分;单纯运算错误, 扣1分. 作图中虚线部分画成实线,不扣分;光路中完全未画箭头,扣1分. 四、(1)频率不变,波长和传播速度改变. (2)伦琴射线、紫外线、可见光线、红外线、无线电波. (3)a.当光照射光电管时,产生电流,这个电流经过放大器放大后使电磁 铁M磁化,M吸引衔铁N,从而断开电动机电路. b.当光停止照射时,电磁铁失去磁性,弹簧使电动机的电路接通. 评分标准:全题9分,每小题3分. (1)频率、波长和传播速度中答错一个,全不给分. (2)任何一个的次序排错,全不给分;如果按频率从小到大的顺序 排列而未加说明的,扣2分. (3)只答出"答案"中的a部分,给全分;在产生电流、经过放大、电 磁铁的磁化三者中,每少答一个,就扣1分;超出"答案"的回答, 无论正确与否,均不影响评分. (4)回答与"答案"不符的,不给分. 五、(1)石墨用来作中子的减速剂. 镉棒用来吸收中子,以控制反应快慢. (2)在温度足够高时,轻原子核具有足够的动能来克服原子核间电的斥力, 于是发生聚变.这种反应叫做热核反应. 评分标准:全题8分,每一小题4分. (1)石墨、镉棒各2分. 只答石墨作减速剂,未说明是作中子的减速剂,扣1分. 只答镉棒吸收中子,未答用来控制反应快慢,扣1分;只答镉棒用来控制反 应快慢,未答吸收中子,扣1分. (2)未答温度足够高的,扣2分;未答轻原子核发生聚变或只答原子核发生 聚变的,扣2分. 超出"答案"的回答,无论正确与否,均不影响评分. 六、62千克. 评分标准:全题10分. 漏算水吸收的热、汽化热、蒸汽吸收的热三者中的任何一项,扣5分;效率 乘除用错,扣3分;这两者都错,全题不给分. 单位错误,扣1分;单纯运算错误,扣1分. 七、(1)约13千克. 评分标准:全题10分. 只算出氧气在标准状况下的体积或题设状况下的密度,给6分. 未用绝对温度来计算而造成错误,扣2分. 单纯运算错误,扣1分,单位错误,扣1分. (2)a.为了避免由于钢梁的热胀冷缩而产生有害的胁强. b.12千克重/〔毫米〕2. 评分标准:全题10分;a.2分,b.8分. 在a的回答中,把"有害的胁强"答成"破坏性的胁强",把"胁强"答成"应力 "、"压强"等,只要意思正确,均不扣分;超出"答案"的回答,无论正确与否,均不 影响评分在b的回答中,如只 八、 1.可变电容器和线圈组成的线路起调谐作用. 2.矿石起检波作用. 评分标准:全题8分;线路图4分,回答4分. 线路图中,未正确画出谐振线路或未画出矿石的,不给分;未画出天地线的, 扣2分;未画出听筒的,扣2分;电容器中未画出可变符号亦未注明其电容是可变 的,扣1分. 线路图与"答案"不一样,只要能谐振、检波和收听的,同样给分. 回答中未说明谐振线路的作用或矿石的作用,各扣2分. 线路图和回答中超出"答案"的部分,无论正确与否,均不影响评分. 九、凡是能近似地测出ε和r的实验,都算正确答案. 评分标准:全题12分;线路图和标明器材名称4分,实验步骤5分,最后计算 式3分. 在线路图中,未画电键、未标明导线的,均不扣分;但从线路图中看出电路 的接法会损坏仪器的,例如安培计短路等,全题不给分. 未绘线路图,未写实验步骤,只写出计算公式的,即使运算正确,也全题不 给分. 绘出线路图,未写实验步骤,写出计算公式的,即使运算正确,也只评线路 图部分的分数. 实验步骤的叙述完整、基本正确,给5分;其中如包含不必要的步骤和测量 不必要的数据,均不扣分;步骤不完整(例如没有测出全部必要数据)的,扣3分. 计算部分如已正确写出原始方程并指出求ε和r的途径,虽未算出最后表 达式,亦不扣分. 如果只答出用伏特计直接测出ε,给3分. 十、(1)约1.2欧姆; (2)190伏特; (3)19千瓦; (4)约4.5吨重. 评分标准:全题15分;(1)4分,(2)4分,(3)3分,(4)4分. 每小题中的单位错误,各扣1分;单纯运算错误,各扣1分;但因前 面算错而影响后面计算结果时,不重复扣分. 1961年试题 一、从光源发出的光投射到一个焦距为f的凹面镜上,所成的像和光源在镜的同 侧,光源到镜的距离为像距的4倍.求像所在的位置,并画出成像的光路图. 答案: (8分) 二、在下列两题中任意选作一题:(如两题都作,必须划去一题;否则,按题1给 分.) 1.为了简便地称量一根较重的粗细不均匀的木料,使左端着地,抬起它的右 端时,用的力是32千克量;使右端着地,抬起它的左端时,用的力是48千克 重.问:(1)这根木料的重量是多少千克重?(2)它的重心离左端的距离是 全长的几分之几? 答案:(1) ;(2) (8分) 2.氢原子中的电子绕原子核作圆周运动的速度是2.2×8厘米/秒,求这时 电子轨道的半径,(电子的质量和电量分别是9.1×10-28克和4.8×10-10静 电系单位.) 答案: (8分) 三、回答下列问题: 1.在如图所示的电路中,怎样用一个伏特计测出电源的路端电压,并近似地 测出它的电动势?在图上画出联接上的伏特计,并注明伏特计正负接线柱 的符号. 答案: (8分) 2.在测定物质比热的实验中,使用天平称量物体的质量时,必须先对天平进 行调整.问:(1)怎样判断天平底座是否水平?如果不水平,应该调整什 么?(2)如果指针不指零点,偏左怎样调整?偏右怎样调整? 答案: (4分) 四、在直流电路里串联着A、B两个安培计.把0.015欧姆的电阻和安培计A并联, 这时安培计A的示数是0.4安培,安培计B的示数是1.2安培.求安培计A的电 阻是多大. 答案: (8分) 五、在下列两题中任意选作一题:(如两题都作,必须划去一题;否则,按题1给 分.) 1.水平放置的平行板电容器,两板间的距离是2厘米,两板间的电势差是180 伏特,上板带正电.一个电子由水平方向射入两板间.(1)求电子所受电场 力的大小和方向(电子的电量是4.8×10-10静电系单位).(2)说明电子在 水平和竖直方向上的分运动各是什么样的运动. 答案: (10分) 2.回答下列问题: (1)什么叫做裂变和链式反应? (2)什么叫做聚变?为什么聚变必须在几百万度以上的高温下进行? 答案: (10分) 六、有一个一端封闭的粗细均匀的细玻璃管,用一段长为16厘米的水银柱封入 适量的空气,如图所示.这个装置可以用来测定大气压强.把管竖直放置: 开口向上时,管内空气柱长是15厘米;开口向下时,管内空气柱长是23厘 米.求: (1)这时的大气压强; (2)把管水平放置时,管内空气柱的长度. 七、一台四缸四冲程的内燃机,活塞面积是300(厘米)2,活塞冲程是300毫米. 在第三冲程中,活塞所受的平均压强是4.5千克重/(厘米)2.在这个冲程中, 燃气所做的功是多少?如飞轮转速是300转/分,这台内燃机燃气做功的功率 是多少马力? 答案: (8分) 八、回答下列问题: 1.一束白光从真空射入玻璃,已知红光在玻璃中的传播速度大于紫光在玻 璃中的传播速度,向红光和紫光的折射角哪个大.为什么? 答案: (4分) 2.在真空中频率是5×1014赫兹的色光,在水中传播时,它的波长是多 大?(水对这种色光的折射率是4/3,光在真空中的传播速度是3×108米/ 秒.) 答案: (4分) 九、在一台直流电动机,它的电枢线圈的电阻是0.6欧姆.把这台电动机接到电 压是120伏特的电路中.这时,电动机的效率是95%.求电动机的输入功率. 如果加在电动机上的电压保持不变,当电动机轴上的负载增加时,电流强 度有什么改变?为什么?这时电动机的效率有什么改变?为什么? 答案: (12分) 十、用一塔式起重机从地面提起2吨重的货物.货物竖直上升的加速度是0.4米/ 秒2.问:(1)起重机的钢绳所受的拉力是多少牛顿?(2)如起重机的效率是 80%,货物做匀加速上升的头五秒内,供给起重机的能量是多少焦耳?(重 力加速度是9.8米/秒2.) 答案:(1) ;(2) (14分) 1961年答案 评分标准:全题8分.计算4分;作图4分. 计算中正负号错误,不给分.单纯运算错误,扣1分. 图中未画出光路进行方向的,扣2分. 设木料的长度是L,重心离左端的距离是x,则: 32L=Px, 48L=P(L-x); ∴ p=80千克重. 评分标准:全题8分.(1)4分;(2)4分. (1)中:直接相加得出答案的,不扣分.未注或注错单位的,扣1分; 单纯运算错误,扣1分. 2.约0.52×10-8厘米. 评分标准:全题8分. 未注或注错单位的,扣1分;单纯运算错误,扣1分. 三、1.如图所示,电键开启时,伏特计的示数是电源的电动势;电键关闭时,伏特 计的示数是路端电压. 评分标准:全题8分.电动势和路端电压各占4分. 只在图上画出伏特计的联法,而未加说明的不给分. 说明正确,仅伏特计正负接线柱接错或未注明正负接线柱符号的, 扣2分. 2.(1)悬锤尖端与底座上固定尖端对正时,则底座水平;否则调整底座上的螺 旋. (2)指计偏左,则旋转横梁上的调整螺旋,使它向左移;指针偏右,则使它 向右移. 评分标准:全题4分.(1)2分;(2)2分. (1)中答成用水准仪判断底座是否水平且回答正确的,不扣分.回 答内容正确,但天平零件名称与答案不一致的,不扣分. 四、0.03欧姆. 评分标准:全题8分. 只列出正确算式,而未进行运算的,给4分;只算出1的,给2分. 单位错误,扣1分;单纯运算错误,扣1分. 五、1.(1)1.44×10-10达因,方向竖直向上. (2)水平方向作匀速运动,竖直方向作初速为零的匀加速运动. 评分标准:全题10分.(1)6分;(2)4分. (1)中:电场强度和电场力各占3分.只列出正确算式,而未进行运 算的,给3分. 未注明电场力的方向或方向答错的,扣1分;单位每错一个扣1 分;单纯运算错误,扣1分. (2)中:竖直方向和水平方向各占2分. 竖直方向只答匀加速运动的,可不扣分;但答加速运动的,扣1 分. 2.裂 变:重原子核受中子打击分裂为轻原子核. 链式反应:裂变时,同时放出中子,这些中子又被重核俘获,使裂变不断 进行. 聚 变:轻原子核聚合成为较重的原子核. 聚变条件:温度高,原子核的动能大,才能克服原子核间的斥力,达到发 生核反应的程度. 评分标准:全题10分.(1)5分;(2)5分. (1)中:裂变占2分,链式反应占3分. 只画出链式反应图而未加说明的,可不扣分.答裂变时,未提 中子打击的,可不扣分. 答裂变时未指出重原子核分裂的,扣1分. (2)中:聚变占2分;聚变条件占3分. 答聚变时未指出轻核聚合的,扣1分. 答聚变条件时,只答出温度高原子核动能大的,给1分. 六、(1)76厘米高水银柱;(2)18厘米. (1)(p0+16)×15=(p0-16)×23, ∴ p0=76(厘米高水银柱). (2)(76+16)×15=76×h, ∴ h=18(厘米). 评分标准:全题12分.(1)8分;(2)4分. (1)中:只列出正确算式而未进行运算的,给5分;只写出开口向上 和开口向下时,管内空气压强为p0+16和p0-16的,给2分; 只写出其中一个的,不给分. (2)中:只列出正确算式而未进行运算的,给2分.全题中,单位每 错一个扣1分;单纯运算错误每错一个扣一分;因前一步运 算错误而引起后一步运算错误的,不重复扣分. 七、405千克重·米;54马力. W=pSL=4.5×300×0.3=405(千克重·米). 评分标准:全题8分.功占4分;功率占4分. 每部分只正确列出算式,而未进行运算的,各给2分. 功率单位未化成马力的,扣1分;其它单位错误,扣1分;单纯运算 错误,扣1分. 2.4500埃(4.5×10-5厘米). 评分标准:全题8分.1.4分;2.4分. 2中:只正确列出算式,而未进行运算的,给2分;单位错误,扣1分; 单纯运算错误,扣1分. 九、1200瓦特. 电动机轴上负载增加时,电枢的转速减小,反电动势随着减小,根据 评分标准:全题12分.计算部分6分;问答部分6分. 计算部分中只算出电流的,给4分, 单位错误,扣1分;单纯运算错误,扣1分. 问答部分中,每小部分各占3分.回答内容正确,措词与答案不一 致的,不扣分. 十、(1)20400牛顿;(2)127500焦耳. 评分标准:全题14分.(1)6分;(2)8分. (1)中:能正确列出算式,而未进行运算的,给3分. 在运算过程中,由于各量采用不同单位系统,而引起错误的,扣2分;未注或 注错单位的,扣1分;单纯运算错误,扣1分. (2)中:路程占3分,功和能占5分;能正确列出算式,而未进行运算的,给4分; 如分步运算,只列出路程算式,而未进行运算的,给1分. 遗漏效率或运算时把效率乘除算错的,扣2分;单位每错一个扣1分;单纯运 算错误,扣1分;由前一步错误而引起后一步错误的,不重复扣分. 1962年试题 一、一个平行板电容器的电容C=0.003微法拉,两板的距离d=0.02厘米,把两板 分别接在电动势ε=6伏特的电源两极上.求: (1)电容器所带的电量Q是多少; (2)两板间的电场强度E是多少. 答案:(1) ; .(8分) 二、硼(5B10)在俘获-粒子后嬗变为具有放射性的氮,同时放出一个中子;放射性 的氮又放出一个正电子而蜕变为碳的同位素.试分别写出硼的嬗变和氮的 蜕变的方程. 8分 三、把下列各题的答案填入题中的括弧内: (1)在100℃时水的饱和汽压等于( ). (2)什么叫做空气的绝对湿度?( ) 什么叫做空气的相对湿度?( ) (3)有甲、乙、丙三个单摆,它们的摆长和质量如图所示.如果甲摆的周期等 于2秒,那么乙摆的周期等于( ),丙摆的周期等于( ). (4)从分子运动论的观点来看,温度是( )标志. 12分 四、已知紫色光的频率大于红色光的频率.问玻璃对于哪一种色光的折射率 大?( )一束平行的白光通过玻璃棱镜,红色光与紫色光的偏向角哪一个 大?( )一束平行的白光通过一个凸透镜后,红色光和紫色光分别会聚在 不同点,哪一种色光的会聚点离透镜较远?( )把答案写在括弧内. 8分 五、已知一个透镜所成物体的像是倒立的,而且像的长度是物体长度的2倍.问: (1)像是实像还是虚像? (2)透镜是凸的还是凹的? (3)如果物体和像之间的距离为90厘米,透镜的焦距等于多少? (4)根据透镜成像的作图法画出光路图. 答案:(1) ;(2) ;(3) (10分) 六、用下图所示的装置测定水的汽化热时, (1)如果在A中的水沸腾以前就把D管插入量热器的水中,实验结果会很不 准确.为什么? (2)需要记录哪些温度,什么时候记录? (3)怎样测出量热器中凝结成水的蒸汽的质量? 10分 七、由高度h=30米处水平抛出一个物体,物体的质量m=20克,初速度V0=20米/ 秒.若物体落地时速度V=30米/秒,求物体克服空气阻力所做的功.(设重力加 速度g=10米/秒2) 答案: (8分) 八、在光滑的水平桌面A上叠放着两个静止的物体B和C,如下图所示.B的质量 mB=500克,C的质量mC=100克.B和C之间有摩擦. 设以F=60克的力沿水平方向向左拉物体B,若B、C两物体仍保持相对 静止而一起运动,求B给C的摩擦力f的大小和方向. 答案: (12分) 九、下图电路中,M为一直流电动机,它的电枢的电阻r=1.5欧姆;跟电动机串联的 电阻R=8.5欧姆,电源的电动势ε=41伏特(电源的内电阻忽略不计).当电 动机转动时,伏特计的示数V=24伏特(通过伏特计的电流忽略不计).求: (1)通过电动机的电流强度; (2)输入到电动机的电功率; (3)转变成机械能的电功率. 答案:(1) ;(2) ;(3) (12分) 十、如下图所示,有一水平的匀强磁场(磁场的方向指向读者).在垂直于磁场方 向的竖直面内放一矩形金属框,框的一边AB可无摩擦地上下滑动(滑动时 AB仍保持水平). (1)在图中画出AB边下落时框中电流的方向. (2)如果AB边匀速下落,试用下列数据求出下落的速度:AB边的质量m=0.2 克,长度l=10厘米,AB边的电阻R=0.2欧姆(框的其他三边的电阻可忽略),磁 场强度H=1000奥斯特,重力加速度g=1000厘米/秒2,空气阻力不计. 答案: (12分) 1962年答案 一、(1)1.8×10-8库仑;(2)1静电系单位. (1)电容器两板间的电势差等于电源的电动势ε,故Q=Cε=0.003×10-6 ×6库仑=1.8×10-8库仑. 评分标准:全题8分.(1)4分;(2)4分. 每问中只列出关系式的,各给2分. 单位每错一个扣1分. 单纯运算错误,共扣1分. 二、5B10+2He4—→7N13+0n1 7N13—→6C13+β+ 评分标准:全题8分.每一方程4分. -粒子、中子、正电子和各元素符号写错的,各扣1分. 正电子写做e+或1e0的,不扣分. 每一方程中,质量数不平衡的,扣2分,电荷数不平衡的,扣2分. 由前一方程错误而引起后一方程错误的,不重复扣分. 每一方程所扣分数不超过4分. 三、(1)(76厘米高水根柱)或(1标准大气压). (2)(空气里所含水汽的压强叫做空气的绝对湿度) (某温度时,空气的绝对湿度跟同一温度下饱和水汽压的百分比叫做当时 空气的相对湿度). (3)乙摆的周期是(2秒),丙摆的周期是(1秒). (4)(分子平均动能). 评分标准:全题12分.(1)2分;(2)4分;(3)4分;(4)2分. (1)中:答做1大气压的,不扣分. (2)中:每一答案2分. 绝对湿度答做单位体积空气中所含水汽的质量,或空气中所含水汽的密度 的,不扣分. 相对湿度中,漏掉上面答案中"同一温度下"字样的,不给分;答作"空气的绝 对湿度跟同一温度下饱和水汽压的比,叫做相对湿度"的,不扣分. (3)中:每一答案2分. (4)中:答作"分子动能"、"分子平均速度"或"分子速度"的,扣1分. 四、(紫光)(紫光)(红光) 评分标准:全题8分.2分;2分;4分. 第二问答错而第三问答对的,第三问不给分. 五、(1)实像;(2)凸透镜;(3)20厘米. (4)光路图:(画出三条线中任意两条即可.) (3)设物距为u,像距为v,透镜的焦距为f; 评分标准:全题10分.(1)2分;(2)2分;(3)3分;(4)3分. (3)中:只正确算出u,v的,给1分. 单纯运算错误,扣1分.单位漏写扣1分. (4)中:图中未画出光路进行方向的,扣1分;方向未画全的,不扣分. 六、(1)这样将有不到沸腾温度的蒸汽进入量热器,以致影响实验结果. (2)通入蒸汽以前,记录量热器中水的温度;取出D管后(或停止通蒸汽后), 记录量热器中水的最高温度. (3)称出通蒸汽前后量热器小筒及其中的水的质量,求出两者之差. 评分标准:全题10分.(1)4分;(2)4分;(3)2分. (2)中:每问2分;每问中每一温度各1分. (3)中:答作:"称出通蒸汽前后水的质量之差"的,不扣分,超出上面"答案"的 回答,无论正确与否,均不影响评分. 七、1焦耳(或107尔格). 设克服空气阻力所做的功为A,则 评分标准:全题8分. 只列出关系式:关系式正确的,给3分;关系式中有正负号错误的,给1分. 关系式中有正负号错误而算出结果的,给2分. 关系式中缺任何一项的,不论有无计算结果,不给分. 关系式中A的正负号和上面"答案"中相反,但说明A系表示空气阻力对物体 的功的,不扣分. 关系式正确,而数值代错算出结果的,扣3分. 单纯运算错误,扣1分. 单位错误扣1分. 八、10克(或9800达因,10,000达因),f向左. 设两物的共同加速度为a,则 评分标准:全题12分.f的大小8分;方向4分. f的大小部分:只正确算出加速度的,给4分. 只正确列出关系式的,每式2分. 用隔离法计算的,按同样标准评分. 单纯运算错误,扣1分. 单位错误扣1分. f的方向部分:答作f与F方向相同,或f与C的运动方向相同,均可,在图上正确 画出f的方向并标出f符号的,作为正确答案.图上画出方向,又有文字答案的,以 文字答案为准. 九、(1)2安培;(2)48瓦特;(3)42瓦特. (1)设通过电动机的电流为I, (2)设输入电动机的电功率为N, N=IV=48瓦特. (3)设转变为机械能的电功率为N' 第一法:N'=N-I2r=42瓦特. 第二法:N'=Iε'=I(V-Ir) =N-I2r=42瓦特. 评分标准:全题12分.(1)7分;(2)2分;(3)3分. (1)中:只正确列出关系式的,给4分. (2)中:只正确列出关系式的,给1分. (3)中:只正确列出关系式的,给2分. 由前一步错误而引起后一步错误的,不重复扣分. 单纯运算错误,共扣1分.单位每错一个扣1分. 十、 (1)电流方向如图;(2)400厘米/秒. (2)设电路中电流强度为I,根据磁力与重力平衡, 0.1HIl=mg, 设电路中感生电动势为ε', ε'=IR=0.04伏特. 设下落速度为v, ε'=10-8Hlv, 评分标准:全题12分.(1)2分;(2)10分. (2)中:根据0.1HIl=mg,求I部分:5分;只正确列出式子,给3分. 根据ε'=IR,求ε'部分:2分;只正确列出式子,给1分. 根据ε'=10-8Hlv,求v部分:3分;只正确列出式子,给2分. 单纯运算错误,共扣1分.单位每错一个,扣1分. 1965年试题 一、在下图所示的杆秤中,挂钩K到提钮O的距离为8厘米,秤杆(包括挂钩和提钮) 的重心G到O点的距离为2厘米,秤锤的重量P=2市斤.当称量3市斤重的物体 时,秤锤的悬挂点A到O点的距离为11厘米.求出: (1)秤杆(包括挂钩和提钮)的重量; (2)此时手提提钮的力的大小.(本题14分) 答案:(1) ;(2) 二、某人民公社购到一台水泵,此泵每秒钟可把80公斤的水提高10米,水泵的效 率为80%. (1)求此水泵抽水时所需的功率. (2)现在有一台功率为10千瓦、效率为85%的电动机,此电动机能否带动这 台水泵抽水? (本题14分) 答案:(1) 三、某锅炉每小时产生1000公斤、350℃的过热蒸汽.如果供给锅炉的水的温 度是90℃,锅炉内的压强是15.9大气压,从下表查出所需数据,并计算出锅炉 每小时供给的热量.(过热蒸汽的平均比热是0.5千卡/公斤) (本题15分) 不同温度时水的饱和汽压P(大气压) 不同温度时水的汽化热L(千卡/公斤) 答案: 四、参看下页的图.在均匀磁场中有一个长方形线圈,线圈的长和宽分别为L1和 L2,线圈按图中箭头所示方向以匀角速度ω绕垂直于磁场的OO'轴转 动,OO'轴平分bc和ad两边. (1)生产最大感生电动势m时,线圈平面和磁力线的方向所成的角度θ是多 大?这时线圈的哪些边产生感生电动势? (2)如果磁场强度H和转动角速度ω都不改变,而线圈的长由L1 改为 变前的m的比值. (本题15分) 答案:(1) ;(2) 五、用下图中所示器材测定串联干电池组的内电阻;干电池组的电动势E已知. (1)绘出这个实验所用的电路图. (2)列出所需的公式,并说明公式中哪些量需要在实验中读取. (3)仔细观察实物图后,把图中所示实物连成所需的电路,在所连的电路中,要 求变阻器的电阻是可调的. (4) 在你所连的电路中,接通电键前,变阻器的滑动接触片不应放在什么位 置? (用箭头在实物图上标出此位置)为什么? (本题18分) 六、如下页的图,用绳把一个重量为100公斤的重物挂在水平横梁上的滑环O上, 悬点到重物重心的距离为2米.用另一绳拉动滑环,使重物在水平方向达到2 米/秒的速度后作匀速运动. (1)求匀速运动时悬挂重物的绳的拉力是多少公斤. (2)如果滑环运动到A点突然停止,这一瞬时悬挂重物的绳的拉力又是多少 公斤? (本题12分) 答案:(1) ;(2) 七、如下页的图,把一凹透镜A放在一凸透镜B前6.5厘米处,能使一束平行于主 轴的光线经过两透镜后,会聚在光屏S上.如果把凹透镜拿走,保持凸透镜和 光屏的位置不变,这时把一个点光源C放在凸透镜的主轴上距这透镜15.6厘 米处,那么,C的像也成在此光屏上. (1) 在下图中,画出凹透镜拿走以前的光路图;并在此图中标出或说明 . (2)求出凹透镜的焦距f. (本题12分) 答案:(2) 做完前七题后,仔细检查一遍,如还有多余时间,可做第八题. 八、(附加题)下图中瓶A为盛有气体的容器,B为水银压强计.用这套装置按图 (一)、(二)、(三)的顺序进行操作,可以测出气体的压强系数γ.在实验过程中, 实验室中的水银气压计的示数始终未变,如图(四)所示. (1)比较图(一)和图(二),并说明瓶A中的气体的状态发生了什么变化. (2)为了测出气体的压强系数,需要把图(二)调节到图(三)所示的状况,说明 这样调节的目的是什么. (3)写出计算压强系数γ的公式;从各图中读取所需的数据后,逐个写出公式 中各量的数值;算出γ的数值. (本题15分) 答案:(3) 1965年答案 一、(1)1市斤;(2)6市斤. (1)设秤杆的重量为W,取O为支点, 得: 3×8=W×2+2×11 ∴W=1市斤. (2)设手提提纽的力为f, 得: f=1+2+3=6市斤. 评分标准:全题14分.(1)9分;(2)5分. (1)中:力臂有错误的,扣4分;力矩正负有错误的,扣5分. (2)中:如果未从(1)中求出W的数值,只写出f=2+3+W的,给4分.只 写出f=2+3=5市斤的,不给分. 如果取G为支点,利用力矩平衡先求出f,再求出W的,求f部分给9 分,求W部分给5分.运算中如有错误,参照上面的标准给分. 二、(1)9.8千瓦. (1)每秒扬出的水的质量m=80公斤/秒,升高的高度h=10米;故水泵需要的功 率为 mgh÷80%=9800焦耳/秒=9.8千瓦. (2)电动机的输出功率为 10×85%=8.5千瓦. 故不能带动. 评分标准:全题14分.(1)10分;(2)4分. 由于效率的概念在生产中比较重要,在本题中起的作用也比较大, 所以有关效率的计算,在本题中共占6分.各问的分数具体分配如 下: (1)中:由mgh求水泵的有用功率占4分;效率占3分,功率单位的换 算占3分(若换算是在(2)中进行的,则此3分移入(2)中). (2)中:求电动机的输出功率占3分;判定电动机能否带动水泵占1 分.其中由于计算错误而引起后面结论错误的,不扣这1分. 三、649×103千卡. 沸点t=200℃,汽化热L=464千卡/公斤. 设c、c'分别为水和蒸汽的比热,m为水的质量,t1为水的初温,t2为过热蒸汽 的温度,则热量 Q=cm(t-t1)+mL+c'm(t2-t) =649×103千卡. 评分标准:全题15分. (i)沸点与饱和蒸汽压的关系,汽化热与温度的关系,以及查表的能 力,是本题考查的一部分重点,所以,从表中正确地查出沸点和 汽化热,各占3分. (ii)Q=水吸收的热量+气化潜热+蒸汽吸收的热量,占9分.这部分 主要考查考生对水变为过热蒸汽需经过三个吸热过程是否 理解,三部分不宜割裂地给分,所以漏掉任何一部分热量的, 扣6分;漏掉两部分以上的,扣9分.运算中沸点或汽化热直接 代入错误数值的,除扣(i)中的分数外,不另扣分;若先已给出 沸点和汽化热的正确数值;只在运算式中代错的,共扣1分. 运算式中的其它量数值代错的,共扣1分. 四、(1)θ=0;ab边和cd边;(2)2. 或直接给出比例式: 评分标准:全题15分.(1)6分;(2)9分. (1)中:每问各占3分.此处不要求说明理由. 第二问中,仅答出ab或cd一边的,可能由疏忽而引起,故仍给2 分;答出ab边和cd边后,又多答了其他边的,说明该生对怎样才 算切割磁力线不清楚,故扣2分. (2)中: (i)仅孤立地写出一个原始公式,如: m=2×10-8HLυ,或υ=rω,而未联系本题的,不给分. (ii)仅知道m正比于ab边或cd边的长度的,给3分. (iii)知道m正比于v,而且知道改变线圈宽度前后,v不相同的, 给2分;又能正确运用线速度与角速度关系的,再给2分. (iv)在m和 的表达式中,比例常数有错的,扣1分. 五、(1)第一种画法: 第二种画法: (2)设电池组内电阻为r,电池端电压为V, E=V+Ir 或 E=IR+Ir V=IR. 需读取V,I. (3)(从略,接线的要求参考评分标准.) (4)滑动接触片不应放在使电池短路的位置. 评分标准:全题18分.(1)3分;(2)5分;(3)8分;(4)2分. (1)中:不要求注明电表及电池的正负,但若已注明正负,而有错误 的,扣1分.不画电键的,扣1分. (2)中: (i)计算公式,占3分.只列出E=IR+Ir的,给2分. (ii)在公式正确的基础上,知道该测哪些量,占2分.多答了需 读取外电阻R的,扣1分.公式错误,或不完全的,不给这2分. (3)中:电池连接有错的,扣2分. 伏特计的正负,或安培计的正负接错的,扣2分;两个电表的 正负都接错的,也扣2分. 导线接在滑动变阻器的两个固定头上的,扣2分. 电键接错,或未接入电路的,扣1分. 导线不在接线柱相接,而在空间相接的,扣1分. 考虑到伏特计和安培计的联法,是电学实验最基本的要求,伏特计 接成与变阻器串联,或安培计接成与变阻器并联的,扣5分.两个电 表都接错的,扣8分. 本小题所扣总分,不应超过8分. (4)中:标出滑动接触片位置,占1分,说明理由,占1分. 理由部分,若答成"变阻器的电阻不应为零"的不扣分. 六、(1)100公斤;(2)120公斤. (2)悬点停止的瞬时,重物以2米/秒的速度做圆周运动.设此时绳的拉力为F, 则 所以 评分标准:全题12分.(1)2分;(2)10分. (1)中:只要答出100公斤的,就给2分. (2)中: (i)向心加速度,占2分; (ii)向心力=m×向心加速度,占2分; (iii)拉力-重力=向心力,占4分.这一步主要考查考生 对牛顿第二定律的理解和运用,所以公式写错的, 向心加速度求错而引起后面错误的,不重复扣分. (iv)单位"公斤"和"牛顿"的换算,占2分.答案中以"牛顿"表 示的,不扣分. 七、(2)9.1厘米. (1) (2)f=15.6-6.5=9.1厘米. 评分标准:全题12分.(1)10分;(2)2分. 本题的特点与一般求焦距的题目不同,只有在先判断出F与C重合的基础上, 才能算出f,而且难点在前一步.所以两个小题的分数作如上的分配. (1)中: (i)光路图的实线部分,占4分.其中经过A的光线不发散,或经过B的光线不会 聚到光屏上的,均不给分;经过B的光线虽会聚在光屏上,但不在主轴上的,扣1分; 光线上未标出箭头的,不扣分. (ii)F点的位置,占6分.判断F点的位置,要求考生较灵活地掌握以下各点:逐 次成像,物、像的一一对应,凹透镜焦点的意义.本题考查的重点更多地放在前两 点上.只要在图上标出或说明了F与C重合,虽表达得不够完善,但已能表明考生 掌握了上述各点的,就给6分.在光路图中画出虚线交于C点,但未指出这交点就 是F的,表明考生对凹透镜焦点的意义不掌握,故只给4分.图中表示出虚线的交 点为F,但未指出F与C重合的,表明考生只掌握了凹透镜焦点的意义,故给2分. 八、(1)气体的温度、压强和体积都增加了. (2)为了保持气体的体积一定. 由图(四)读得大气压强P=75.6厘米高水银柱, 由图(一)读得0℃时气体的压强 P0=P-3.6=72.0厘米高水银柱, 由图(三)读得t℃时等容气体的压强 Pt=P+2.8=78.4厘米高水银柱, 及t=24℃. 将数据代入上式,即得γ=0.0037度-1. 评分标准:全题15分.(1)2分;(2)3分;(3)10分. (1)中:温度、压强和体积三者错一个的,扣1分,错两个的,不给分. (2)中:答成"为了缩小体积"的,可能是表达不确切,给1分.答成"为了提高水 银面"的,不给分. (3)中: (i)公式,占2分. (ii)正确使用压强计,是本实验的关键,所以读取压强数据,占7分.遗漏大气 压强的,扣6分.未从图中读取大气压强数据,而采用76厘米的,扣1分.考虑了大气 压强,但在计算P1或P0时加减号错的,扣3分.从图(二)读取Pt数据的,扣3分.水银 面位置读数有错的,共扣1分(要求写到毫米位).直接写出Pt与P0正确数值的,不 扣分. 压强单位写成"厘米"的,不扣分.扣分总数累计不超过7分. (iii)读取温度数据,占1分. 1979年试题 一、[33分]本题分11个小题.每小题提出了四上答案,其中只有一个答案是正确 的.选出你认为正确的答案,把它的号码填写在本小题后的圆括弧内.每小题 选出正确答案的,得3分;选错的,扣1分;不答的,不得分,也不扣分.每小题只许 选一个答案.如果选了两个答案,其中必然有错的,本小题扣1分. 1.通过一个电阻的电流是5安培.经过4分钟时间通过这电阻的一个截面的 电量是 (1)20库仑 (2)50库仑 (3)1200库仑 (4)2000库仑 答( ) 2.把电阻是1欧姆的一根金属丝截成等长的十段,把这十段金属丝并联起来, 这样并联的一组金属丝的总电阻是 (1)0.01Ω(2)0.10Ω (3)10Ω (4)100Ω 答( ) 3.单摆的周期在发生下述情况时将增大: (1)摆锤的质量增大 (2)摆长减小 (3)单摆从赤道移到北极 (4)单摆从海平面移到高山上 答( ) 4.放在光滑斜面上加速下滑的物体受到的力是 (1)重力和斜面支持力 (2)重力、下滑力和斜面支持力 (3)重力、斜面支持力和加速力 (4)重力、斜面支持力、下滑力和正压力 答( ) 5.光线透过空气中的平行平面厚玻璃板,问下图所示四种情形中哪一种是 正确的? 答( ) (1)α粒子 (2)质子 (3)中子 (4)β粒子 答( ) 7.把100克0℃的冰投入100克50℃的水中,混合时与外界无热量交换,达到 热平衡后的温度是 (1)25℃ (2)-15℃ (3)0℃ (4)4℃ 答( ) 8.一个平行板电容器充电后,把电源断开,再用绝缘的工具把电容器的 两金属板拉开一些.这使 (1)电容器中的电量增加 (3)电容器的电压不变 (2)电容器的电容增加 (4)电容器的电压增加 答( ) 9.一条绳能承受的最大拉力是100牛顿(超出此值,绳就被拉断).用这条绳拉 一个质量是2千克的物体在光滑的水平面上运动,绳和水平面的夹角是 60°.在绳不被拉断的条件下,物体的最大加速度可以达到 (1)12.5米/秒2 (2)25米/秒2 (3)43米/秒2 (4)100米/秒2 答( ) 10.质量是5吨的汽车在水平路面上以加速度a=2米/秒2起动,所受阻力是 1.0×103牛顿.汽车起动后第1秒末的瞬时功率是 (1)2千瓦 (2)11千瓦 (3)20千瓦(4)22千瓦 答( ) 11.一个长螺线管通有交流电.把一个带电粒子沿管轴射入管中,粒子将在 管中 (1)作圆周运动 (3)作匀加速直线运动 (2)沿轴线来回运动 (4)作匀速直线运动 答( ) 二、[10分]一架显微镜物镜的焦距f1=0.3厘米,目镜的焦距f2=2厘米,目镜和物镜 相距16厘米.如果从这显微镜观察到的像离目镜25厘米,被观察的物体离 物镜多远? 三、[10分]一个潜水艇位于水面下200米.艇上有一个容积V1=2米3的贮气钢筒, 筒内贮有压缩空气.将筒内一部分空气压入水箱(水箱有排水孔和海水相 连)排出海水10米3.此时筒内剩余气体的压强是95个大气压.设在排水过 程中温度不变,求贮气钢筒内原来的压缩空气的压强.(计算时取1个大气 压=1.0千克/(厘米)2,海水密度取1.0×103千克/米3) 四、[13分]有一质量是m的小球Ⅰ用长度是l的绳子悬挂在O点.把球Ⅰ拉到A 点 ,OA 是 水 平 线 , 如 图 所 示 . 另 一 质 量 相 等 的 小 球 Ⅱ 静 止 放 在 B 点 OB的中点).当在A点的小球Ⅰ从静止下落到B点时,跟小球Ⅱ作弹性碰撞使小 球Ⅱ沿轨道BC滑出(不考虑摩擦).求小球Ⅱ经过D点时对轨道的压力.(圆弧 轨道BD所对的圆心角θ=60°,m=1千克,g用10米/秒2计算) 五、[13分]有电路如图,R1=3000Ω,VA是内阻为6000Ω的电压表,VB是内阻为 3000Ω的电压表.已知: K1断开,K2接到A时,电压表的读数是4伏特; K1接通,K2接到A时,电压表的读数是8伏特; K1接通,K2接到B时,电压表的读数是7.5伏特. 求R2的值. 六、[13分]在加速行驶的火车上固定一斜面,斜面角是θ(见图).有一物体静止在 斜面上.如果火车加速度小于某一值a0,物体就会下滑.设物体和斜面间的 静摩擦系数是μ,推导a0的表达式. 七、[8分]本题分2小题.每小题提出了五个答案,其中只有一个答案是正确的.选 出你认为正确的答案,把它的号码填写在本小题后的圆括弧内.每小题选出 正确答案的得4分;选错的扣1分;不答的不得分,也不扣分.每小题只许选一 个答案.如果选了两个答案,其中必然有错的,本小题扣1分. 1.两条直导线互相垂直(如图),但相隔一个小的距离,其中一条AB是固定的, 另一条CD能自由活动.当直流电流按图中所示方向通入两条导线时,导 线CD将 (1)不动 (2)顺时针方向转动,同时靠近导线AB (3)逆时针方向转动,同时离开导线AB (4)顺时针方向转动,同时离开导线AB (5)逆时针方向转动,同时靠近导线AB 答( ) 2.一闭合线圈放在均匀磁场中,线圈的轴线与磁场方向成30°角,磁感应强 度随时间均匀变化.在下述办法中(如需要改绕线圈,用原规格的导线),用 哪一种办法可以使线圈中的感生电流增加一倍? (1)把线圈的匝数增加一倍 (2)把线圈的面积增加一倍 (3)把线圈的半径增加一倍 (4)改变线圈轴线对磁场的方向 (5)把线圈的匝数减少到原来的一半 答( ) 1978年试题 一、填空白 (1)当穿过一个线圈的( )发生变化时,线圈中产生感应电动势;感应电动 势的大小,除与线圈的匝数成正比外,还与( )成正比. (2)单摆在摆动过程中,其速度和加速度都是随时间变化的.从最大位移处向 平衡位置运动的过程中,速度越来越( ),加速度越来越( ). (3)在天然放射性元素的放射线中,已经查明,射线是( ),γ射线是 ( ). (4)在20℃的空气中,声音的传播速度是340米/秒.如果它的频率是100赫兹, 那么它的波长是( ). (5)两个点电荷之间距离为a,相互作用力为f;如果距离变为2a,则相互作用 力变为( ). (10分) 二、如图所示的电路中,三个电阻的阻值分别是R1=2欧姆,R2=4欧姆,R3=4欧姆. 电池的电动势E=4.2伏特,内阻r=0.2欧姆,求: (3)两个开关都接通时,通过R1的电流强度I1. (10分) 三、用照相机对着一个物体照相,已知镜头(相当于一个凸透镜)的焦距为13.5 厘米,当底片与镜头的距离为15厘米时,在底片上成5厘米高的像. (1)求物体的高; (2)绘出光路图. (13分) 四、一个安培-伏特两用表的电路如图所示,电流计G的量程是0.001安培,内阻 是100欧姆,两个电阻的阻值是R1=9900欧姆,R2=1.01欧姆. 问:(1)双刀双掷电键接到哪边是安培计,接到哪边是伏特计? (2)安培计、伏特计的量程各是多大? (13分) 五、一个14克重的比重计(如图所示),放在水中,水面在它的刻度A处;放在煤油 中,油面在它的刻度B处.已知煤油的比重d=0.8克/厘米3,比重计刻度部分的玻 璃管外半径r=0.75厘米,求AB之间距离. (14分) 六、一质量M=2千克的木块,放在高h=0.8米的光滑桌面上,被一个水平方向飞来 的子弹打落在地面上 (子弹留在木块中),落地点与桌边的水平距离S=1.6米,子弹的质量m=10克. (1)求子弹击中木块时的速度. (2)子弹射入木块时产生的热量,若90%被子弹吸收,子弹的温度能升高多 少?(设子弹的比热为0.09卡/克·度,取g=10米/秒2,空气阻力不计)(20分). 七、如图所示,一个U形导体框架,宽度l=1米,其所在平面与水平面交角α=30°, 其电阻可以忽略不计.设匀强磁场与U形框架的平面垂直,磁感应强 R=0.1欧姆,跨放在U形框上,并且能无摩擦地滑动.求: (1)导体ab下滑的最大速度vm; (2)在最大速度,vm时,在ab上释放出来的电功率. (20分) 1978年答案 一、(1)(磁通量),(磁通量的变化率) (2)(大) (小) (4)(3.4米) (5)(f/4) 评分标准:全题10分,每小题2分. (1)中,第二空白中,答为"磁通量变化"的,不给分. (3)中,上面每个空白给了两种答法,考生答任何一种或用适当的文字表达了 正确答案的,每个空白给1分.漏掉"流"字的,不扣分. 参考解法: 评分标准:全题10分,(1)3分,(2)3分,(3)4分. (3)中,公式中漏了内阻r的,不给分. 三、45厘米或0.45米 参考解法: (2)光路图(略) 评分标准:全题13分,(1)8分,(2)5分. (1)中,能列出两个文字方程的给2分.经过文字运算,得到物体高度的文字表 达式,并正确代入需要的数值的,再给4分.最后算出正确答案的,共给8分. 列出两个文字方程后,立即正确代入需要的数值的,给6分;正确算出答案 的,共给8分. (2)中,用透镜前面或后面的焦点都可以,同样给分.图中未用箭头表示光线 进行方向的,扣1分;图中尺寸完全不合比例的,扣1分. 四、(1)接到c,d上(或左边)是安培计;接到a,b上(或右边)是伏特计. (2)0.1安培,10伏特. 参考解法: (2)Igrg=IsR2 评分标准:全题13分,(1)5分;(2)两表的量程各4分,共8分. 五、≈2厘米,或0.02米 参考解法: V水×1=14, V油×0.8=14 评分标准:全题14分 能列出两个方程,表示没入液体中的体积乘液体比重等于比重计的重量 的,给7分;进一步表示出两个体积之差同刻度AB的距离的关系的,再给5分;计算 完毕无误的,给全分. 答数写"≈2厘米","=1.98厘米"或"=2厘米"都作为正确答案. 六、(1)804米/秒;(2)升高772度. 参考解法: (1)根据动量守恒定律,mv=(M+m)V. ① =3216(焦耳) =3216×0.24(卡) =772(卡) 子弹温度的增高△T由下式算出: 10×0.09×△T=772×0.9 △T=772度. 评分标准:全题20分,(1)10分,(2)10分. (1)中,列出动量守恒公式的,给3分. 如果分段进行数字计算,算出t值的,给3分,再算出V值的,再给2分.用动量 守恒定律再算出v的数值的,再给5分,共10分. (2)中,算出能量损耗数值的,给4分;换算成热量值的,再给2分;继续算出温度 升高度数的,共给10分.如果把温度升高的度数说成子弹的温度,扣3分. 七、(1)vm=2.5米/秒;(2)2.5瓦. 参考解法: (1)感应电动势ε=Blv; 评分标准:全题20分,(1)13分,(2)7分. (1)中,列出感应电动势公式的,给2分.列出电磁力公式的,再给4分.列出电磁 力同重力分量平衡的,再给5分.算出vm的数值的,共给13分. 1979年答案 一、1.(3) 4.(1) 7.(3) 10.(4) 2.(1) 5.(2) 8.(4) 11.(4) 3.(4) 6.(1) 9.(2) 评分标准:全题33分;每小题正确答案3分,错误答案-1分,不答的,不给分. 二、0.31厘米 参考解法: v2=-25厘米,f2=2厘米 ∴物体距物镜距离 u1=0.31厘米 评分标准:全题10分,(1)6分,(2)3分.(3)1分. (1)能正确利用公式计算目镜物距,给6分.只正确写出透镜公式,不知道目镜 的像是虚像,须用负数代入的,只给2分.如果说明是虚像,但未用负数代 入的,给3分.透镜公式写错但知道目镜所成的像是虚像,像距用负数计算 的,给2分. (2)求出物镜的像距,给3分.不会用显微镜结构来计算物镜的像距,不给分. (3)求出物镜的物距,给1分. 三、200个大气压. 参考解法: 设贮气筒内原有压缩空气的压强为P1,筒内剩余气体的质量为原有质量 的x倍. (1)潜水艇水箱中空气压强应等于该深度海水的压强,为 P2=hdg+P0 (P0)为海面大气压 〈1〉 =21(大气压) (2)对水箱中的空气, P2V2=21×10=(1-x)P1V1 〈2〉 对贮气筒内剩余气体, P3V1=95V1=xP1V1 〈3〉 〈2〉+〈3〉得 (P2V2+P3V1)=P1V1 〈4〉 P1=(21×10+95×2)/2=200(大气压) ∴贮气筒内原有气体的压强为200大气压. 评分标准:全题10分.(1)4分,(2)6分. (1)正确算出水箱中空气压强的,给4分.漏算海面大气压的,扣1分.只列出 <1>,没有代入数据计算的,只给2分. <1>式中没有写g,但计算正确的,不扣分.单位不配套的,扣1分. (2)正确算出贮气筒中原有气体压强的,给6分. 只列出〈2〉式、〈3〉式的,各给1分. 只导出或只直接写出〈4〉式的,给4分. 四、35牛顿 参考解法: V1是球Ⅰ达到B点时的速度. 〈1〉 (2)Ⅰ、Ⅱ两球在B点作弹性碰撞,由动量守恒和能量守恒得出球Ⅱ碰撞后的 速度VⅡ: VⅡ- VⅠ (3)球Ⅱ在D点的速度VⅡ'可以如下算出: (4)设P'是在D点轨道对球Ⅱ的压力,那么 =mg[3cosθ+2] =35牛顿 球Ⅱ对轨道的压力P同轨道对球Ⅱ的压力P'大小相等方向相反. ∴P=P'=35牛顿 评分标准:全题13分.(1)2分,(2)3分,(3)3分,(4)5分. (1)知道利用能量守恒并写出公式〈1〉,给2分. (2)知道m1=mⅡ的两球作弹性碰撞时,VⅠ=VⅡ,并求出VⅡ的表达式〈2〉的, 给3 分,根据能量守恒和动量守恒定律只写出公式而未算出VⅡ的表达式的, 只给2分. (4)知道求压力时考虑向心力和重力,得出正确答案,给5分.未说明轨道对球 的压力和球对轨道压力的关系的,不扣分. 扣去〈2〉中的3分,如只未说 明速度交换的,扣1分;如对能量守恒,动量守恒和速度交换作了文字说明, 虽未写出中间过程的公式,不扣分. 在分析压力时,知道要考虑向心力,但方向分析错误,或知道向心力,但对 重力未分解或分析错误,扣3分. 五、2500欧姆 参考解法: (3)设电源电动势是ε,内阻是r当K1闭合,K2接通A时 当K1闭合,K2接通B时 当K1断开,K2接通A时 (4)令(R3+r=R),解〈1〉、〈2〉得 ε=10伏特,R=500欧姆 代入<3>得 评分标准:全题13分.(1)1分,(2)1分(3)9分(4)2分. (1)计算R1与VA并联的总电阻,1分.公式错、运算错不给分,不写单位不扣分. (2)计算R1与VB并联的总电阻,1分.公式错、运算错不给分,不写单位不扣分. (3)正确列出方程〈1〉、〈2〉、〈3〉,给9分(每方程3分). 如果不把电压表的内阻计入电路参数,虽然形式列出三个方程,仍扣3 分.(因为不合物理原理). (4)算出R2,2分. 如果没有提到r或设r为零解题,同样给分. 参考推导: 设斜面上物体的质量是m,正压力是P. (1)在平行于斜面的方向上: ma0cosθ=mgsinθ-μP 〈1〉 (2)在垂直斜面的方向上: ma0sinθ=p-mgcosθ 〈2〉 (3)由〈1〉、〈2〉解得 评分标准:全题13分.〈1〉5分,〈2〉5分,〈3〉3分. (1)正确写出〈1〉式,给5分. (1)式有错误,但能正确写出这三项的大小的,给2分.漏掉一项的,不给分. (2)正确写出〈2〉式,给5分. (2)式有错误,但能正确写出这三项的大小的,给2分.漏掉一项的,不给分. (3)由〈1〉、〈2〉式解出〈3〉式的,给3分.答案中还出现正压力的,不给分. 按其他方向分解,例如在竖直和水平方向进行分解的,参照上述标准给分. 用惯性力概念分析(即把ma方向反转,然后按静力学的方法计算),得到结 果的也参照上述标准给分. 七、1.(5) 2.(3) 评分标准:全题8分;每小题正确答案4分,错误答案-1分,不答的,不给分. 1980年试题 一、(30分)本题分10个小题.每小题提出了四个答案,其中只有一个答案是正确 的.选出你认为正确的答案,把它的号码填写在本小题后的圆括弧内.每小 题选出正确答案的,得3分;选错的,得-1分;不答的,得0分.每小题只许选一 个答案.如果写了两个答案,不论写在括弧内或括弧外,本小题得-1分. 一个质量为600千克的飞行器在月球表面上: (1)质量是100千克,重力是5880牛顿. (2)质量是100千克,重量是980牛顿. (3)质量是600千克,重量是980牛顿. (4)质量是600千克,重量是5880牛顿. 答( ) 2.一架梯子斜靠在光滑的竖直墙上,下端放在水平的粗糙地面上.下面是 梯子受力情况的简单描述.哪一句是正确的?梯子受到: (1)两个竖直的力,一个水平的力. (2)一个竖直的力,两个水平的力. (3)两个竖直的力,两个水平的力. (4)三个竖直的力,两个水平的力. 答( ) 3.一个平行板电容器,它的电容: (1)跟正对的面积成正比,跟两板间的距离成正比. (2)跟正对的面积成正比,跟两板间的距离成反比. (3)跟正对的面积成反比,跟两板间的距离成正比. (4)跟正对的面积成反比,跟两板间的距离成反比. 答( ) 4.把220伏特的交流电压加在440欧姆的电阻上,在电阻上: (1)电压的有效值为220伏特,电流的有效值为0.5安培. (2)电压的最大值为220伏特,电流的有效值为0.5安培. (3)电压的有效值为220伏特,电流的最大值为0.5安培. (4)电压的最大值为220伏特,电流的最大值为0.5安培. 答( ) 5.某媒质对空气的折射率为1.414,一束光从媒质射向空气,入射角为60°. 它的光路图是: 答( ) 6.物体竖直上抛后又落向地面,设向上的速度为正,它在整个运动过程 中速度v跟时间t的关系是: 答( ) 7.在电场中,A点的电势高于B点的电势, (1)把负电荷从A点移到B点,电场力作负功. (2)把负电荷从A点移到B点,电场力作正功. (3)把正电荷从A点移到B点,电场力作负功. (4)把正电荷从B点移到A点,电场力作正功. 答( ) 8.把质量为m1的0℃的冰放进质量为m2的100℃的水中,最后得到温度是 10℃的水.如果容器吸热和热量损失均可忽略,那么m1和m2的关系是: (1)m1=9m2 (2)m1=17m2 (3)6.1m1=m2 (4)m1=m2 答( ) 9.一定质量的理想气体吸热膨胀,并保持压强不变,则它的内能增加. (1)它吸收的热量等于内能的增量. (2)它吸收的热量小于内能的增量. (3)它吸收的热量大于内能的增量. (4)它吸收的热量可以大于内能的增量,也可以小于内能的增量. 答( ) 10.如右图所示,一水平放置的矩形闭合线圈abcd,在细长磁铁的N极附近 竖直下落,保持bc边在纸外,ad边在纸内,由图中的位置Ⅰ经过位置Ⅱ到 位置Ⅲ,位置Ⅰ和Ⅲ都很靠近Ⅱ.在这个过程中,线圈中感生电流: (1)沿abcd流动. (2)沿dcba流动. (3)由Ⅰ到Ⅱ是沿abcd流动,由Ⅱ到Ⅲ是沿dcba流动. (4)由Ⅰ到Ⅱ是沿dcba流动,由Ⅱ到Ⅲ是沿abcd流动. 答( ) 二、(20分)本题是填空题,分5个小题,把正确答案填写在题中空白处.(本题不 要求写出演算过程) 1.12个相同的电池联接如图,每一个电池的电动势为1.5伏特,内电阻为0.3 欧姆,电池组的总电动势为 ,总内电阻为 . 3.一束光自真空射入玻璃.已知光在真空中的波长为0.60微米,传播速度 为3.0×108米/秒,玻璃的折射率为1.5.则光进入玻璃后的波长为 , 频率为 . 4.一个小物体从光滑半球的顶点滑下,初速度很小,可以忽略不计,球半径 为0.40米,物体落地时速度的大小是 . 5.在标准状态下,一细长而均匀的玻璃管,上端开口,一段长度为38厘米的 水银柱,把一定量的空气封闭在管内.当玻璃管跟竖直方向成60°时,管 内空气柱的长度为60厘米.如果使管竖立,在管内空气达到平衡状态后, 这段空气柱的长度是 . 三、(10分)在薄凸透镜成像中,设u表示物距,v表示像距,f表示透镜的焦 况).再根据上式找出像距v为正负值的条件,并指出它跟像的虚实的关系. 四、(12分)如右图所示,一束具有各种速率的带一个基本正电荷的两种铜离子, 质量数分别为63和65,水平地经小孔S进入有匀强电场和匀强磁场的区域. 电场E的方向向下,磁场B的方向垂直纸面向里.只有那些路径不发生偏折 的离子才能通过另一个小孔S'.为了把从S'射出的两种铜离子分开,再让 它们进入另一方向垂直纸面向外的匀强磁场B'中,使两种离子分别沿不 同半径的圆形轨道运动.试分别求出两种离子的轨道半径.(应明确说明 演算过程的物理上的根据). 已知:E=1.00×105伏特/米,B=0.4特斯拉,B'=0.50特斯拉,基本电荷e=1.60 ×10-19库仑,质量数为63的铜原子的质量m1=63×1.66×10-27千克, 质量数为65的铜原子的质量m2=65×1.66×10-27千克. 五、(14分)右图表示用惠斯登电桥来测量未知电阻Rx.图中R是已知电阻;K是电 键;G是电流计;AC是一条粗细均匀的长直电阻丝;D是滑动触头,按下时就使 电流计的一端与电阻丝接通;L是米尺. (1)简要说明测量Rx的实验步骤(其中要写出计算Rx的公式). (2)如果滑动触头D在从A向C移动的整个过程中,每次按下D时,流过G的电 流总是比前一次增大,已知AC间的电阻丝是导通的,那么,电路可能在什 么地方断路了?说明理由.(分析时可认为电池内电阻和电阻R'均可忽 略). 六、(14分)有两个物体,质量分别为m1和m2·m1原来静止,m2以速度v向右运动, 如 图 所 示 . 它 们 同 时 开 始 受 到 向 右 的 大 小 相 同 的 恒 力 F, 在 m1m2三种情况下,它们能否达到相同的速度(矢量)?试列 出它们速度的表达式,并根据此式分别进行讨论,讨论中要注意说明理 由. 如果它们受到的恒力F的方向都跟v垂直,它们能否达到相同的速度(矢 量)?为什么? 1980年答案 一、1.(3) 2.(3) 3.(2) 4.(1) 5.(4) 6.(2) 7.(1) 8.(4) 9.(3) 10.(1) 评分说明:全题30分,每小题3分. (1)每小题答案正确的给3分,答案错误的给-1分,未答的给0分. (2)每小题选择了两个或两个以上答案的,无论答案写在括号里面或近旁, 均给-1分. (3)十个小题分数的代数和,如果是正数或0,这就是本题的得分;如果小题 分数的代数和是负数,本题得分记作0. 二、(1)6伏特,0.4欧姆. (2)4, 2. (4)2.8米/秒. (5)50厘米. 评分说明:全题20分.每小题4分. (1)前面三个小题各有两个答案,每个答案2分.数值错误的不得分;数值正 确而缺单位或单位错误的,每个单位扣1分. (2)后面两个小题的答案中数值错误的不得分;缺单位或单位错误的,每个 单位扣2分.第4小题的答案中如果由于开方不精确而得到相近数值的, 扣1分. 三、参考解法: (1)绘出凸透镜成像的光路图. 由于AB=O'O,因此: 由于BO=u,B'O=v,OF'=f,B'F'=v-f,因此: 化简后可得: 这就是所要证明的薄凸透镜成像的公式. (3)从上式可得: 由于fu总是正值,如果u>f,即物体位于焦点之外,则v为正值,所成的是实像; 如果ua2,m1的速度增加得比m2的快,虽然m2已有一定初速度, 它们仍可在某一时刻达到相同的速度. m1=m2时,由于a1=a2,它们的速度增加得一样快,m2已有一初速度v,因此m1 的速度将总是比m2的速度小v,它们不可能达到相同的速度. m1>m2时,由于a1m2的讨论中未说到v1比v2 越来越小的,均不扣分. (3)中,未指出F对v方向上的速度没有影响的,扣2分;未指出v1和v2的方向 不可能相同的,扣3分. 考生也可以根据(1)、(2)两式,设v1=v2,求出它们速度相等的时间 否达到相同的速度,只要说理正确,应同样给分.答卷中只要基本点都已 说到,无概念错误,但叙述层次不清、文句不够通顺的,这次考试暂不扣 分,明显的概念错误可酌情扣分. 1981年试题 一、(18分)本题分6个小题.每小题提出了四个答案,其中只有一个是正确的. 选出你认为正确的答案,把它的号码填写在本小题后的方括号内.每小题 选出正确答案的,得3分;选错的,得-1分;不答的,得0分.每小题只许选一 个答案.如果写了两个答案,不论写在括号内或括号旁,本小题得-1分. (1)如图,在光滑的水平桌面上有一物体A,通过绳子与物体B相连.假设绳 子的质量以及绳子与定滑轮之间的摩擦力都可忽略不计,绳子不可伸长. 如果物体B的质量是物体A的质量的3倍,即mB=3mA,那么物体A和B的加速 度的大小等于: 答( ) (2)在光滑的水平桌面上放一物体A,A上再放一物体B,A、B间有摩擦.施加 一水平力F于B,使它相对于桌面向右运动.这时物体A相对于桌面: 答( ) (3)平行板电容器,其两板始终保持和一直流电源的正、负极相连接,当两 板间插入电介质时,电容器的带电量和两板间的电势差的变化是: 1.带电量不变,电势差增大. 2.带电量不变,电势差减小. 3.带电量增大,电势差不变. 4.带电量减小,电势差不变. 答( ) (4)把一个架在绝缘支座上的导体放在负电荷形成的电场中,导体处于静 电平衡时,导体表面上感应电荷的分布如图所示,这时导体: 1.A端的电势比B端的电势高. 2.A端的电势比B端的电势低. 3.A端的电势可能比B端的电势高,也可能比B端的电势低. 4.A端的电势与B端的电势相等. 答( ) (5)一段粗细均匀的镍铬丝,横截面的直径是d,电阻是R.把它拉制成直径是 2.10000R 4.100R 答( ) (6)假设火星和地球都是球体,火星的质量M火和地球的质量M地之比 M火/M地=p,火星的半径R火和地球的半径R地之比R火/R地=q,那么火星表面 处的重力加速度g火和地球表面处的重力加速度g地之比 g火/g地等于: 1.p/q2. 2.pq2. 3.p/q 4.pq 答( ) 二、(24分)本题分6个小题,每小题4分.把正确答案填写在题中空白处(不要求 写出演算过程). (1)1标准大气压= 毫米水银柱= 帕斯卡(即牛顿/米2)(水银的密度 ρ=13.6克/厘米3,重力加速度g=9.81米/秒2.本题答案要求取三位有效 数字). (2)一照相机,用焦距f=0.20米的凸透镜做镜头,用来为一个站立在镜头前 4.2米处的儿童照相时,底片应该离镜头 米,底片上像的高度和儿童 的身高之比是1: . (3)把5欧姆的电阻R1和10欧姆的电阻R2串联起来,然后在这段串联电路的 两端加15伏特的电压,这时R1消耗的电功率是 瓦特,R2消耗的电功 率是 瓦特.把R1和R2改为并联,如果要使R1仍消耗与原来同样大小 的电功率,则应在它们两端加 伏特的电压,这时R2消耗的电功率是 瓦特. (4)右下图是一列沿x轴正方向传播的机械横波在某一时刻的图像.从图上 可看出,这列波的振幅是 米,波长是 米,P处的质点在此时刻的 运动方向 . (5)质量是m的质点,以匀速率v作圆周运动,圆心在坐标系的原点O.在质点 从位置1运动到位置2(如右图所示)的过程中,作用在质点上的合力的功 等于 ;合力冲量的大小是 ,方向与x轴正方向成 (逆时针计 算角度). 三、(10分) (1)用游标卡尺(图1)测一根金属管的内径和外径时,卡尺上的游标位置分 别如图2和图3所示.这根金属管的内径读数是 厘米,外径读数是 厘米,管壁厚是 厘米. (2)用下图所示的天平称质量前,先要进行哪些调节?说明调节哪些部件和 怎样才算调节好了. (3)用万用电表电阻挡判断一只PNP型晶体三极管的基极时,电表指针的偏 转情况如下图所示.哪只管脚是基极? 四、(10分)使一定质量的理想气体的状态按图1中箭头所示的顺序变化,图线BC 是一段以纵轴和横轴为渐近线的双曲线. (1)已知气体在状态A的温度TA=300K,求气体在状态B、C和D的温度各是多 少. (2)将上述状态变化过程在图2中画成用体积V和温度T表示的图线(图中要 标明A、B、C、D四点,并且要画箭头表示变化的方向).说明每段图线各 表示什么过程. 五、(10分)一光电管的阴极用极限波长λ0=5000埃的钠制成.用波长λ=3000 埃的紫外线照射阴极,光电管阳极A和阴极K之间的电势差U=2.1伏特,光电 流的饱和值I=0.56微安. (1)求每秒内由K极发射的电子数. (2)求电子到达A极时的最大动能. (3)如果电势差U不变,而照射光的强度增到原值的三倍,此时电子到达A极 时的最大动能是多大? (普朗克恒量h=6.63×10-34焦耳·秒,电子电量e=1.60×10-19库仑,真空中 的光速c=3.00×108米/秒.) 六、(14分)用均匀导线弯成正方形闭合线框abcd,线框每边长8.0厘米,每边的 电阻值为0.010欧姆.把线框放在磁感应强度为B=0.050特斯拉的匀强磁场 中,并使它绕轴OO'以ω=100弧度/秒的匀角速度旋转,旋转方向如图所示. 已知轴OO'在线 框平面内,并且垂直于B, 当线框平面转至和B平行的 瞬时(如图所示): (1)每个边产生的感生电动势的大小各是多少? (2)线框内感生电流的大小是多少?在图中用箭头标出感生电流的方向. (3)e、f分别为ab和cd的中点,e、f两点间的电势差Uef(即Ue-Uf)是多大? 七、(14分)在光滑水平面的两端对立着两堵竖直的墙A和B,把一根倔强系数是k 的弹簧的左端固定在墙A上,在弹簧右端系一个质量是m的物体1.用外力压 缩弹簧(在弹性限度内)使物体1从平衡位置O向左移动距离s,紧靠着1放一 个质量也是m的物体2,使弹簧、1和2都处于静止状态,然后撤去外力,由于弹 簧的作用,物体开始向右滑动. (1)在什么位置物体2与物体1分离?分离时物体2的速率是多大? (2)物体2离开物体1后继续向右滑动,与墙B发生完全弹性碰撞.B与O之间 的距离x应满足什么条件,才能使2在返回时恰好在O点与1相遇?设弹簧 的质量以及1和2的宽度都可忽略不计. 1981年答案 一、(1)〔3〕(2)〔2〕(3)〔3〕(4)〔4〕(5)〔2〕(6)〔1〕 评分说明:全题18分,每小题3分. (1)每小题答案正确的给3分,答案错误的给-1分,未答的给0分. (2)每小题选择了两个或两个以上答案的,无论答案写在括号内或括号旁, 都给-1分. (3)六个小题分数的代数和,如果是正数或0,这就是本题的得分;如果是负 数,本题得分记作0. 二、(1)760;1.01×105. 评分说明:共4分.第一个答案正确的给2分.错误的给0分.第二个答案,正 确的给2分;数量级和前两位有效数字正确而第三位有效数字 有出入的,给1分;数量级不对的,给0分. (2)0.21;20 评分标准:共4分.两个答案都要求准确到第二位有效数字.每个答案正确 的给2分.错误的给0分. (3)5,10,5,2.5 评分说明:共4分,每个答案正确的,给1分,错误的,给0分. (4)0.03,2,沿y轴正方向(或向上). 评分说明:共4分.前两个答案,正确的各给1分;错误的给0分.第三个答案 正确的给2分;未用文字回答而画一向上箭头表示方向的,同样 给2分;错误的给0分. 评分说明:共4分.第一个答案1分.第二个答案,正确的给2分;错误的给0 分.第三个答案,正确的给1分. 评分说明:共4分.每个答案,正确的给2分;错误的给0分. 三、(1)2.37,3.03,0.33. 评分说明:共3分.每个答案,正确的给1分;错误的给0分. (2)要进行两步调节:1.使天平的底板B水平.调节螺旋S,直到重垂线Q的小 锤尖端跟小锥体Z的尖端对正,这就表示底板水平了.2.使天平平衡.调 节螺旋S',使指针D指在标尺K的中央,这就表示天平平衡了. 评分说明:共4分.第一步调节,占2分;只答出要调节底板水平的,给1分; 既答对调节S又答出调节好了的标志的,再给1分,二者缺一的, 不再给分.第二步调节,也占2分;只答出要调节天平平衡的,给 1分;既答对调节S'又答出调节好了的标志的,再给1分,二者缺 一的,不再给分. (3)管脚3是基极. 评分说明:本小题3分.用文字答出管脚3是基极或在图中管脚3旁注明基 极(或注明b)的,都给3分.不要求说明理由. 四、(1)由P-V图上可知,气体在A、B、C、D各状态下的压强和体积分别为PA=4 大气压,VA=10升;PB=4大气压;PC=2大气压,VC=40升;PD=2大气压,VD=20 升.已知 TA=300K,设气体在C、D各状态下的温度分别为TC、TD,则根据理想气 体状态方程有: 由此可求得: TC=600K;TD=300K. 由于在P-V图上.图线BC是一段以纵轴和横轴为渐近线的双曲线,在这一 状态变化过程中PV=常数,所以PBVB=PCVC由此可求得: 由PV=常数,可断定BC是等温过程,设气体在状态B的温度为TB则: TB=TC=600K. (2)在V-T图上状态变化过程的图线如下: AB是等压过程,BC是等温过程,CD是等压过程. 评分说明:全题10分.(1)3分,(2)7分. (1)中,每个答案,正确的给1分;数值错误的,给0分.数值正确而缺单位或 单位错误的,无论在几个答案中出现,只扣1分.直接由P-V图上断定 VB=20升,并正确求出TB的,同样给分. (2)中,A、B、C、D四个点,每个点正确标明的,各给1分.AB、BC、CD三条图 线画正确并说明是什么过程的,各给1分;图线画对而未说明是什么过程 的,各给0分;在图上未画箭头或三个箭头未画全的,扣1分.图上两个坐 标轴每有一个未注明坐标分度值的,扣1分. 五、(1)每秒内发射的电子数为: (2)每个光子的能量为: =6.63×10-19焦耳 (b) 钠的逸出功为: =3.98×10-19焦耳 (c) 每个电子在电场中被加速而获得的能量为: eU=1.60×10-19×2.1焦耳=3.36×10-19焦耳 (d) 根据能量守恒定律,电子的最大动能为: =(6.63-3.98+3.36)×10-19焦耳 =6.01×10-19焦耳(或=3.76电子伏特). (3)当光强增到3倍时,以上结果不变. 评分说明:全题10分.(1)3分,(2)6分,(3)1分. (1)中,正确算出结果(a)的,给3分.单纯数字计算错误(包括数字正确而数 量级错误)扣1分. (2)中,正确算出光子能量(b)的,给1分.正确算出逸出功(c)的,再给2分. 正确算出电子在电场中获得的能量(d)的,再给1分.根据能量守恒关系 正确算出电子最大动能的,再给2分. 直接列出式(e)并算出正确结果的,同样给6分.如果考生根据爱因 斯坦方程,正确算出电子离开K时的最大动能,即只漏去eU一项,把它当 作本题答案的,给4分.单纯数字计算错误扣1分.数值正确而缺单位或单 位错误的,扣1分. (3)中,答案正确的,给1分, 六、(1)令l表示每边边长,R表示其电阻值,则l=0.08米,R=0.010欧姆,设cd段感 生电动势的大小为ε1 ,ab段感生电动势的大小为ε2 ,则 = ×0.05×0.082×100伏特=0.024伏特. (a) da段和bc段不切割磁力线,所以它们的电动势都是零. (2)线框中的总电动势为: ε=ε1 +ε2 = 0.032伏特. (c) 线框中的感生电流为: 根据楞次定律或右手定则,可判断电流方向沿dcbad. (3)解法一: Ue-Uf是ebcf一段电路两端的电势差.它应等于eb段的路端电压 Ueb,bc段两端的电势差Ubc与cf段的路端电压Ucf的代数和,即: Uef=Ueb+Ubc+Ucf. (e) =0伏特, (f) Ubc=-IR=-0.8×0.010伏特=-0.008伏特 (g) =0.008伏特, (h) 所以:Uef=Ueb+Ubc+Ucf (i) 解法二: 闭合电路dcbad中的总电动势等于总电势降落.在edcf一段电路中,电 动势等于总电动势的一半;电阻等于总电阻的一半,因而电势降落是总电势 降落的一半,于是,在这段电路中,电势升正好等于电势降.由此可见,两端 的电势差等于零. 评分说明:全题14分.(1)5分,(2)4分,(3)5分. (1)中,正确求出ε1和ε2的,各给2分;答出da和bc段电动势都是零的,合给 1分;单纯数字计算错误扣1分;ε1和ε2的答案数值正确而缺单位或单 位错误的,无论出现一次或二次,只扣1分. 因把v=ωr的关系搞错而引起答案错误的,只扣1分. (2)中,正确算出总电动势(c)的,给1分;进一步正确算出电流(d)的,再给2 分;直接求出结果(d)的,同样给3分;电流方向正确的再给1分;数值错误 和单位错误的扣分同(1)中规定. (3)中,(e)、(f)、(g)和(h)各占1分;利用以上四式进一步正确算出结论(i) 的,再给1分;不分步计算,直接正确列出(i)式并算出结果的,同样给5分; 单纯运算错误的,扣1分. 只有Uef等于零的结论,而无任何推算过程、无任何论述或论述错误 的,均不给分.结论正确但论述不够清楚的,酌情给分. 七、(1)到达平衡位置O前,1和2一起作加速运动.到O点后,1开始减速,2开始作 匀速运动.因而2和1将在O点分离. 到达O点前,把1、2和弹簧看作一个系统只有系统内的弹簧的弹性 力作功,所以系统的机械能守恒,令v表示1和2到达O点时的速率,则有: 这就是分离时物体2的速率. (2)分离后,在下一次相遇前,1以O点为平衡位置作简谐振动,振动的周期 为: 从1和2分离时开始计时,即令该时刻t=0,则1通过O点的时刻为: 过O点后,2以匀速率v向右作直线运动.与B相碰时,由于碰撞是完全弹 性 的,碰撞后2的速率不变,运动反向. 令x表示B与O点间的距离,则2返回O点的时刻为: 如2恰好在O点与1相遇,则: t2=t1. (d) 将(b)、(c)两式代入(d),即得x应满足的条件为: 评分说明:全题14分.(1)4分,(2)10分. (1)中,答出在O点分离的,给2分;列出机械能守恒方程并求出2的速率的, 再给2分. (2)中,知道和2分离后1作简谐振动,并写出振动周期公式(a)的,给2分;正 确列出1经过O点的时刻t1,即式(b)的,再给4分.对于t1,只回答了n=1或 n=2一次的,扣3分;只回答了n=1和n=2两次的,扣2分;只回答了n为奇数 或为偶数一种情形的,扣2分.由于这一步考虑不全面,导致本题最后答 案不全的,后面不重复扣分. 答出2与墙B碰撞后,速率不变,运动反向的(不要求证明),给1分, 又正确求出2返回O点的时刻(c)的,再给1分. 正确列出1和2在O点相遇的条件,即(d)的,给1分;进一步求出距离 x,即(e)的,再给1分.将(d)、(e)两步并作一步直接求出结果的,同样给2 分. 本题中的单纯演算错误,可视其对物理过程或最后结果的影响程 度,酌情扣分. 1982 年物理高考试题 一、每小题 3 分.每小题选出一个正确的答案,把它的号码填写在题后的方括号内.选对的,得 3 分;选错的,得-1 分;不答的,得 0 分;如选了两个答案,不论写在括号内或括号旁,得-1 分. (1)一质点作简谐振动,其位移 x 与时间 t 的关系曲线如图所示.由图可知,在 t=4 秒时,质点的: 1.速度为正的最大值,加速度为零. 2.速度为负的最大值,加速度为零. 3.速度为零,加速度为正的最大值. 4.速度为零,加速度为负的最大值. 答( ) (2)如图所示,在垂直于纸面向内的匀强磁场中,从 P 处垂直于磁场方向发射出两个电子 1 和 2, 其速度分别为 v1 和 v2.如果 v2=2v1,则 1 和 2 的轨道半径之比 r1:r2 及周期之比 T1:T2 分别为: 1.r1:r2=1:2 T1:T2=1:2 2.r1:r2=1:2 T1:T2=1:1 3.r1:r2=2:1 T1:T2=1:1 4.r1:r2=1:1 T1:T2=2:1 答( ) (3)有两根相互平行的无限长直导线 a 和 b,相距 15 厘米,通过两导线的电流大小和方向如下 图所示.在两导线所在的平面内、两电流产生的磁场中,磁感应强度为零的点到导线 a 和 b 的距离 分别为: 1.10 厘米和 5 厘米,如 A 点. 2.30 厘米和 15 厘米,如 B 点. 3.12 厘米和 3 厘米,如 C 点. 4.20 厘米和 5 厘米,如 D 点. 答( ) (4)两个质量相同的物体 1 和 2 紧靠在一起放在光滑水平桌面上,如下图所示.如果它们分别 受到水平推力 F1 和 F2,且 F1>F2,则 1 施于 2 的作用力的大小为: 1.F1 2.F2 答( ) (5)设行星 A 和行星 B 是两个均匀球体.A 与 B 的质量之比 mA:mB=2:1;A 与 B 的半径之比 RA:RB=1:2.行星 A 的卫星 a 沿圆轨道运行的周期为 TA,行星 B 的卫星 b 沿圆轨道运行的周期为 TB,两卫星的圆轨道都非常接近各自的行星表面,则它们运行的周期之比为: 1.TA:TB=1:4. 2.TA:TB=1:2. 3.TA:TB=2:1. 4.TA:TB=4:1. 答( ) 二、每小题 5 分 (1)质量为 2000 千克的均匀横梁,架在相距 8 米的东、西墙上.一质量为 3200 千克的天车停 在横梁上距东墙 3 米处.当天车下端未悬吊重物时,东墙承受的压力为 牛顿;当天车吊着一质量为 1600 千克的重物使它以 4.9 米/秒 2 的加速度上升时,东墙承受的压力比原来增大 牛顿. (2)一定质量的理想气体,当体积保持不变时,其压强随温度升高而增大.用分子运动论来解释, 当气体的温度升高时,其分子的热运动加剧,因此:1. ;2. ,从而导致压强增大. (3)在右图所示的电路中,直流电源的电压为 U=18 伏特,电容器 A 和 B 的电容分别为 CA=20 微法和 CB=10 微法.开始时,单刀双掷开关 K 是断开的,A 和 B 都不带电. (1)把 K 扳到位置 1,A 的带电量 QA= 库仑; (2)然后把 K 从位置 1 换接到位置 2,则 B 的带电量 QB= 库仑; (3)再把 K 扳到位置 1,使 A 充电,然后把 K 换接到位置 2,则 B 的 带电量变为 QB= 库仑. (4)两个相同的声源 S1 和 S2(见下图)相距 10 米,频率为 1700 赫兹,振动位相相同,已知空气中的 声速是 340 米/秒,所以由 S1 和 S2 发出的声波在空气 中的的波长是_______米。图中 Q 是 S1S2 的中点,OQ 是的 S1S2 中垂线,长度 OQ =400 米,OP 平行于 S1S2。 长度。长度 OP =16 米。由于两列声波干涉的结果,在 O 点声波振动将______,并且在 O、P 之间的线段上会出现______个振动最弱的位置。 (5)飞机从一地起飞,到另一地降落.如果飞机在竖直方向的分速度vy与时间t的关系曲线如图 所示(作图时规定飞机向上运动时 vy 为正),则在飞行过程中,飞机上升的最大高度是 米;在 t=2200 秒到 t=2400 秒一段时间内,它在竖直方向的分加速度 ay 为 米/秒 2. 三、(1)如右图所示,S 是侧壁上开有一个三角形小孔的光源箱,M 是平面镜,L 是凸透镜.图中 上方所画的是各个器件在光路图中采用的符号.现在要用这些器件和光具座用平面镜补助法测 定凸透镜 L 的焦距. 1.画出反映测量原理的光路图. 2.扼要说明实验步骤. (2)有一个量程为 0~0.6~3 安培的安培表(0~0.6 安培挡的内阻为 0.125 欧姆,0~3 安培挡的 内阻为 0.025 欧姆),一个量程为 0~15 伏特的伏特表(内阻为 1.5×104 欧姆),一个滑动变阻器(电阻 为 0~200 欧姆),一个电压为 12 伏特的蓄电池组,一个电键和一些导线.要使用这些器件用伏安法 测量一个阻值约为几十欧姆的线圈的电阻. 1.画出实验应采用的电路图. 2.在上图中画线把所示的器件联接成电路(只要求完成以上两项). 四、如右图,汽缸 A 和容器 B 由一细管经阀门 K 相联.A 和 B 的壁都是透热的,A 放置在 27℃、,1.00 大气压的空气中,B 浸在 127℃的恒温槽内.开始时,K 是关断的,B 内为真空,容积 VB=2.40 升;A 内装有理想气体,体积为 VA=4.80 升.假设汽缸壁与活塞 D 之间无摩擦,细管的容积 可忽略不计.打开 K,使气体由 A 流入 B.等到活塞 D 停止移动时,A 内气体的体积将是多少? 五、已知氢原子基态的电子轨道半径为 r1=0.0528×10-10 米量子数为 n 的 (1)求电子在基态轨道上运动时的动能. (2)有一群氢原子处于量子数 n=3 的激发态.画一能级图在图上用箭头标明这些氢原子能发 出哪几条光谱线. (3)计算这几条光谱线中波长最短的一条的波长. (静电力恒量 k=9.0×109 牛顿·米 2/库仑 2,电子电量 e=1.60×10-19 库仑,普朗克恒量 n=6.63×10-34 焦耳·秒,真空中光速 c=3.00×108 米/秒.) 六、如图 1,闭合的单匝线圈在匀强磁场中以角速度ω绕中心轴 OO'逆时针匀速转动.已知: 线圈的边长 ab=cd=l1=0.20 米,bc=da=l2=0.10 米,线圈的电阻值 R=0.050 欧姆,角速度ω=300 弧度/ 秒;匀强磁场磁感应强度的大小 B=0.50 特斯拉,方向与转轴 OO'垂直.规定当线圈平面与 B 垂直, 并且 ab 边在纸面(即过 OO'轴平行于 B 的平面)前时开始计算线圈的转角θ. (1)当θ=ωt=30°时,线圈中感生电动势的大小、方向如何?线圈所受电磁力矩 M 磁的大小、方 向如何? (2)这时,作用在线圈上电磁力的即时功率等于多少? (3)要维持线圈作匀角速转动,除电磁力矩 M 磁外,还必须另有外力矩 M 外作用在线圈上.写出 M 外随时间 t 变化的关系式,并以 t 为横坐标、M 外为纵坐标画出 M 外 随 t 变化的图线. 七、在一原子反应堆中, 用石墨(碳)作减速剂使快中子减速.已知碳核的质量是中子的 12 倍. 假设把中子与碳核的每次碰撞都看作是弹性正碰,而且认为碰撞前碳核都是静止的. (1)设碰撞前中子的动能是 E0,问经过一次碰撞中子损失的能量是多少? (2)至少经过多少次碰撞,中子的动能才能小于 10-6E0?(lg13=1.114,lg11=1.041.) 1982 年答案 一、(1)〔4〕(2)〔2〕(3)〔2〕(4)〔3〕(5)〔1〕 评分说明:全题 15 分,每小题 3 分. 1.每小题答案正确的给 3 分,答案错误的给-1 分,未答的给 0 分. 2.每小题选择了两个或两个以上答案的,无论答案写在括号内或括号旁,都给-1 分. 3.五个小题分数的代数和,如果是正数或 0,这就是本题的得分;如果是负数,本题得分记作 0. 二、全题 25 分,每小题 5 分.各题中,在每一空格内填入正确答案的,按该答案后面方括号内的 分数给分,错误的给 0 分. (1)2.94×104,1.47×104 (2)1.分子每次碰壁时给器壁的冲量增大;2.分子在单位时间内对单位面积器壁的碰撞次数增 多.(两问都答对的给 5 分,答对一问的给 3 分.1 中只答出分子平均速率增大或分子动量增大的,这 一问不给分.2 中未答出"单位时间"和"单位面积"的,不扣分.) (3)3.6×10-4,1.2×10-4,1.6×10-4. (4)0.20,加强,2. (5)8000,0.10. 三、1.光路图如右图. 2.实验步骤如下: ①把光源箱、凸透镜和平面镜放在光具座上,凸透镜放在光源箱和平面镜之间.调整三者的高 度,使光源箱的小孔、透镜的光心 O 和平面镜的中心在一条直线上. ②调节透镜和光源箱之间的距离,直到从三角形小孔射出的光线经透镜折射与平面镜反射 后,在光源箱开孔壁上形成小孔的清晰的倒像. ③量出小孔与透镜之间的距离,它就等于透镜的焦距. 评分说明:本小题共 6 分,其中光路图占 3 分,实验步骤占 3 分. 正确画出光路图的,给 3 分;光路图中未标明光线进行方向或未标全的,扣 1 分. 步骤①中答出使三者在一条直线上或共轴的,给 1 分;步骤②中答出在光源箱壁上得到小孔 的清晰的像的,给 1 分;步骤③中答出测量小孔与透镜距离的,给 1 分. 1.应采用的电路图如右图: 2.按照电路图画线把实物连接成的实验电路如右图: 评分说明:本小题共 6 分.其中电路图占 2 分,画线连接电路占 4 分. 1 中,正确画出电路图的给 2 分;把安培表画作并联或把伏特表画作串联的,给 0 分;未画出滑 动变阻器的,只给 1 分.如果在电路图上标出安培表或伏特表的正负端而又标错了的,此处不扣分. 2 中,电路连接正确的,给 4 分,把安培表并联进电路或把伏特表串联进电路的,给 0 分.安培表 量程选择错误的,扣 1 分;安培表和伏特表正负端连接错误的,各扣 1 分;滑动变阻器连接成固定电 阻、接成短路或未接入电路的,扣 1 分. 无论在 1 或 2 中,安培表内接的不扣分;把滑动变阻器接作分压用的不扣分. 四、打开 K 前气体的状态方程为: P0VA=nRTA. (a) 打开 K 后 A 和 B 内的气体状态方程分别为: PAV 抇 A=nARTA,(b) P BvB=nBRTB.(c) 鈅活塞停止移动时 A 和 B 内气体的压强均应等于 P0,即 PA=PB=P0 (d) K 打开前后气体的总摩尔数不变,即: n=nA+nB. (e) 由以上方程可得: =3.00 升. (f) 评分说明:全题 10 分. 正确写出三个状态方程(a)、(b)、(c)的,各给 1 分;写出式(d)的,再给 2 分;写出式(e)或总质量 不变方程的,再给 2 分.解方程并算出最后结果(f)的,给 3 分,其中数值计算和单位各占 1 分. 五、(1)设电子的质量为 m,电子在基态轨道上的速率为 v1,根据牛顿第二定律和库仑定律,有: 由上式可得电子动能为: =2.18×10-18 焦耳 =13.6 电子伏特. (c) (2)当氢原子从量子数 n=3 的能级跃迁到较低能级时,可以得到三条光谱线,如能级图中所示: (3)n=3 的能级值为. 与波长最短的一条光谱线对应的能级差为: E3-E1=12.09 电子伏特. (d) 利用: hv=E3-E1 (e) 可得光的波长为: =1.03×10-7 米. (g) 评分说明:全题 10 分.(1)3 分,(2)3 分,(3)4 分. (1)中,列出式(a)的,给 1 分;进一步得出电子动能表达式(b)的,再给 1 分;算出动能值(c)的,再给 1 分.答案(c)中,对能量单位不作限制,但数值和单位必须一致方能得分. (2)中,正确画出能级图并且标明三个量子数 n=1、2、3 和三条光谱线的,给 3 分.每缺一条谱 线扣 1 分,缺量子数或量子数不全的扣 1 分,总共最多扣 3 分. (3)中,算出能级差(d)的,给 1 分;写出频率表达式(e)和波长表达式(f)的,各给 1 分;算出波长(g) 的,给 1 分.有效数字只要求三位. 六、(1)ab 段和 cd 段的速率 v 为: 线圈内的感生电动势 e 等于 ab 段和 cd 段切割磁力线产生的感生电动势之和,即: e=2Bl1vsinθ=Bl1l2ωsinθ. 当θ=30°时, e=0.50×0.20×0.10×300×0.5 伏特 =1.5 伏特 (b) 方向沿 abcd. 线圈内的电流强度: ab 段所受电磁力 fab 的方向如图,大小为: fab=il1B =30×0.20×0.50 牛顿 =3 牛顿. (d) 电磁力 fab 对于 OC'轴的力矩方向在俯视图中是顺时针的,大小为: cd 段所受电磁力矩 Mcd 的方向和大小与 ab 段相同.因此总电磁力矩方向是顺时针的,大小为: M 磁=Mab+Mcd=fabl2sinθ =3×0.10×0.5 牛顿·米=0.15 牛顿·米. (e) (2)ab 段上电磁力的方向与速度方向间的夹角是 90°+θ.垂直于速度方向的分力不作功;平行 于速度方向的分力为 fabsinθ,方向总是与速度相反,所以电磁力 fab 的即时功率为: Pab=-(fabsinθ)v. 作用在线圈上的电磁力的即时功率为: P=2Pab=-2fabsinθ =-2×3×15×0.5 瓦特=-45 瓦特. (f) (3)当线圈绕 OO'轴匀速转动时,线圈所受力矩的代数和等于零.因此,外力矩 M 外的大小与电 磁力矩 M 磁相等,方向相反,是逆时针的.所以利用(1)中已经得到的各公式及θ=ωt 可得: M 外=fabl2sinθ=il1l2Bsinωt =0.60Sin2300t0 (h) 以 t 为横坐标和 M 外为纵坐标的 M 外-t 图线如右图所示. 评分说明:全题 16 分.(1)9 分,(2)3 分,(3)4 分. (1)中,算出速率(a)的,给 1 分;算出电动势(b)的,再给 2 分,得出电动势方向的,再给 1 分.算出电 流(c)的,给 1 分;算出 ab 段或 cd 段上电磁力大小(d)的,再给 1 分;得出电磁力方向的,再给 1 分;算 出总电磁力矩大小(e)的,再给 1 分;得出总电磁力矩方向的,再给 1 分.在求 e 和 M 磁时,未分步计算, 直接写出 e=2Bl1vsinθ和 M 磁=il1l2Bsinθ的.不扣分. (2)中,算出电磁力即时功率(f)的,给 3 分.符号错误的,扣 1 分.按其他方法计算,例如,按 P=-ie 或 p=-i2R 计算的,同样给分. (3)中,得出M 外表达式(g)或(h)的,给 2 分;作出M 外-t曲线的,再给 2 分.在图上,将2π/ω标作π/150 以及将(Bl1l2)2ω/R 标作 0.6 的,同样给分.曲线上只要 t=0、π/2ω、π/ω、3π/2ω、2π/ω各时刻的 M 外 值正确,形状虽与正弦平方曲线稍有出入,不扣分.所得 M 外表达式与(g)式或(h)式差一负号,并且曲 线全在横轴下方的,不扣分. 七、(1)设中子和碳核的质量分别为 m 和 M,碰撞前中子的速度为 v0,碰撞后中子和碳核的速 度分别为 v 和 V.根据动量守恒定律,可得: mv0=mv+MV (a) 根据弹性碰撞中动能守恒,可得: 由(a)、(b)二式可解出: v0+v=V 将上式代入(a)中,得: 已知 M=12m,代入(c)中可得: 因此在碰撞过程中中子损失的能量为: (2)设 E1,E2……En 分别表示中子在第 1 次、第 2 次……第 n 次碰撞后的动能.由(d)可得: 同理可得: …… …… 已知 En=10-6E0.代入上式可得: 即: 取对数可得: 2n(lg13-lg11)=6. 将 lg13=1.114,lg11=1.041 代入上式,即得: 因此,至少需经过 42 次碰撞,中子的动能才能小于 10-6E0 评分说明:全题 12 分.(1)6 分,(2)6 分. (1)中,列出(a)和(b)的,各给 1 分;从(a)、(b)解出(c)的,再给 2 分.不写出(a)和(b)直按写出(c)的, 只给 1 分.得出(e)的,再给 1 分;求出结果(f)的,再给 1 分. (2)中,列出(h)的,给 4 分;只写出(g)的,给 1 分.将 En=10-6E0 代入(h),并得出(i)的,再给 1 分.由(i) 算出最后结果的,再给 1 分. 1983年试题 一、每小题4分.第(1)到第(4)小题,把答案填写在题中横线上空白处,不要求写出 演算过程.第(5)小题,按题意要求在附图上作图. (1)写出下列物理量的单位或数值: 北京地区的重力加速度是9.801 . 冰的熔点是 K. 第一宇宙速度大约是 千米/秒. 基本电荷e=1.60×10-19 . (2)以60°的仰角抛出一物体(空气阻力不计).它到达最高点时的动量的大 小跟刚抛出时的动量的大小之比等于 .物体到达最高点时的动能 跟刚抛出时的动能之比等于 . (3)使金属钠产生光电效应的光的最长波长是5000埃.因此,金属钠的逸出 功W= 焦耳. 现在用频率在3.90×1014赫兹到7.50×1014赫兹范 围内的光照射钠,那么,使钠产生光电效应的频率范围是从 赫兹到 赫兹. (普朗克恒量h=6.63×10-34焦耳·秒) (4)已知氢原子的基态能级是E1=-13.6电子伏特.第二能级是E2=-3.4电子伏 特.如果氢原了吸收 电子伏特的能量,它即可由基态跃迁到第二能 级.如果氢原子再获得1.89电子伏特的能量,它还可由第二能级跃迁到第 三能级.因此,氢原子的第三能级E3= 电子伏特. (5)物体AB被置于一薄凸透镜的焦点F和光心O之间,并垂直于透镜的主轴. 透镜的大小、焦点的位置、物体AB的长度和位置都如附图所示. 1.在下图上用作图法(以透镜中通过光心的一段虚线代表薄透镜)画出凸 透镜成像光路和像. 2.要想看到物体AB的完整的像,眼睛必须处在某一范围内.试作图确定 下图上的这一范围(用斜线标明.) 二、每小题3分.每小题选出一个正确的答案,把它的号码填写在题后的括号内. 选对的,得3分,选错的,得-1分;不答的,得0分;如果选了两个答案,不论写在括 号内或括号旁,得-1分. (1)在右图所示的电路里,r是电源的内阻,R1和R2是外电路中的电阻.如果用 Pr、P1 和P2 分别表示电阻r、R1 和R2 上所消耗的功率,当R1=R2=r 时,Pr:P1:P2等于: 1.1:1:1. 2.2:1:1. 3.1:4:4. 4.4:1:1. 答( ) (2)一个静止的、质量为M的不稳定原子核,当它放射出质量为m,速度 答( ) (3)在右图所示的电路中,电容器A的电容CA=30微法,电容器B的电容 CB=10微法.在开关K1、K2都是断开的情况下,分别给电容器A、B充电. 充电后,M点的电势比N点高5伏特,O点的电势比P点低5伏特.然后把K1、 K2都接通,接通后M点的电势比N点高. 1.10伏特. 2.5伏特. 3.2.5伏特. 4.0伏特. 答( ) 1.核A的中子数减核C的中子数等于2. 2.核A的质量数减核C的质量数等于5. 3.原子核为A的中性原子中的电子数比原子核为B的中性原子中的电子 数多1. 4.核C的质子数比核A的质子数少1. 答( ) (5)如下图所示,在两根平行长直导线M、N中,通过同方向同强度的电流.导 线框ABCD和两导线在同一平面内.线框沿着与两导线垂直的方向,自右 向左在两导线间匀速移动.在移动过程中,线框中感生电流的方向: 1.沿ABCDA,不变.2.沿ADCBA,不变. 3.由ABCDA变形ADCBA. 4.由ADCBA变成ABCDA. 答( ) 三、每小题3分.本题中每小题给出的4个回答中,有一个或几个是正确的.把它们 全选出来,并把它们的号码填写在题后的方括号内.每小题,全部选对的,得3 分;未选全或有选错的,得0分;不答的,得0分.填写在方括号外的号码,不作为 选出的答案. (1)右图中的曲线是一列简谐横波在某一时刻的图象.根据这个图可以确定: 1.周期. 2.波长. 3.振幅. 4.波速. 答( ) (2)以下光学器件是用凸透镜制成的,或相当于一个凸透镜. 1.放大镜.3.远视眼镜. 2.近视眼镜. 4.幻灯机镜头. 答( ) (3)日光灯电路主要由镇流器、起动器和灯管组成.在日光灯正常工作的情 况下: 1.灯管点燃发光后,起动器中两个触片是分离的. 2.灯管点燃发光后,镇流器起降压限流作用. 3,镇流器起整流作用. 4.镇流器给日光灯的开始点燃提供瞬时高电压. 答( ) (4)一个铜块和一个铁块,质量相等,铜块的温度T1比铁块的温度T2高.当它 们接触在一起时,如果不和外界交换能量,那么: 1.从两者开始接触到热平衡的整个过程中,铜块放出的总热量等于铁块 吸收的总热量. 2.在两者达到热平衡以前的任一段时间内,铜块放出的热量不等于铁块 吸收的热量. 4.达到热平衡时,两者的温度相等. 答( ) (5)一定量的理想气体,处在某一初始状态.现在要使它的温度经过状态变化 后回到初始状态的温度,用下列哪些过程可能实现? 1.先保持压强不变而使它的体积膨胀,接着保持体积不变而减小压强. 2.先保持压强不变而使它的体积减小,接着保持体积不变而减小压强. 3.先保持体积不变而增大压强,接着保持压强不变而使它的体积膨胀. 4.先保持体积不变而减小压强,接着保持压强不变而使它的体积膨胀. 四、(1)把答案填写在题中横线上空白处. 1.用螺旋测微器(千分尺)测小球直径时,可动刻度H的位置如右图所示.这时 读出的数值是 ,单位是 . 2.用伏特表的0~3伏特档测电路中某两点间的电压时,表的指针位置如图1 所示.从伏特表读出的电压是 . 3.用安培表的0~0.6安培档测某一电路中的电流时,表的指针位置如图2所 示.从安培表读出的电流是 . (2)给定一个空玻璃瓶(如图3所示),要利用天平和水来测定这个玻璃瓶刻度 线下的容积,测定中主要应进行. A.用天平称量瓶装着水时瓶和水的总质量. B.用天平称量空瓶的质量. C.算出瓶内水的质量,求出瓶的容积. D.调节天平横梁两端的螺旋,使天平平衡. E.调节天平底板下面的螺旋,使天平的底板成为水平. 把以上各项的英文字母代号按实验的合理顺序填写在下面横线上 空白处. 1. ,2. ,3. 4. 5. . (3)电源的输出功率P跟外电路的电阻R有关.下页图1是研究它们关系的实 验电路.为了便于进行实验和保护蓄电池,给蓄电池串联了一个定值电阻 R0,把它们一起看作电源(图中虚线框内部分).于是电源的内电阻就是蓄 电池的内电阻和定值电阻R0之和,用r表示.电源的电动势用ε表示. 1.在下面空白处列出P跟ε、r、R的关系式(R中包含安培表的内阻;伏特 表中电流不计.) 2.在上图2中,按照图1画出连线,把所示的器件连接成实验电路. 3.上表给出了六组实验数据,根据这些数据,在图3的直角坐标系中画出 P-R关系图线.根据图线得出的电源输出功率的最大值是 ,对应 的外电阻的阻值是 . 五、一个负离子,质量为m,电量大小为q,以速率v垂直于屏S经过小孔O射入存 在着匀强磁场的真空室中(如图).磁感应强度 的方向与离子的运动方向 垂直,并垂直于右图纸面向里. (1)求离子进入磁场后到达屏S上时的位置与O点的距离. (2)如果离子进入磁场后经过时间t到达位置P,证明:直线OP与离子入射方 向之间的夹角θ跟t的关系是 六、把上端A封闭、下端B开口的玻璃管插入水中,放掉部分空气后放手,玻璃管 可以竖直地浮在水中(如下图).设玻璃管的质量m=40克,横截面积S=2厘 米2,水面以上部分的长度b=1厘米,大气压强P0=105帕斯卡.玻璃管壁厚度 不计,管内空气质量不计. (1)求玻璃管内外水面的高度差h. (2)用手拿住玻璃管并缓慢地把它压入水中,当管的A端在水面下超过某一 深度时,放手后玻璃管不浮起.求这个深度. (3)上一小问中,放手后玻璃管的位置是否变化?如何变化?(计算时可认为 管内空气的温度不变) 七、一个光滑的圆锥体固定在水平的桌面上,其轴线沿竖直方向,母线与轴线之 间的夹角θ=30°(如右图).一条长度为l的绳(质量不计),一端的位置固定 在圆锥体的顶点O处,另一端拴着一个质量为m的小物体(物体可看作质点, 绳长小于圆锥体的母线).物体以速率v绕圆锥体的轴线做水平匀速圆周运 动(物体和绳在上图中都没画出). (要求说明每问解法的根据) 1983年答案 一、全题20分,每小题4分.各题中,在每一横线上空白处填入正确答案的,按下列 该答案后面方括号内的分数给分;错误的,给0分. (1)米/秒2(1分).273(1分).7.9(1分).答7.8或8的,同样给分.库仑(1分). (3)3.98×10-19(2分). 6.00×1014,7.50×1014(2分).本小题答两位有效数字 也可.不填数值答案的,本答案给0分.6.00×1014和7.50×1014两个数中 有一个错的,本答案给0分. (4)10.2(2分).-1.51(2分). (5) 1.成像光路图如下图所示(2分). (光线AM、AN、BM、BN不属于要求的成像光路图) 只正确画出A(或B)的成像光路并按比例得出全像的,也给2分.光路图 中,未标明光线进行方向或未画出全像的,扣1分. 2.眼睛位置在上图中斜线标出的范围内(2分). 只画出A'MP和B'NP两条线并用斜线标出范围的,也给2分.A'M和 B'N画成实线的,不扣分.作图不太准确的,不扣分.凡用斜线标出的范围 与参考答案不符的,不给分. 二、(1)〔4〕(2)〔2〕(3)〔3〕(4)〔4〕(5)〔2〕 评分说明:全题15分,每小题3分. 1.每小题,答案正确的,给3分;答案错误的,给-1分;未答的,给0分. 2.每小题,选择了两个或两个以上答案的,无论答案写在括号内或括号旁,都 给-1分. 3.五个小题分数的代数和,如果是正数或0,这就是本题全题的得分;如果是 负数,本题全题得分记作0. 三、(1)〔2,3〕(2)〔1,3,4〕(3)〔1,2,4〕(4)〔1,4〕(5)〔1,4〕 评分说明:全题15分,每小题3分. 每小题,答案全部选对的,给3分;未选全或有选错的,给0分.填写在方括 号外的号码不作为考生的答案. 四、(1)参考答案及评分说明:本小题3分. 13.858,毫米〔1分〕.数值写成13.857或13.859并且单位正确的,也给1 分.数值或单位有错的,给0分. 2.2.0伏特(1分).答作2.2伏特的,也给1分.数值或单位有错的,给0分. 0.48安培(1分).答作0.480安培的,或0.481安培的,或0.482安培的,也给1 分.数值或单位有错的,给0分.参考答案的根据是:题图给的这类电表以 估读半小格为宜. (2)参考答案及评分说明:本小题3分. 1.E. 2.D 3.B 4.A 5.C. 答作E,D,A,B,C的,给2分,其他任何顺序都不得给分. (3)参考答案及评分说明:本小题共8分.1.占2分;2.占3分;3.占3分. 2.连线如下图(3分). 只有R0未接入而其它正确的,给2分.安培表或伏特表或两者的正负端接 错,而其他正确的,给1分.变阻器的接法跟下图不同,但接法正确的,不影 响评分. 3.P-R关系图线如下图(2分). 0.80瓦特,5.0欧姆(1分). 作图点在正确位置附近,曲线基本平滑即可.作图点点错两个及以上的,图线 给0分.只是作图点点错一个的,图线给1分. 最大输出功率值允许在0.79瓦特以上到0.81瓦特之间,对应的外电 阻的阻值允许在4.6欧姆到5.4欧姆之间.超出这个范围的,不给这1分. 图线连成明显折线的,不给本小问这3分. 五、(1)离子的初速度与匀强磁场的方向垂直,在洛仑兹力作用下,做匀速圆周运 动.设圆半径为r,则: 离了回到屏S上的位置A与O点的距离为: (2)当离子到位置P时,圆心角(见图): 评分说明:全题10分.(1)4分,(2)6分. (1)中,列出(a)式的,给3分.直接列出(b)式的,给2分.再利用(c)式得出正确结 果的,再给1分.只列出(c)式的,给0分,单纯运算错误,扣1分. (2)中,得出(e)式的,给4分.又利用(f)式得出正确结果的,再给2分.只写出(d) 式和(f)式, 但未用(b)式的只给3分.有单纯运算错误的,扣2分(因为是证 明题). 六、(1)玻璃管A端浮在水面上方时,管受力平衡.设管中空气压强为P1,则管所受 内外空气压力之差(竖直方向)是 f=(P1-P0)S0 (a) 用ρ表示水的密度, P1=P0+ρgh, (b) 则: f=ρghS. (c) f应与管所受重力平衡: ρghS=mg. (d) (2)管竖直没入水中后,设管A端的深度为H,管内气柱长度为l,则A端所在处 水内压强为: PA=P0+Hρg, (f) 管内气压,由管内水面在水下的深度可知:为: P2=P0+Hρg+lρg. (g) 管所受两者压力之差(竖直方向)为: f'=(P2-PA)S=lρgS. (h) 随着管的下降,管内水面也必下降,即管内水面在水下的深度增大〔若 管内水面的深度不变(或减小),则P2不变(或减小),而因管A端的下降,管内空 气的体积却减小了,这与玻-马定律不符〕.因此,P2增大,l减小,故f'减小.当管 A端到达某一深度H0时,f'与管所受重力相等,超过这一深度后,f'小于重力, 放手后管不浮起.由此,当H=H0时, f'=lρgS=mg, (i) 这时,由玻-马定律: P2lS=P1(b+h)S. (k) 代入数值后, (3)由上一小问解答的分析可知,当管A端的深度超过H0时,f'vb时,物体不再与锥面接触. 或:T=1.03mg. 只受重力和绳子拉力作用(如图2所示).用表示绳与圆锥体轴线之间的夹 角,将力沿水平方向和竖直方向分解,按牛顿定律得: Tcos=mg. (e) 2T2-3mgT-2m2g2=0 解此方程,取合理值,得: T=2mg. 评分说明:全题12分. 本题要求考生说明每问解法的根据,即要求得出(c)式,并将(1)、(2) 两问中的速率与(c)式相比较.这部分内容占6分.不论考生用什么方法 解题,得出(c)式的给4分,再将(1)、(2)两问中的速率与(c)式比较的,再各 给1分. 在(1)中,列(a)、(b)式及求解占3分.(a)、(b)两式中有一个列错的, 扣2分.单纯运算错误,扣1分.答案最后结果写作T=mg的,不扣分. 在(2)中,列(d)、(e)式及求解占3分.(d)、(e)两式中有一个列错的, 扣2分.单纯运算错误,扣1分.若误认为=30°,扣2分. 1984年试题 一、(21分)每小题3分.把答案填写在题中横线上空白处,不要求写出演算过程. (1)我国在1984年4月8日成功地发射了一颗通信卫星.这颗卫星绕地 (2)平衡下列核反应方程式: (3)如右图所示,一正离子以速度v从左向右射入匀强电场和匀强磁场并存 的区域中.电场强度E=4×104牛顿/库仑.磁感应强度B=0.2特斯拉,方向 垂直纸面向里.电场、磁场和速度三者的方向互相垂直.如果该离子在场 中运动时不发生偏转,则电场方向在附图中为从 向 ;离子速度 大小v= 米/秒. (4)频率为ν的光照射到一金属表面上,有电子从金属表面逸出.当所加反 向电压U的大小增大到3伏特时,光电流刚好减小到零.已知这种金属的 极限频率为ν0=6×1014赫兹,因此照射光的频率ν= 赫兹. (5)太阳光谱中含有许多暗线,这些暗线是由于 而形成的. (6)氢原子的基态能量E1=-13.6电子伏特,则氢原子处于量子数n=5的能级 时的能量为 电子伏特. (7)S1和S2是两个相干波源.在图中分别以S1和S2为圆心作出了两组同心圆 弧,分别表示在同一时刻两列波的波峰和波谷.实线表示波峰,虚线表示 波谷.在图中方框内标出了三个点a、b、c.在这三个点中,振动加强的点 是 ,振动减弱的点是 . 二、(16分) 每小题4分.本题中每小题给出的几个说法中,有一个或几个是正 确的.把正确的说法全选出来,并将正确说法的号码填写在题后方括号内. 每小题,全部选对的,得4分;选对但不全的,得少量分;有选错的,得0分; 不答的,得0分.填写在方括号外的号码,不作为选出的答案. (1)火车在长直水平轨道上匀速行驶.门窗紧闭的车厢内有一人向上跳起, 发现仍落回到车上原处.这是因为: 1.人跳起后,厢内空气给他以向前的力,带着他随同火车一起向前运动. 2.人跳起的瞬间,车厢的地板给他一个向前的力,推动他随同火车一起 向前运动. 3.人跳起后,车在继续向前运动,所以人落下后必定偏后一些,只是由于 时间很短,偏后距离太小,不明显而已. 4.人跳起后直到落地,在水平方向上人和车始终具有相同的速度. 答( ) (2)法拉第电磁感应定律可以这样表述:闭合电路中感生电动势的大小: 1.跟穿过这一闭合电路的磁通量成正比. 2.跟穿过这一闭合电路的磁感应强度成正比. 3.跟穿过这一闭合电路的磁通量的变化率成正比. 4.跟穿过这一闭合电路的磁通量的变化量成正比. 答( ) (3)在下图所示的电路中,当可变电阻R的阻值增大时: 1.AB两点间的电压U增大. 2.AB两点间的电压U减小. 3.通过R的电流I增大. 4.通过R的电流I减小. 5.通过R的电流I不变. 答( ) (4)下列几种说法中,哪种说法正确? 1.电场中电力线上每一点的切线方向都跟该点的场强方向一致. 2.沿电力线方向,场强一定越来越小. 3.沿电力线方向,电势一定越来越低. 4.在电场力作用下,正电荷一定从电势高的地方向电势低的地方移动. 5.在电场力作用下,负电荷一定从电势高的地方向电势低的地方移动. 答( ) 三、(14分) (1)测定电流表内电阻的实验中备用的器件有: A.电流表(量程0~100μA), B.标准伏特表(量程0~5V), C.电阻箱(阻值范围0~999Ω), D.电阻箱(阻值范围0~9999Ω), E.电源(电动势2V,有内阻), F.电源(电动势6V,有内阻), G.滑动变阻器(阻值范围0~50Ω,额定电流1.5A),还有若干电键和导 线. 1.如果采用图1所示的电路测定电流表A的内电阻并且要想得到较高的精 确度,那末从以上备用的器件中,可变电阻R1应选用 ,可变电阻R2应 选用 ,电源ε应选用 .(用字母代号填写) 2.如果实验时要进行的步骤有: A.合上K1; B.合上K2; C.观察R1的阻值是否最大,如果不是,将R1的阻值调至最大; D.调节R1的阻值,使电流表指针偏转到满刻度; E.调节R2的阻值,使电流表指针偏转到满刻度的一半; F.记下R2的阻值. 把以上步骤的字母代号按实验的合理顺序填写在下面横线上空白处: ① ;② ;③ ;④ ;⑤ ;⑥ . 3.如果在步骤F中所得R2的阻值为600欧姆,则图1中电流表的内电阻Rg的测 量值为 欧姆. (2)如果要将第(1)小题中的电流表A改装成量程为0~5V的伏特表,则改 装的方法是跟电流表 联一个阻值为 欧姆的电阻. (3)图2所示器件中,一部分是将电流表改装为伏特表所需的,其余是为 了把改装成的伏特表跟标准伏特表进行核对所需的.首先在下面空 白处画出改装和核对都包括在内的电路图(要求对0~5V的所有刻度 都能在实验中进行核对),然后在图2上画出连线,将所示器件按以上 要求连接成实验电路. 四、(26分) 第(1)到第(3)小题,每题7分;第(4)小题5分. (1)根据牛顿运动定律证明:两物体沿一直线运动,相互作用但不受外力时, 它们的总动量保持不变. (2)在右图所示的电路中,一理想变压器的原线圈跟副线圈的匝数比为N1∶ N2=1∶2.电源电压U=220伏特.A是额定电流为I0=1安培的保险丝.R是可 变电阻.为了不使原线圈中的电流超过I0,调节电阻R时,其阻值最低不 能小于多少欧姆? (3)附图为天平的原理示意图,天平横梁的两端和中央各有一刀口,图中分 别用A、B、O三点代表;三点在一条直线上,并且 =L.横梁(包括固 定在横梁上的指针OD)可以中央刀口为轴转动.两边的挂架及盘的质量 相等.横梁的质量为M.当横梁水平时,其重心C在刀口的正下方,C到O的 距离为h,此时指针竖直向下.设只在一盘中加一质量为△m的微小砝码, 最后横梁在某一倾斜位置上达到平衡,此时指针与竖直方向成θ角.已 知L、h、M及△m,求θ. (4)估算地球大气层空气的总重量.(最后结果取1位有效数字) 五、(13分) 一辆车通过一根跨过定滑轮的绳PQ提升井中质量为m的物体,如图 所示.绳的P端拴在车后的挂钩上,Q端拴在物体上.设绳的总长不变,绳的质 量、定滑轮的质量和尺寸、滑轮上的摩擦都忽略不计. 开始时,车在A点,左右两侧绳都已绷紧并且是竖直的,左侧绳长为H. 提升时,车加速向左运动,沿水平方向从A经过B驶向C.设A到B的距离也为H, 车过B点时的速度为vB.求在车由A移到B的过程中,绳Q端的拉力对物体做的 功. 六、(10分) 在两端封闭、内径均匀的直玻璃管内,有一段水银柱将两种理想气体a 和b隔开.将管竖立着,达到平衡时,若温度为T,气柱a和b的长度分别为la和 lb;若温度为T',长度分别为l抋和l抌.然后将管平放在水平桌面上,在平衡 时,两段气柱长度分别为l攁和l攂.已知T、T挕 七、(10分,本题是附加题,成绩不计入总分) 在真空中速度为u=6.4×107米/秒的电子束连续地射入两平行极板之间.极 板长度为l=8.0×10-2米,间距为d=5.0×10-3米.两极板不带电时,电子束将 沿两极板之间的中线通过.在两极板上加一50赫兹的交变电压V=V0sinωt, 如果所加电压的最大值V0超过某一值Vc时,将开始出现以下现象:电子束有 时能通过两极板;有时间断,不能通过. (1)求Vc的大小. (2)求V0为何值才能使通过的时间(△t)通跟间断的时间(△t)断之比为 (△t)通∶(△t)断=2∶1. 1984年答案 一、全题21分,每小题3分.各题中,在每一横线上空白处填入正确答案的,按下 列该答案后面方括号内的分数给分;错误的,给0分. (1)1〔3分〕. (3)上,下〔1分〕;2×105〔2分〕. (4)13.2×1014〔3分〕. (5)太阳光经过温度较低的太阳大气层时某些特征谱线的光被吸收〔3分. 要求答出两个要点:太阳大气层;吸收.答出一个要点的给1分,两个都答 出的给3分〕. (6)-0.544〔3分〕. (7)a、b〔2分〕,c〔1分〕. 二、(1)〔4〕. (2)〔3〕. (3)〔1,4〕. (4)〔1,3〕. 评分说明:全题16分,每小题4分. 每小题,答案全部选对的,给4分;未选全而无选错的,给1分;有选错的, 给0分;未答的,给0分.填写在方括号外的号码不作为考生的答案. 三、(1)本小题7分. 1.D,C,F〔3分.D,C答对的,给2分,有一个答错的,不给这2分.F答对的,给 1分〕. 2.CADBEF〔3分.顺序有错的,不给分〕. 3.600〔1分〕. (2)本小题2分.每小问占1分. 串,49400. (3)本小题共5分.电路图如图1,占2分,如有错误,不给这2分.连线如图2, 占3分,如有错误,不给这3分. 接法只要正确,能满足实验要求,虽与参考答案不同,不影响评分;不 满足实验要求,例如将滑动变阻器当作串联可变电阻使用,不给分. 四、(1)牛顿第二定律F=ma可以写成: 其中p=mv为物体的动量. 当两物体相互作用而不受外力时,令F1和F2分别表示每个物体所受的 力,可得: 根据牛顿第三定律: F1=-F2 (b) 可得: △p1=-△p2. 令p表示两物体的总动量,则得: △p=△p1+△p2=0. (c) 即总动量的改变量等于零,总动量不变. 评分说明:本小题7分. 列出(a)式的,给2分.列出(b)式的,给1分.得出(c)式的,再给4分. 在证明过程中,有正负号错误的,扣2分. 一开始就把第二定律写成动量定理形式的,可不扣分.把相互作用力 设为恒力的,可不扣分. (2)设所求的电阻值为R,此时R两端的电压为U2,电流为I2,则: UI0=U2I2 (a) U2=I2R (b) 由以上各式可得: 评分说明:本小题7分. (a)、(b)、(c)三式列对一个给1分;列对两个给3分;三个都列对给5分.得 出(d)式再给1分.最后答数正确再给1分. (3)有固定转动轴物体的平衡条件是力矩的代数和等于零.设两边挂盘(包 括质量为△m的微小砝码)对横梁的作用力分别为F1和F2则: F1Lcosθ=F2Lcosθ+Mghsinθ. (a) 因为 F1-F2=△mg, 所以得: △mgLcosθ=Mghsinθ. (b) 评分说明:本小题7分. 列出(a)式和(b)式的,给6分;得出(c)式再给1分.直接列出(b)式的, 不扣分.把倾斜后△mg的力臂Lcosθ写作L的也可不扣分(因题设△m很 小因而偏角很小). (4)设地球半径为R,地球表面处的大气压强为p,则大气的总重量: G=4πR2p. 因R=6.4×108米,p=1.0×105帕,代入得: G=5×109牛顿. 评分说明:本小题5分. 列出G=4πR2p给4分,最后答数正确再给1分. 五、设绳的P端到达B处时,左边绳与水平地面所成夹角为θ,物体从井底上升的 高度为h,速度为v,所求的功为W,则: 因绳总长不变,所以: v=vBcosθ.(c) 将(b)、(c)两式代入(a)式,得: 评分说明:全题13分. 列出(a)式的,给3分.列出(b)式的,给3分.列出(c)式的,给5分.列出 (d)式的,给1分.最后结果正确的,再给1分. 六、对于a段气体,有: 对于b段气体,有: 压强关系有:pb-pa=p抇b-p抇a,(e) pa=pb. (f) 由以上各式可得: 评分说明:全题10分. (a)、(b)、(c)、(d)四式全都列对的,给4分;部分列对但无列错的,给 1分;有列错的,不给分. (e)式列对给3分;(f)式列对给1分. 最后结果正确再给2分. 因此,电子通过平行极板时,极板间的电压从场强可看作是恒定不变的. 电子进入平行极板中间后,其运动沿水平方向为匀速运动,沿竖直 方向为匀加速运动.设电子束刚好不能通过平行极板的电压为Vc,电子 经过平行极板的时间为t,所受的电场力为f,则: 由以上三式,可得: 代入数值,得: Vc=91伏特. (2)因为(△t)通=2(△t)断,所以: 由此得: 代入数值得: 评分说明:全题10分.(1)5分,(2)5分. (1)中,列出(a)式和(b)式的,给1分.列出(c)式的,给3分.最后答数正确 的再给1分. 没有对电子通过极板的时间和交流电的周期进行分析比较的, 不扣分. (2)中,列出(d)式的,给4分.最后答数正确再给1分. 1985年试题 一、(21分)每小题3分.把答案填写在题中横线上空白处,不要求写出演算过程. (1)在两条平行长直导线中通以相同方向的电流时,它们之间的作用为互 相 ;通以相反方向的电流时,则互相 . (2)有一群处在量子数n=3的激发态中的氢原子,在它们的发光过程中发出 的光谱线共有 条. (3)图中所示为一双线摆,它是在一水平天花板上用两根等长细绳悬挂一小 球而构成的,绳的质量可以忽略,设图中的l和为已知量.当小球垂直于 纸面作简谐振动时,周期为 . 粒子;如果衰变时产生的新核处于激发态,将会辐射出 . (5)附图为一摄谱仪的示意图.来自光源的光经过狭缝S和透镜L后,成为平 行光射在三棱镜P上.为了在一照相底片MN上拍摄下清晰的光谱,在P与 MN之间须放置一个 .黄、红、绿三种颜色光谱线在照相底片上从M 端到N端的次序为 . (6)设a和b为长度相同的两段均匀纯铜丝,它们的截面积之比为Sa∶Sb=1∶2. 在每一铜丝两端都加以相同电压U,这时两铜丝中自由电子的定向运动 速度之比为va∶vb= ∶ . (7)某地强风的风速约为v=20米/秒.设空气密度为ρ=1.3千克/米3.如果把 通过横截面积为S=20米2的风的动能全部转化为电能,则利用上述已知 量计算电功率的公式应为P= ,大小约为 瓦特(取一位有效数字). 二、(21分)每小题3分.本题中每小题给出的几个说法中,有一个或几个是正确的. 把正确的说法全选出来,并将正确说法前的字母填写在题后方括号内.每小 题,全部选对的,得3分;选对但不全的,得1分;有选错的,得0分;不答的,得0分. 填写在方括号外的字母,不作为选出的答案. (1)一圆盘可绕一通过圆盘中心O且垂直于盘面的竖直轴转动.在圆盘上放 置一木块.当圆盘匀角速转动时,木块随圆盘一起运动.那么, A.木块受到圆盘对它的摩擦力,方向背离圆盘中心. B.木块受到圆盘对它的摩擦力,方向指向圆盘中心. C.因为木块随圆盘一起运动,所以木块受到圆盘对它的摩擦力,方向与木 块的运动方向相同. D.因为摩擦力总是阻碍物体运动,所以木块所受圆盘对它的摩擦力的方 向与木块的运动方向相反. E.因为二者是相对静止的,圆盘与木块之间无摩擦力. 答( ) (2)有一定质量的气体,其温度由T1升高到T2,在这过程中, A.如果气体体积膨胀并因而对外界作功,则分子的平均平动能可能会减 少. B.如果气体体积保持不变,则分子的平均平动能可能不变. C.只有当气体体积被压缩并因而外界对气体作功时,分子的平均平动能 才会增加. D.不管气体的体积如何变化,分子的平均平动能总是增加的. 答( ) (3)ab是一弯管,其中心线是半径为R的一段圆弧.将它置于一给定的匀强磁 场中,磁场方向垂直于圆弧所在平面(即纸面)并且指向纸外:有一束粒 子对准a端射入弯管,粒子有不同的质量、不同的速度,但都是一价正离 子. A.只有速度大小一定的粒子可以沿中心线通过弯管. B.只有质量大小一定的粒子可以沿中心线通过弯管. C.只有动量大小一定的粒子可以沿中心线通过弯管. D.只有能量大小一定的粒子可以沿中心线通过弯管. 答( ) (4)已知一光电管的阴极的极限频率为υ0,现将频率υ大于υ0的光照射在 阴极上. A.照射在阴极上的光的强度愈大,单位时间内产生的光电子数目也愈多. B.加在AK间的正向电压愈大,通过光电管的光电流饱和值也愈大. C.为了阻止光电子到达A,必须在AK间加一足够高的反向电压. D.阴极材料的逸出功等于hυ0. 答( ) A.这定义式只适用于点电荷产生的电场. B.上式中,F是放入电场中的电荷所受的力,q是放入电场中的电荷的电 量. C.上式中,F是放入电场中的电荷所受的力,q是产生电场的电荷的电量. 荷 q2处的场强大小. 答( ) (6)一均匀的扁平条形磁铁与一圆形线圈同在一平面内,磁铁中央与圆心O 重合.为了在磁铁开始运动时在线圈中得到一方向如图所示的感生电流i, 磁铁的运动方式应为: A.N极向纸内,S极向纸外,使磁铁绕O点转动. B.N极向纸外,S极向纸内,使磁铁绕O点转动. C.使磁铁沿垂直于线圈平面的方向向纸外作平动. D.使磁铁沿垂直于线圈平面的方向向纸内作平动. E.使磁铁在线圈平面内绕O点沿顺时针方向转动. F.使磁铁在线圈平面内绕O点沿逆时针方向转动. 答( ) (7)如图,一细绳的上端固定在天花板上靠近墙壁的O点,下端拴一小球.L点 是小球下垂时的平衡位置.Q点代表一固定在墙上的细长钉子,位于OL 直线上.N点在Q点正上方,且QN=QL.M点与Q点等高.现将小球从竖直 位置(保持绳绷直)拉开到与N等高的P点,释放后任其向L摆动.运动过程 中空气阻力可忽略不计.小球到达L后,因细绳被长钉挡住,将开始沿以Q 为中心的圆弧继续运动.在这以后, A.小球向右摆到M点,然后就摆回来. B.小球向右摆到M和N之间圆弧上某点处,然后竖直下落. C.小球沿圆弧摆到N点,然后竖直下落. D.小球将绕Q点旋转,直到细绳完全缠绕在钉上为止. E.关于小球的运动情况,以上说法都不正确. 答( ) 三、(14分) (1)下页图中表示用一零点准确的游标卡尺(主尺上每一小格等于0.1厘米) 测量一圆柱体的直径.从放大了的插图中读出的测量结果为直径D= 厘米. (2)下图为一可供使用的万用表.S为选择开关,Q为欧姆挡调零旋钮.现在要 用它检验两个电阻的阻值(图中未画出电阻),已知阻值分别为R1=60Ω 和R2=470kΩ.下面提出了在测量过程中一系列可能的操作.请你选出能 尽可能准确地测定各阻值和符合于万用表安全使用规则的各项操作,并 且将它们按合理顺序填写在后面的横线上空白处. A.旋动S使其尖端对准欧姆挡×1k(即OHMS×1K). B.旋动S使其尖端对准欧姆挡×100. C.旋动S使其尖端对准欧姆挡×10. D.旋动S使其尖端对准欧姆挡×1. E.旋动S使其尖端对准V1000. F.将两表笔分别接到R1的两端,读出R1的阻值.随后即断开. G.将两表笔分别接到R2的两端,读出R2的阻值.随后即断开. H.两表笔短接,调节Q使表针对准欧姆挡刻度盘上的0,随后即断开. 所选操作及其顺序为(用字母代号填写): , , , , , , , . (操作总数应视实际需要而定) (3)有一改装的安培表A1需要与一标准安培表A2进行核对,采用如图1所示 的电路,其中E为电源,R0为一限流电阻,R为一可变电阻,K为电键.限流电 阻能够限制住电路中的最大电流,使之不超出安培表的量程过多,从而对 安培表起保护作用.实验中已有的器件及其规格如下: 蓄电池E(电动势6V,内阻约为0.3Ω), 改装的安培表A1(量程0—0.6A,内阻约为0.1Ω),标准安培表A2(量程0— 0.6—3A,内阻不超过0.04Ω). 实验中备用的电阻器及其规格如下: A.固定电阻(阻值8Ω,额定电流2A), B.固定电阻(阻值15Ω,额定电流2A), C.滑动变阻器(阻值范围0—20Ω,额定电流2A), D.滑动变阻器(阻值范围0—200Ω,额定电流2A). 已知两个表的刻度盘上都将量程均分为6大格,要求从0.1A起对每条刻 线一一进行核对.为此,从备用的电阻器中,R0 应选用 ,R应选 用 ,(用字母代号填写) 图2所示为电路中所需器件,在图上画出连线,将所示器件按电路图连接 成实验电路. 四、(6分) 在一水平长直轨道上,一动力车牵引一质量为m=5000千克的小车 厢以v0=36千米/小时的速度匀速行驶.这时动力车对该车厢的输出功率为 P=15000瓦特.如果使车厢与动力车脱开,车厢将滑行多长的距离而停止? 五、(7分) 内径均匀的U形管中装入水银.两管中水银面与管口的距离均为 l=10.0厘米.大气压强为p0=75.8厘米汞柱时,将右侧管口密封,然后从左侧管 口处将一活塞缓慢向下推入管中,直到左右两侧水银面高度差达h=6.0厘米 时为止.求活塞在管内移过的距离. 六、(7分)一列横波在x轴线上传播着,在t1=0和t2=0.005秒时的波形曲线如图所 示. (1)由图中读出波的振幅和波长. (2)设周期大于(t2-t1),如果波向右传播,波速多大?如果波向左传播,波速又 是多大? (3)设周期小于(t2-t1)并且波速为6000米/秒,求波的传播方向. 七、(12分) 如图所示,一条长为l的细线,上端固定,下端拴一质量为m的带电小 球.将它置于一匀强电场中,电场强度大小为E,方向是水平的.已知当细线离 开竖直位置的偏角为α时,小球处于平衡. (1)小球带何种电荷?求出小球所带电量. (2)如果使细线的偏角由α增大到φ,然后将小球由静止开始释放,则φ应为 多大,才能使在细线到达竖直位置时小球的速度刚好为零? 八、(12分) 图1中A和B表示在真空中相距为d的两平行金属板.加上电压后, 它们之间的电场可视为匀强电场.图2表示一周期性的交变电压的波形,横坐 标代表时间t,纵坐标代表电压U.从t=0开始,电压为一给定值U0,经过半个周 期,突然变为-U0;再过半个周期,又突然变为U0;……如此周期性地交替变化. 在t=0时,将上述交变电压U加在A、B两板上,使开始时A板电势比B板高, 这时在紧靠B板处有一初速为零的电子(质量为m,电量为e)在电场作用下开 始运动.要想使这电子到达A板时具有最大的动能,则所加交变电压的频率 最大不能超过多少? 1985年答案 一、全题21分,每小题3分.答案正确的,按下列该答案后面方括号内的分数给分; 错误的,给0分. (1)吸引,排斥〔答对一个给1分,答对两个给3分〕. (2)3〔3分〕. (4)α〔1分〕,β〔1分〕,γ光子(或γ射线)〔1分〕. (5)凸透镜〔2分〕,红黄绿〔1分〕. (6)1∶1〔3分〕. 二、(1)B. (2)D. (3)C. (4)ACD. (5)BD. (6)A. (7)E. 评分标准:全题21分,每小题3分. 每小题,答案全部选对的,给3分;未选全而无选错的,给1分;有选错的,给 0分;未答的,给0分. 三、(1)本小题2分. 2.23. 答对的,给2分.答作2.220的同样给分. (2) D H F A H G E. 评分标准:本小题5分,DHF和AHG各占2分,末尾的E占1分.答案为 AHGDHFE的,同样给分.将DHF答为CHF的,扣1分,将AHG 答为BHG的,扣1分.答案中缺一个H(欧姆挡调0)的,扣1分;两 个H都缺的,扣3分. (3)A,D. 评分标准:本小题共7分. 元件选择占3分.只答对一个的,给1分. 电路连线占4分.连法只要正确,虽与参考答案不同,不影响得分. 将两个安培表并联的,给0分.将一个安培表并联在电源两端的,给0分, 将电源短路或未连人电路中的,给0分.未将变阻器连入电路中或连成 不能调的,给0分.安培表正负端连接错误的,扣2分.标准安培表量程选0 —3A的,扣2分. 四、因为车厢原来作匀速运动,所以车厢所受牵引力的大小等于阻力的大小.因 此,车厢所受的阻力f为 脱开动力车后,车厢在阻力作用下滑行的距离若为s,根据功能关系可知 由以上两式得 代入数值得 评分标准:本题6分. 列出(a)式的,给2分.列出(b)式的,给2分.得出(c)式的,再给1分.最 后答数正确的,再给1分. 五、令p1、p2分别代表管中左、右两侧气体的压强,ρ代表水银密度,则有 p1-p2=ρgh. (a) 令x代表活塞移动的距离,由玻意耳定律可知 由以上三式可解出x,得 将p0=75.8厘米汞柱,ρgh=6.0厘米汞柱,l=10.0厘米和h=6.0厘米代入得 x=6.4厘米. 评分标准: 本题7分. 列出(a)式的,给1分.列出(b)式的,给2分.列出(c)式的,给2分.得出(d)式 的,再给1分.最后答数正确的,再给1分. 六、(1)振幅=0.2米.波长=8米. (2)当(t2-t1)小于一个周期时,波的传播距离小于一个波长.如果波向右传播, 则传播距离由图上可看出为2米.由此得 如果波向左传播,由图可知传播距离为6米,由此得 (3)波的传播距离为6000米/秒×0.005秒=30米.比三个波长多出6米.由图 可知波是向左传播的. 评分标准:全题7分.(1)2分,(2)4分,(3)1分. (1)中,振幅和波长各占1分. (2)中,两个传播速度各占2分. (3)中,只有最后答案而无算式或解释说明的,不给分. 七、(1)由小球所受电场力的方向与场强方向相同,可知小球带正电. 小球受三个力作用:重力mg,线拉力T,电场力qE,平衡时 Tcosα=mg, (a) Tsinα=qE. (b) 由以上两式可得电量为 (2)当小球由初始位置1运动到竖直位置2时,线对它的拉力不做功,且已知 原来的动能和后来的动能都等于0,根据能量守恒可知,重力势能的减少 量应等于电势能的增加量.因为 重力势能的减少量=mgl(1-cosφ), (d) 电势能的增加量=qElsinφ, (e) 所以得 mgl(1-cosφ)=qElsinφ. (f) 将上式改写为 与(c)式比较可得 φ=2α. (g) 评分标准: 全题12分.(1)4分,(2)8分. (1)中,正确判断出电荷正负的,给1分.(a)、(b)两式列对一个的,给1分.两个都 列对的,给2分.得出(c)式的,再给1分.没列(a)、(b)式直接写出(c)式的,也 给3分. (2)中,正确列出重力势能的变化(或重力做功)即(d)式的,给2分.正确列出电 势能的变化(或电场力做功)即(e)式的,给2分.列出(f)式的,给3分.得出最 后结果(g)式的,再给1分. 八、开始t=0时,因A板电势比B板高,而电子又紧靠B板处,所以电子将在电场力 作用下向A板运动.在交变电压的头半个周期内,电压不变,电子做匀加速直 线运动,其动能不断增大.如果频率很高,即周期很短,在电子尚未到达A板之 前交变电压已过了半个周期开始反向,则电子将沿原方向开始做匀减速直 线运动.再过半个周期后,其动能减小到零.接着又变为匀加速运动,半个周期 后,又做匀减速运动,……最后到达A板. 在匀减速运动过程中,电子动能要减少.因此,要想电子到达A板时具有 最大的动能,在电压的大小给定了的条件下,必须使电子从B到A的过程中始 终做加速运动.这就是说,要使交变电压的半周期不小于电子从B板处一直 加速运动到A板处所需的时间.即频率不能大于某一值. 其中e和m分别为电子的电量大小和质量.令t表示电子从B一直加速 令T表示交变电压的周期,ν表示频率,根据以上的分析,它们应满足以下的 要求: 由(a)、(b)、(d)三式可解得 评分标准: 本题12分. 列出(a)式的,给2分.列出(b)式的,给2分.这一部分共占4分. 能按题意得出条件(c)式或(d)式的,给5分.得出最后结果(e)式的,再给3 分.这一部分共占8分. 1986年试题 一、(27分)每小题3分.把答案填写在题中横线上空白处,不要求写出演算过程. 其中第(5)、(6)小题按题中要求做答. (1)平衡下列核反应方程: 在核反应堆中,石墨起 的作用;镉棒起 的作用. (2)肥皂泡在阳光照射下呈彩色,这是属于光的 现象. 低压汞蒸气发光产生的光谱,是属于连续光谱、明线光谱、吸收光谱中 的哪一种?答: . 无线电波、可见光、伦琴射线、γ射线中的哪一种是原子内层电子受到 激发后原子辐射的电磁波?答: . (3)汽车沿半径为R的圆跑道行驶,设跑道的路面是水平的,路面作用于 最大不能超过 (4)某金属用频率为ν1的光照射时产生的光电子的最大初动能,是用频率 为ν2的光照射时产生的光电子的最大初动能的2倍,则这种金属的逸出 功W= . (5)附图是电台发出的无线电信号的接收电路图(P为耳机),图中少画了一 个元件,请用惯用的符号把这个元件补画在电路图中.图中电容器C1起 着 的作用.电容器C2起着 的作用. (6)一列振幅是2.0厘米,频率是4.0赫兹的简谐横波,以32厘米/秒的速度 沿上图中x轴的正方向传播.在某时刻,x坐标为-7.0厘米处的介质质点 正好经平衡位置且向y轴正方向运动.试在图中画出此时刻的波形图(要 求至少画出两个波长). (7)在图示的电路中,灯泡A和B都是正常发光的.忽然灯泡B比原来变暗了 些,而灯泡A比原来变亮了些.试判断电路中什么地方出现了断路的故障 (设只有一处出了故障). 答: . (8)长为l的导体棒原来不带电,现将一带电量为q的点电荷放在距棒左端R 处,如图所示.当达到静电平衡后,棒上感应的电荷在棒内中点处产生的 场强的大小等于 . (9)如下图所示,平行板电容器的极板沿水平方向放置,电子束从电容器左 边正中间a处沿水平方向入射,电子的初速都是v0,在电场力的作用下, 刚好从图中所示的c点射出,射出时的速度为v.现若保持电场不变,再加 一个匀强磁场,磁场的方向跟电场和电子入射的方向都垂直(图中垂直 于纸面向里),使电子刚好由图中d点射出,c、d两点的位置相对于中线ab 是对称的,则从d点射出时每个电子的动能等于 . 二、(28分)每小题4分.本题中每小题给出的几个说法中,有一个或几个是正确 的.把正确的说法全选出来,并将正确说法前的字母填写在题后方括号内. 每小题,全部选对的,得4分;选对但不全的,得2分;有选错的,得0分;不答的, 得0分.填写在方括号外的字母,不作为选出的答案. (1)卢瑟福提出原子的核式结构学说的根据是,在用α粒子轰击金箔的实 验中发现α粒子. A.全部穿过或发生很小的偏转. B.绝大多数穿过,只有少数发生很大偏转,甚至极少数被弹回. C.绝大多数发生很大的偏转,甚至被弹回,只有少数穿过. D.全都发生很大的偏转. 答( ) (2)两个分子甲和乙相距较远(此时它们之间的分子力可忽略),设甲固定 不动,乙逐渐向甲靠近直到不能再靠近的整个过程中, A.分子力总是对乙做正功. B.乙总是克服分子力做功. C.先是乙克服分子力做功,然后分子力对乙做正功. D.先是分子力对乙做正功,然后乙克服分子力做功. 答( ) (3)如图所示,一个箱子放在水平地面上,箱内有一固定的竖直杆,在杆上 套着一个环.箱和杆的质量为M,环的质量为m.已知环沿着杆加速下滑, 环与杆的摩擦力的大小为f,则此时箱对地面的压力 A.等于Mg. B.等于(M+m)g. C.等于Mg+f. D.等于(M+m)g-f. E.无法确定. 答( ) (4)在有空气阻力的情况下,以初速v1竖直上抛一物体,经过时间t1到达最 高点.又经过时间t2,物体由最高点落回到抛出点,这时物体的速度为v2. 则 A.v2=v1,t2=t1. B.v2>v1,t2>t1. C.v2v1,t2t1. 答( ) (5)指出下页左图所示的哪些情况中,a、b两点的电势相等,a、b两点的电 场强度矢量也相等. A.平行板电容器带电时,极板间除边缘附近外的任意两点a、b. B.静电场中达到静电平衡时的导体内部任意两点a、b. C.离点电荷等距的任意两点a、b. D.两个等量异号的点电荷,在其连线的中垂线上,与连线中点O等距的两 点a、b. 答( ) (6)下页右图是氢原子中电子绕核做快速的圆周运动(设为逆时针)的示意 图.电子绕核运动,可等效为环形电流.设此环形电流在通过圆心并垂直 于圆面的轴线上某一点P处产生的磁感应强度的大小为B1.现在沿垂直 于圆轨道平面的方向加一磁感应强度为B0的外磁场,这时设电子的轨道 半径没变,而它的速度发生了变化.若用B2表示此时环形电流在P点产生 的磁感应强度的大小.则当B0的方向 A.垂直于纸面向里时,B2>B1. B.垂直于纸面向里时,B2B1. D.垂直于纸面向外时,B2BC.现有平行光线垂直入射到 棱镜的AB面上(如图示),若每个面上的反射都不能忽略,求出射光线.要 求 (1)画出所有典型光线从入射到射出的光路图.(为了图面简洁,表示光线 进行方向的箭头只在棱镜外面的光线上标出即可) (2)简要说明所画光路的根据,并说明每条典型光线只可能从棱镜表面的 哪部分射出. 七、(10分,本题是附加题,成绩不计入总分)用大炮轰击和炮在同一水平面上的 目标.当炮筒仰角为ψ1时,着弹点比目标偏近了一段距离d1;当仰角为ψ2 时,着弹点又比目标偏远了一段距离d2.由这些已知量,求出要想正好击 中目标所需的仰角ψ0.设炮弹出口速率是一定的,空气阻力不计. 1987年答案 一、全题24分,每小题3分.答案正确的,按下列该答案后面方括号内的分数给分; 错误的,给0分.第(5)小题按该题标准给分. (3)油膜的前表面和后表面反射(2分).衍射(1分). (4)μmg(3分). (5)如右图(3分).画对一个波形的,只给1分. (7)6600(3分). (8)如图(3分). 二、(1)A,D. (2)A. (3)D. (4)B,C. (5)B. (6)D. (7)A. (8)B,D. (9)C. (10)A,B,C. 评分标准:全题40分,每小题4分. (1)至(9)小题:每小题,答案全部选对的,给4分;未选全而无选错 的,给2分;有选错的,给0分;未答的,给0分. (10)小题:选对一个答案的给1分,选对两个答案的给2分,选对三 个答案的,给4分;有选错的或未答的给0分. 三、全题12分,每小题按下列答案后面方括号内的分数给分;答错的,给0分. (1)平衡摩擦力(4分). (2)换用欧姆挡的量程时,要重新调整零旋钮(2分).万用表使用后不能把 选择开关置于欧姆挡(2分). (3)查明电流表指针的偏转方向与线圈B中电流方向的关系(4分). 四、可能.条件求如下: 用R1和R2分别表示A和B的电阻(设电阻不随温度改变),由题意可知 当A接到电源上时,因为它正好符合额定功率P1的要求,所以有 当换上B后,B上实际消耗的功率为 根据题意,要求 P2<2.0瓦. (4) 由(1)(2)(3)(4)式,可解得(取合理值): 评分标准:本题8分.列出(1)式给1分,列出(2)式给1分,列出(3)式给2分, 列出(4)式给2分.得出(5)式和(6)式的再各给1分.(如果考生最 后给出的是开方后的四舍五入的数值,同样给分) 五、当小车固定不动时:设平台高为h,小球弹出时的速度的大小为v,则由平抛 运动可知 当小车不固定时:设小球弹出时相对于地面的速度的大小为v′,车 速的大小为V,由动量守恒可知 因为两次的总动能是相同的,所以有 设小球相对于小车的速度的大小为v″,则 v″=v′+V. (4) 由(1)(2)(3)(4)(5)式可解得 评分标准:本题8分. 列出(1)式给1分,列出(2)式给1分,列出(3)式给2分,列出(4)式 给2分,列出(5)式给1分,得出(6)式给1分. 六、参考解答: (1)有三条典型光线①,②,③,光路如下页图所示. (2)因为媒质的折射率n=2.4,所以媒质的临界角 一条典型光线是①,垂直入射到AB面上BE之间(CE⊥AB),部分垂直反 射;部分垂直透射.到BC面,因入射角60°大于θ,发生全反射.到DC面,入射 角30°仍大于θ,又发生全反射.到AB面,垂直入射,部分垂直射出媒质;部 分垂直反射回去,根据光路可逆性,最后由原入射处射出媒质,其反射部分 又重复原路.总之,光线①只能由AB面上FB(BF=BC)间垂直射出. 一条典型光线是②,垂直入射到AB面上EF之间,部分垂直反射;部分垂 直透射.到DC面,入射角30°大于θ,发生全反射.到BC面,入射角60°大于 θ,全反射.到AB面,垂直入射,部分垂直射出媒质;部分垂直反射回去,按光 路的可逆性,由原入射处射出媒质,其反射部分又重复原路.总之,光线②只 能由AB面上FB间垂直射出. 一条典型光线是③,垂直入射到AB面上FA之间,部分垂直反射;部分垂 直透射.到DC面,入射角30°大于θ,全反射.到AB面,入射角 60°大于θ,全反射.到BC面,垂直入射,部分垂直射出媒质;部分垂直反射 回去,按光路的可逆性,由原入射处射出媒质,其反射部分又重复原路.总之, 光线③只能由BC面和AB面上FA间垂直射出. 评分标准:本题8分.(1)占3分,(2)占5分. (1)中,画光路:①占1分,②占1分,③占1分. 每条光线的画法只要有错误或没画,就不能给该条光线的分. (2)中,解释光路:正确说明各次全反射,给1分;正确利用光路的 可逆性,给1分.三条光线的射出范围,答对一条给1分. 本题不要求画出并讨论经E和F点入射的光线的出射线. 七、令x、y分别表示炮弹的水平位移和竖直位移,v0表示其初速度,则有 x=v0cosψ·t, 当炮弹落地时,y=0,由上两式可得水平射程 当仰角为ψ1、ψ2、ψ0时,分别有 已知 x0-x1=d1, x2-x0=d2 (3) 由(2)(3)两式得 评分标准:本题10分.列出(1)式给4分,列出(2)式给2分,列出(3)式给2分,得出 (4)式再给2分. 1989年试题 第Ⅰ卷(选择题) 一、本题中每小题给出的几个说法中,只有一个是正确的. (1)设地球表面的重力加速度为g0,物体在距地心4R(R是地球半径)处,由 于地球的作用而产生的加速度为g,则g/g0为: (B卷2题) A.1. B.1/9. C.1/4. D.1/16. (2)若单摆的摆长不变,摆球的质量增加为原来的4倍,摆球经过平衡位置 时的速度减小为原来的1/2,则单摆振动的 (B卷3题) A.频率不变,振幅不变. B.频率不变,振幅改变. C.频率改变,振幅改变. D.频率改变,振幅不变. (B卷1题) (4)两辆汽车在同一平直路面上行驶,它们的质量之比m1∶m2=1∶2,速度之 比v1∶v2=2∶1.当两车急刹车后,甲车滑行的最大距离为s1,乙车滑行的 最大距离为s2.设两车与路面间的滑动摩擦系数相等,不计空气阻力,则 (B卷5题) A.s1∶s2=1∶2. B.s1∶s2=1∶1. C.s1∶s2=2∶1. D.s1∶s2=4∶1. (5)一轻弹簧上端固定,下端挂一重物,平衡时弹簧伸长了4厘米.再将重物 向下拉1厘米,然后放手,则在刚释放的瞬间重物的加速度是(g取10米/ 秒2) (B卷6题) A.2.5米/秒2.B.7.5米/秒2. C.10米/秒2. D.12.5米/秒2. (6)一质量为m的小球,用长为l的轻绳悬挂于O点.小球在水平拉力F作用下, 从平衡位置P点很缓慢地移动到Q点(如图所示),则力F所做的功为 A.mglcosθ. B.mgl(1-cosθ). C.Flsinθ. D.Flθ. (B卷7题) (7)在图示的电路中,已知电容C=2微法,电源电动势ε=12伏特,内电阻不 计,R1∶R2∶R3∶R4=1∶2∶6∶3.则电容器极板a所带的电量为 A.-8×10-6库. B.4×10-6库. C.-4×10-6库. D.8×10-6库. (B卷8题) (8)在下页图所示的电路中,当滑线变阻器的滑动触点向b端移动时, A.伏特表V的读数增大,安培表A的读数减小. B.伏特表V和安培表A的读数都增大. C.伏特表V和安培表A的读数都减小. D.伏特表V的读数减小,安培表A的读数增大. (B卷9题) (9)一矩形通电线框abcd,可绕其中心轴OO′转动,它处在与OO′垂直的匀 强磁场中(如图).在磁场作用下线框开始转动,最后静止在平衡位置.则 平衡后 A.线框四边都不受磁场的作用力. B.线框四边受到指向线框外部的磁场作用力,但合力为零. C.线框四边受到指向线框内部的磁场作用力,但合力为零. D.线框的一对边受到指向线框外部的磁场作用力,另一对边受到指向线 框内部的磁场作用力,但合力为零. (B卷4题) (10)一架飞机水平地匀速飞行.从飞机上每隔1秒钟释放一个铁球,先后共 释放4个.若不计空气阻力,则四个球 (B卷11题) A.在空中任何时刻总是排成抛物线;它们的落地点是等间距的. B.在空中任何时刻总是排成抛物线;它们的落地点是不等间距的. C.在空中任何时刻总在飞机正下方排成坚直的直线;它们的落地点是等 间距的. D.在空中任何时刻总在飞机正下方排成竖直的直线;它们的落地点是不 等间距的. (11)p-T图上的图线abc表示一定质量的理想气体的状态变化过程,此过程 在p-V图上的图线应为 (B卷12题) (12)在下列四个日光灯的接线图中(S为起辉器,L为镇流器),正确的是 (B卷10题) 二、(10分)每小题2分.本题中每小题给出的几个说法中,有一个或几个是正确 的.把正确的说法全选出来. (13)在光滑水平面上有三个完全相同的小球排成一条直线.2、3小球静止, 并靠在一起,1球以速度v0射向它们(如图).设碰撞中不损失机械能, 则碰后三个小球的速度可能值是 (B卷14题) (14)红色、绿色和黄色的三束平行光分别沿主轴射向同一个玻璃凸透镜,通 过透镜后会聚到主轴上,会聚点到光心的距离分别是f红、f绿、f黄,则 (B卷15题) A.f红=f绿=f黄. C.f绿f绿>f黄. (15)在光滑水平地面上有一木板,一木棒可沿水平轴O转动,其下端B搁在 木板上,而整个系统处于静止状态(如图).现在用水平力F向左推木板, 但木板仍未动.由此可以得出结论:施力F后,木板和木棒之间的正压 力 (B卷13题) A.变大. B.不变. C.变小. D.条件不足,不能判断如何改变. (16)玻尔在他提出的原子模型中所做的假设有: (B卷17题) A.原子处于称为定态的能量状态时,虽然电子做加速运动,但并不向 外辐射能量. B.原子的不同能量状态与电子沿不同的圆轨道绕核运动相对应,而电 子的可能轨道的分布是不连续的. C.电子从一个轨道跃迁到另一轨道时,辐射(或吸收)一定频率的光 子. D.电子跃迁时辐射的光子的频率等于电子绕核做圆周运动的频率. (17)一个点电荷,从静电场中的a点移到b点,其电势能的变化为零,则 (B卷18题) A.a、b两点的场强一定相等. B.该点电荷一定沿等势面移动. C.作用于该点电荷的电场力与其移动方向总是垂直的. D.a、b两点的电势一定相等. (18)一平行板电容器C,极板是水平放置的,它和三个可变电阻及电源联接 成如图所示的电路.今有一质量为m的带电油滴悬浮在两极板之间静 止不动.要使油滴上升,可采用的办法是 (B卷16题) A.增大R1. B.增大R2. C.增大R3. D.减小R2. (19)一平行板电容器充电后与电源断开,负极板接地.在两极板间有一正 电荷(电量很小)固定在P点,如下图所示.以E表示两极板间的场强,U 表示电容器的电压,W表示正电荷在P点的电势能.若保持负极板不动, 将正极板移到图中虚线所示的位置,则 A.U变小,E不变. B.E变大,W变大. C.U变小,W不变. D.U不变,W不变. (B卷20题) (20)对于一定质量的理想气体,在下列各种过程中,可能发生的过程是 (B卷19题) A.气体膨胀对外做功,温度升高. B.气体吸热,温度降低. C.气体放热,压强增大. D.气体放热,温度不变. 第Ⅱ卷(非选择试题共60分) 三、(24分)每小题3分,把答案填写在题中横线上空白处,不要求写出演算过 程. (21)中子的质量为1.0087u,质子的质量为1.0073u,氘核的质量为2.0136u. 中子和质子结合成氘核时释放的能量为 焦耳. 计算结果取两位有效数字. 1u=1.7×10-27千克. (22)图中A、B是体积相同的气缸,B内有一导热的、可在气缸内无摩擦滑动 的、体积不计的活塞C,D为不导热的阀门.起初,阀门关闭,A内装有压 强p1=2.0×105帕,温度T1=300K的氮气.B内装有压强p2=1.0×105帕, 温度T2=600K的氧气.阀门打开后,活塞C向右移动,最后达到平衡.以V1 和V2分别表示平衡后氮气和氧气的体积,则V1∶V2= . (假定氧气和氮气均为理想气体,并与外界无热交换,连接气缸的 管道体积可忽略) (23)下左图中实线是一列简谐波在某一时刻的波形图线.虚线是0.2秒后 它的波形图线.这列波可能的传播速度是 . (24)某一用直流电动机提升重物的装置,如上右图所示.重物的质量m=50 千克,电源的电动势ε=110伏特,不计电源内阻及各处的摩擦.当电动 机以v=0.90米/秒的恒定速度向上提升重物时,电路中的电流强度I=5 安培,由此可知电动机线圈的电阻R= 欧姆. (25)在测定匀变速直线运动的加速度的实验中,用打点计时器记录纸带运 动的时间.计时器所用电源的频率为50赫.图为做匀变速直线运动的 小车带动的纸带上记录的一些点,在每相邻的两点中间都有四个点未 画出.按时间顺序取0、1、2、3、4、5六个点,用米尺量出1、2、3、4、 5点到0点的距离分别是(单位:厘米),由此可得小车的加速度的大小 为 米/秒2,方向 . 8.78 16.08 21.87 26.16 28.94 (26)电阻为R的矩形导线框abcd,边长ab=L,ad=h,质量为m,自某一高度自 由落下,通过一匀强磁场,磁场方向垂直纸面向里,磁场区域的宽度为 h(如下页图所示).若线框恰好以恒定速度通过磁场,线框中产生的焦 耳热是 .(不考虑空气阻力) (27)质量为m的运动员站在质量为m/2的均匀长板AB的中点,板位于水平地 面上,可绕通过B点的水平轴转动,板的A端系有轻绳,轻绳的另一端绕 过两个定滑轮后,握在运动员手中.当运动员用力拉绳时,滑轮两侧的 绳都保持在竖直方向,如上右图所示.要使板的A端离开地面,运动员 作用于绳的最小拉力是 . (28)一个房间的地面面积是15米2,高3分.试估算该房间内空气的质量.已 知空气的平均摩尔质量是2.9×10-2千克/摩尔. 答: 千克. 四、(10分)在测定金属的电阻率的实验中,金属导线长约0.8米,直径小于1毫米, 电阻在5欧左右.实验步骤如下: (29)用米尺测量金属导线的长度,测三次,求出平均值L.在金属导线三个 不同的位置上用 测量直径,求出平均值d. (30)用伏安法测量金属导线的电阻R.试把下图中所给的器材连接成测量R 的合适的线路.图中安培表的量程为0.6安,内阻接近1欧;伏特表的量 程为3伏,内阻为几千欧;电源的电动势为6伏;变阻器的阻值为0~20 欧.在闭合电键前,变阻器的滑动触点应处于正确位置. (31)用上面测得的金属导线长度L、直径d和电阻R,可根据电阻率的表达式 ρ= 算出所测金属的电阻率. 五、(26分)本题共有三个计算题,要求写出主要的文字说明、方程式和演算步 骤.只写出最后答案,而未写出主要演算过程的,不能得分.有数值计算的 题,答案中必须明确写出数值和单位. (32)(9分)如图所示,AB、CD是两根足够长的固定平行金属导轨,两导轨间 的距离为L,导轨平面与水平面的夹角是θ.在整个导轨平面内都有垂 直于导轨平面斜向上方的匀强磁场,磁感应强度为B.在导轨的AC端连 接一个阻值为R的电阻.一根垂直于导轨放置的金属棒ab,质量为m,从 静止开始沿导轨下滑,求ab棒的最大速度. 要求画出ab棒的受力图.已知ab与导轨间的滑动摩擦系数μ,导 轨和金属棒的电阻都不计. (33)(8分)把一个点光源放在焦距为f的凸透镜的焦点上,在透镜的另一侧 2倍焦距处放一个垂直于主轴的光屏,在光屏上看到一个半径为R的光 亮的圆.现保持透镜和光屏不动,而在主轴上移动点光源,若要使光屏 上亮圆的半径缩为R/2,则这个点光源应移到什么位置上? (34)(9分)一个质量为m、带有电荷-q的小物体,可在水平轨道Ox上运动,O 端有一与轨道垂直的固定墙.轨道处于匀强电场中,场强大小为E,方 向沿Ox轴正向,如图所示.小物体以初速v0从x0点沿Ox轨道运动,运动 时受到大小不变的摩擦力f作用,且f 31、质点在磁场中作半径为R的圆周运动, 根据题意,质点在磁场区域中的轨道是半径等于R的圆上的1/4圆周, 这段圆弧应与入射方向的速度、出射方向的速度相切.过a点作平行于x轴 的直线,过b点作平行于y轴的直线,则与这两直线均相距R的O′点就是圆 周的圆心.质点在磁场区域中的轨道就是以O′为圆心、R为半径的圆(图 中虚线圆)上的圆弧MN,M点和N点应在所求圆形磁场区域的边界上. 在通过M、N两点的不同的圆周中,最小的一个是以MN连线为直径 的圆周.所以本题所求的圆形磁场区域的最小半径为 所求磁场区域如图中实线圆所示. 1995年全国普通高等学校招生统一考试 物理试卷 第Ⅰ卷 一、本题共11小题,每小题3分,共33分.在每小题给出的四个选项中只有一项是 正确的. 1.两个物体A和B,质量分别为M和m,用跨过定滑轮的轻绳相连,A静止于水 平地面上,如图1所示.不计摩擦,A对绳的作用力的大小与地面对A的作用 力的大小分别为( ) A.mg,(M-m)g; B.mg,Mg; C.(M-m)g,Mg; D.(M+m)g,(M-m)g. 2.在观察光的衍射现象的实验中,通过紧靠眼睛的卡尺测脚形成的狭缝,观 看远处的日光灯管或线状白炽灯丝(灯管或灯丝都要平行于狭缝),可以 看到 ( ) A.黑白相间的直条纹; B.黑白相间的弧形条纹; C.彩色的直条纹; D.彩色的弧形条纹. 3.已知铜的密度为8.9×103千克/米3,原子量为64.通过估算可知铜中每个 铜原子所占的体积为 ( ) A.7×10-6米3;B.1×10-29米3; C.1×10-26米3; D.8×10-24米3; 期是 ( ). A.2小时; B.1.5小时; C.1.17小时; D.0.75小时. 5.在LC振荡电路中,用以下的哪种办法可以使振荡频率增大一倍? ( ) A.自感L和电容C都增大一倍; B.自感L增大一倍,电容C减小一半; C.自感L减小一半,电容C增大一倍; D.自感L和电容C都减小一半. 6.在演示光电效应的实验中,原来不带电的一块锌板与灵敏验电器相连.用 弧光灯照射锌板时,验电器的指针就张开一个角度,如图2所示.这时 ( ) A.锌板带正电,指针带负电; B.锌板带正电,指针带正电; C.锌板带负电,指针带正电; D.锌板带负电,指针带负电. 7.两个定值电阻R1、R2串联后接在输出电压U稳定于12伏的直流电源上. 有人把一个内阻不是远大于R1、R2的电压表接在R1两端(如图3),电压表 的示数为8伏.如果把此电压表改接在R2的两端,则电压表的示数将 ( ) A.小于4伏; B.等于4伏; C.大于4伏;小于8伏; D.等于或大于8伏. 8.两颗人造卫星A、B绕地球作圆周运动,周期之比为TA:TB=1:8,则轨道半径 之比和运动速率之比分别为 A.RA:RB=4:1,vA:vB=1:2; B.RA:RB=4:1,vA:vB=2:1; C.RA:RB=1:4,vA:vB=1:2; D.RA:RB=1:4,vA:vB=2:1. 9.如图4质量为m的物体A放置在质量为M的物体B上,B与弹簧相连,它们一起在 光滑水平面上作简谐振动,振动过程中A、B之间无相对运动.设弹簧的倔强系 数为k.当物体离开平衡位置的位移为x时,A、B间摩擦力的大小等于 A.0; B.kx; 10.在如图5电路中,电键K1、K2、K3、K4均闭合,C是极板水平放置的平行板电 容 器,板间悬浮着一油滴P.断开哪一个电键后P会向下运动? ( ) A.K1 B.K2 C.K3 D.K4 11.图6表示一交流电的电流随时间而变化的图像.此交流电流的有效值是 ( ) 二、本题共7小题:每小题5分,共35分.在每小题给出的四个选项中,至少有一项 是正确的.全部选对的得5分,选对但不全的得2分,有错或不答的得0分. 12.在下面列举的物理量单位中,哪些是国际单位制的基本单位? ( ) A.千克(kg); B.米(m); C.开尔文(K);D牛顿(N). 13.关于机械波的概念,下列说法中正确的是 ( ) A.质点振动的方向总是垂直于波传播的方向; B.简谐波沿长绳传播,绳上相距半个波长的两质点振动位移的大小相 等; C.任一振动质点每经过一个周期沿波的传播方向移动一个波长; D.相隔一个周期的两时刻,简谐波的图像相同. 14.在静电场中 ( ) A.电场强度处处为零的区域内,电势也一定处处为零; B.电场强度处处相同的区域内,电势也一定处处相同; C.电场强度的方向总是跟等势面垂直的; D.沿着电场强度的方向,电势总是不断降低的. 15.已知介质对某单色光的临界角为θ,则( ) B.此单色光在该介质中的传播速度等于csinθ(c是真空中的光速); C.此单色光在该介质中的波长是在真空中波长的sinθ倍; 16.两个粒子,带电量相等,在同一匀强磁场中只受磁场力而作匀速圆周运 动. ( ) A.若速率相等,则半径必相等; B.若质量相等,则周期必相等; C.若动量大小相等,则半径必相等; D.若动能相等,则周期必相等. 17,一粒钢珠从静止状态开始自由下落,然后陷入泥潭中.若把在空中下落 的过程称为过程Ⅰ,进入泥潭直到停住的过程称为过程Ⅱ,则 ( ) A.过程Ⅰ中钢珠动量的改变量等于重力的冲量; B.过程Ⅱ中阻力的冲量的大小等于过程Ⅰ中重力冲量的大小; C.过程Ⅱ中钢珠克服阻力所做的功等于过程Ⅰ与过程Ⅱ中钢珠所减少 的重力势能之和; D.过程Ⅱ中损失的机械能等于过程Ⅰ中钢珠所增加的动能. 18.一弹簧振子作简谐振动,周期为T, ( ) A.若t时刻和(t+△t)时刻振子运动位移的大小相等、方向相同,则△t一定 等于T的整数倍; B.若t时刻和(t+△t)时刻振子运动速度的大小相等、方向相反, 第Ⅱ卷 三、本题共8小题;其中第24、25题每题6分,其余各题每题5分,共42分.把答案 填在题中的横线上. 19.一人坐在雪橇上,从静止开始沿着高度为15米的斜坡滑下,到达底部时 速度为10米/秒.人和雪橇的总质量为60千克,下滑过程中克服阻力做的功等于 焦(取g=10米/秒2). 20.图7给出氢原子最低的四个能级.氢原子在这些能级之间跃迁所辐射的 光子的频率最多有 种,其中最小的频率等于 赫.(保留两个数字) 21.已知质量为m的木块在大小为T的水平拉力作用下沿粗糙水平地面作匀 加速直线运动,加速度为a,则木块与地面之间的滑动摩擦因数为 .若在木块 上再施加一个与水平拉力T在同一竖直平面内的推力,而不改变木块速度的大 小和方向,则此推力与水平拉力T的夹角为 . 22.图8中a、b和c表示点电荷的电场中的三个等势面.它们的电势分 作用而运动.已知它经过等势面b时的速率为v,则它经过等势面c时的速率 为 . 23.图9中AB表示一直立的平面镜,P1P2是水平放置的米尺(有刻度的一面 朝着平面镜),MN是屏,三者互相平行.屏MN上的ab表示一条竖直的缝(即a、b之 间是透光的.)某人眼睛紧贴米尺上的小孔S(其位置见图),可通过平面镜看到米 尺的一部分刻度.试在本题的图上用三角板作图求出可看到的部位,并在P1P2 上把这部分涂以标志. 24.在研究电磁感应现象的实验中所用的器材如图10所示,它们是: (1)电流计,(2)直流电源,(3)带铁心的线圈A, (4)线圈B,(5)电键,(6)滑动变阻器.(用来控制电流以改变磁场强弱) 试按实验的要求在实物图上连线.(图中已连好一根导线) 若连接滑动变阻器的两根导线接在线柱C和D上,而在电键刚闭合时电流计 指针右偏,则电键闭合后滑动变阻器的滑动触头向接线柱C移动时,电流计指针 将 .(填"左偏"、"右偏"或"不偏") 25.在研究平抛物体运动的实验中,用一张印有小方格的纸记录轨迹,小方 格的边长l=1.25厘米.若小球在平抛运动途中的几个位置如图11中的a、b、c、 d所示,则小球平抛的初速度的计算式为v0= (用l、g表示),其值 是 .(取g=9.8米/秒2) 26.某人用万用电表按正确步骤测量一电阻阻值,指针指示位置如图12,则 这电阻值是 .如果要用这万用电表测量一个约200欧的电阻,为了使测量比 较精确,选择开关应选的欧姆挡是 . 四、本题包括4小题,共40分.解答应写出必要的文字说明、方程式和重要演算 步骤.只写出最后答案的不能得分.有数值计算的题,答案中必须明确写出数值 的单位. 27.(6分)一发光点S位于焦距为12厘米的薄凸透镜的主轴上.当S沿垂直于 主轴的方向移动1.5厘米时,它的像点S′移动0.5厘米.求移动前发光点S到像 点S′的距离. 28.(10分)两根相距d=0.20米的平行金属长导轨固定在同一水平面内,并处 于竖直方向的匀强磁场中,磁场的磁感应强度B=0.2特,导轨上面横放着两条金 属细杆,构成矩形回路,每条金属细杆的电阻为r=0.25欧,回路中其余部分的电 阻可不计.已知两金属细杆在平行于导轨的拉力的作用下沿导轨朝相反方向匀 速平移,速度大小都是v=5.0米/秒,如图13所示.不计导轨上的摩擦. (1)求作用于每条金属细杆的拉力的大小. (2)求两金属细杆在间距增加0.40米的滑动过程中共产生的热量. 29.(12分)一个质量可不计的活塞将一定量的理想气体封闭在上端开口的 直立圆筒形气缸内,活塞上堆放着铁砂,如图14所示.最初活塞搁置在气缸内壁 的固定卡环上,气体柱的高度为H0,压强等于大气压强p.现对气体缓慢加热,当 气体温度升高了△T=60K时,活塞(及铁砂)开始离开卡环而上升.继续加热直到 气柱高度为H1=1.5H0.此后,在维持温度不变的条件下逐渐取走铁砂,直到铁砂 全部取走时,气柱高度变为H2=1.8H0,求此时气体的温度.(不计活塞与气缸之间 的摩擦) 30.(12分)如图15所示,一排人站在沿x轴的水平轨道旁,原点O两侧的人的 序号都记为n(n=1,2,3…).每人只有一个沙袋,x>0一侧的每个沙袋质量为m=14 千克,x<0一侧的每个沙袋质量m′=10千克.一质量为M=48千克的小车以 某初速度从原点出发向正x方向滑行.不计轨道阻力.当车每经过一人身旁时,此 人就把沙袋以水平速度u朝与车速相反的方向沿车面扔到车上,u的大小等于扔 此袋之前的瞬间车速大小的2n倍.(n是此人的序号数) (1)空车出发后,车上堆积了几个沙袋时车就反向滑行? (2)车上最终有大小沙袋共多少个? 1995年全国普通高等学校招生统一考试 物理试卷答案及评分标准. 一、答案及评分标准:全题33分,每小题3分.答错的或不答的,都给0分. 1.(A) 2.(C) 3.(B) 4.(A) 5.(D) 6.(B) 7.(A) 8.(D) 9.(D) 10.(C) 11.(B) 二、答案及评分标准:全题35分,每小题5分,每小题全选对的给5分,选对但不全 的给2分,有选错的给0分,不答的给0分. 12.(A、B、C) 13.(B、D) 14.(C、D) 15.(A、B、C) 16.(B、C) 17.(A、C) 18.(C) 三、答案及评分标准:全题42分,其中24、25题各6分,其余的每小题5分.答案正 确的,按下列答案后面括号内的分数给分:答错的,不答的,都给0分. 19.6000……(5分) 20.6……(2分) 1.6×1014……(3分) 22.1.5v……(5分) 23.答案如图16 评分标准:全题5分,所定部位两端只要有一个错,就给零分.没有利用像 只按照入射角等于反射角而估画出的,只给1分. 24.图17所示为正确连线的一种.左偏. 评分标准:全题6分,电路图连接正确给4分,有任何错误都不给这4分.答 出"左偏"给2分. 26.1.2×103欧(3分,写成1.2×103欧或1200欧也给分;不写单位不给分)× 10(2分) 四、参考解答及评分标准 27.解:用h和h′分别表示S和S′移动的距离,用l表示S和S′未移动时的 距离,则有 l=u+v ② 根据透镜成像公式 由①②③式并代入数据可解得 l=64厘米 评分标准:全题6分,写出④式给2分,写出②式给2分,写出③式给1分.得出正 确结果再给1分. 28.解:(1)当两金属杆都以速度v匀速滑动时,每条金属杆中产生的感应电动 势分别为 ε1=ε=Bdv ① 由闭合电路的欧姆定律,回路中的电流强度 因拉力与安培力平衡,作用于每根金属杆的拉力的大小为 F1=F2=IBd ③ 由①②③式并代入数据得 (2)设两金属杆之间增加的距离为△L,则两金属杆共产生的热量 代入数据得 Q=1.28×10-2焦 评分标准:全题10分.第一问6分:求出①式给1分,求出②③式各得2分,结果 正确再给1分.第二问4分:求出④式给3分,结果正确再给1分.若用Q=F1△L 代替④式也同样给分. 29.第一种解法: 设气体最初温度为T0,则活塞刚离开卡环时温度为T0+△T,压强p1.由等容 升温过程得 设气柱高度为H1时温度为T1,由等压升温过程得 设气柱高度为H2时温度为T2,由等温膨胀过程(T2=T1)得 由①和③两式求得 由②和④两式得 代入数字得 T2=540K 评分标准:全题12分.求得①、②、③式各给3分.正确求得⑦式给2分,结果 正确再给1分(若利用①、②、③式得出正确结果而未写⑦式,也给这3分). 第二种解法: 设气体最初温度为T0,则活塞刚离开卡环时温度为T0+△T0.设气柱高度为 H1时温度为T1,高度为H2时温度为T2. 由等压升温过程得 利用T1=T2,由①、②两式解得 代入数值得 T2=540K 评分标准:全题12分.求得①式给4分;求得②式给5分;正确求得③式给2分, 结果正确再给1分(若利用①、②式得出正确结果而未写③式的,也给这3 分). 30.解: (1)在小车朝正x方向滑行的过程中,第(n-1)个沙袋扔到车上后的车速为vn-1, 第n个沙袋扔到车上后的车速为vn,由动量守恒定律有 小车反向运动的条件是vn-1>0,vn<0,即 M-nm>0 ② M-(n+1)m<0 ③ 代入数字,得 n应为整数,故n=3,即车上堆积3个沙袋后车就反向滑行. (2)车自反向滑行直到接近x<0一侧第1人所在位置时,车速保持不变,而车的 质量为M+3m.若在朝负x方向滑行过程中,第(n-1)个沙袋扔到车上后车速为 vn-1′,第n个沙袋扔到车上后车速为vn′,现取在图中向左的方向(负x方向)为速 度vn′、vn-1′的正方向,则由动量守恒定律有 车不再向左滑行的条件是 vn-1′>0,vn′≤0 即 M+3m-nm′>0 ⑤ M+3m-(n+1)m′≤0 ⑥ n=8时,车停止滑行,即在x<0一侧第8个沙袋扔到车上后车就停住.故车上最终共 有大小沙袋3+8=11个. 评分标准:全题12分.第(1)问4分:求得①式给2分,正确分析车反向滑行条件 并求得反向时车上沙袋数再给2分.(若未求得①式,但求得第1个沙袋扔到车上 后的车速,正确的也给2分.通过逐次计算沙袋扔到车上后的车速,并求得车开始 反向滑行时车上沙袋数,也再给2分.) 第(2)问8分:求得④式给3分,⑤式给1分,⑥式给2分.求得⑦式给1分.得到最后 结果再给1分.(若未列出⑤、⑥两式,但能正确分析并得到左侧n=8的结论,也可 给上述⑤、⑥、⑦式对应的4分.) 1996年全国普通高等学校招生统一考试 (全国卷) 一、本题共8小题;每小题4分,共32分.在每小题给出的四个选项中只有一项是正 确的. 1.下列核反应方程式中,表示核聚变过程的是( ). 2.红、橙、黄、绿四种单色光中,光子能量最小的是( ). (A)红光 (B)橙光 (C)黄光 (D)绿光 3.一平面线圈用细杆悬于P点,开始时细杆处于水平位置,释放后让它在如图 所示的匀强磁场中运动.已知线圈平面始终与纸面垂直,当线圈第一次通 过位置Ⅰ和位置Ⅱ时,顺着磁场的方向看去,线圈中感应电流的方向分别 为( ) 位置Ⅰ 位置Ⅱ (A)逆时针方向 逆时针方向 (B)逆时针方向 顺时针方向 (C)顺时针方向 顺时针方向 (D)顺时针方向 逆时针方向 4.只要知道下列哪一组物理量,就可以估算出气体中分子间的平均距 离?( ). (A)阿伏伽德罗常数、该气体的摩尔质量和质量 (B)阿伏伽德罗常数、该气体的摩尔质量和密度 (C)阿伏伽德罗常数、该气体的质量和体积 (D)该气体的密度、体积和摩尔质量 5.根据玻尔理论,氢原子的电子由外层轨道跃迁到内层轨道后( ). (A)原子的能量增加,电子的动能减少 (B)原子的能量增加,电子的动能增加 (C)原子的能量减少,电子的动能减少 (D)原子的能量减少,电子的动能增加 6.在右图所示的实验装置中,平行板电容器的极板A与一灵敏的静电计相接, 极板B接地.若极板B稍向上移动一点,由观察到的静电计指针变化作出平 行板电容器电容变小的结论的依据是( ). (A)两极板间的电压不变,极板上的电量变小 (B)两极板间的电压不变,极板上的电量变大 (C)极板上的电量几乎不变,两极板间的电压变小 (D)极板上的电量几乎不变,两极板间的电压变大 7.一焦距为f的凸透镜,主轴和水平的x轴重合.x轴上有一光点位于透镜的左 侧,光点到透镜的距离大于f而小于2f.若将此透镜沿x轴向右平移2f的距 离,则在此过程中,光点经透镜所成的象点将( ) (A)一直向右移动 (B)一直向左移动 (C)先向左移动,接着向右移动 (D)先向右移动,接着向左移动 8.质量为1.0千克的小球从高20米处自由下落到软垫上,反弹后上升的最大 高度为5.0米.小球与软垫接触的时间为1.0秒,在接触时间内小球受到合 力的冲量大小为( ).(空气阻力不计,g取10米/秒2) (A)10牛·秒 (B)20牛·秒 (C)30牛·秒 (D)40牛·秒 二、本题共6小题;每小题6分,共36分.在每小题给出的四个选项中,至少有一项 是正确的.全部选对的得6分,选对但不全的得2分,有选错或不答的得0分. 9.一物体作匀变速直线运动,某时刻速度的大小为4米/秒,1秒钟后速度的大 小变为10米/秒.在这1秒钟内该物体的( ). (A)位移的大小可能小于4米 (B)位移的大小可能大于10米 (C)加速度的大小可能小于4米/秒2 (D)加速度的大小可能大于10米/秒2 10.LC回路中电容两端的电压u随时刻t变化的关系如下图所示,则( ). (A)在时刻t1,电路中的电流最大 (B)在时刻t2,电路的磁场能最大 (C)从时刻t2至t3,电路的电场能不断增大 (D)从时刻t3至t4,电容的带电量不断增大 11.如图a,b,c是一条电力线上的三个点,电力线的方向由a到c,a、b间的距离 等于b、c间的距离。用Ua、Ub、Uc和Ea、Eb、Ec分别表示a、b、c三点 的电势和电场强度,可以断定( ). (A)UaUbUc (B)EaEbEc (C)Ua-Ub=Ub-Uc (D)Ea=Eb=Ec 12.一根张紧的水平弹性长绳上的a、b两点,相距14.0米,b点在a点的右方.当 一列简谐横波沿此长绳向右传播时,若a点的位移达到正极大时,b点的位 移恰为零,且向下运动.经过1.00秒后,a点的位移为零,且向下运动,而b点 的位移恰达到负极大,则这简谐横波的波速可能等于( ). (A)4.67米/秒 (B)6米/秒 (C)10米/秒 (D)14米/秒 13.半径相等的两个小球甲和乙,在光滑水平面上沿同一直线相向运动.若甲 球的质量大于乙球的质量,碰撞前两球的动能相等,则碰撞后两球的运动 状态可能是( ). (A)甲球的速度为零而乙球的速度不为零 (B)乙球的速度为零而甲球的速度不为零 (C)两球的速度均不为零 (D)两球的速度方向均与原方向相反,两球的动能仍相等 14. 如果表中给出的是作简谐振动的物体的位移x或速度v与时刻的对应关系,T 是振动周期,则下列选项中正确的是( ). (A)若甲表示位移x,则丙表示相应的速度v (B)若丁表示位移x,则甲表示相应的速度v (C)若丙表示位移x,则甲表示相应的速度v (D)若乙表示位移x,则丙表示相应的速度v 三、本题共3小题;其中第15题5分,其余的每题6分,共17分.把答案填在题中的横 线上或按题目要求作图. 15.在"验证机械能守恒定律"的实验中,已知打点计时器所用电源的频率为 50赫.查得当地的重力加速度g=9.80米/秒2.测得所用的重物的质量为 1.00千克.实验中得到一条点迹清晰的纸带,把第一个点记作0,另选连续 的4个点A、B、C、D作为测量的点.经测量知道A、B、C、D各点到0 点的距离分别为62.99厘米、70.18厘米、77.76厘米、85.73厘米.根据以 上数据,可知重物由0点运动到C点,重力势能的减少量等于 焦,动能 的增加量等于 焦(取3位有效数字). 16.在用电流场模拟静电场描绘电场等势线的实验中,在下列所给出的器材 中,应该选用的是 (用器材前的字母表示). (A)6伏的交流电源 (B)6伏的直流电源 (C)100伏的直流电源 (D)量程0~0.5伏,零刻度在刻度盘中央的电压表 (E)量程0~300微安,零刻度在刻度盘中央的电流表在实验过程中,要把复 写 纸、导电纸、白纸铺放在木板上,它们的顺序(自上而下)是① ② ③ . 在实验中,按下电键,接通电路.若一个探针与基准点O接触,另一 探针已分别在基准点O的两侧找到了实验所需要的两点a、b(如右上 图),则当此探针与a点接触时,电表的指针应 (填"左偏"、"指零"或" 右偏");当此探针与b点接触时,电表的指针应 (填"左偏"、"指零"或"右偏"). 17.在用伏安法测电阻的实验中,所用电压表的内阻约为20千欧,电流表的内 阻约为10欧,选择能够尽量减小误差的电路图接线进行实验,读得的各 组数据用实心圆点标于坐标图上(如右图所示). (1)根据各点表示的数据描出I-U图线,由此求得该电阻的阻值 Rx= 欧(保留两位有效数字). (2)画出此实验的电路原理图. 四、本题共4小题;每小题5分,共20分.把答案填在题中的横线上,或按题目要求 作图. 18.如右图所示,一细导体杆弯成四个拐角均为直角的平面折线,其ab、cd段 长度均为l1,bc段长度为l2.弯杆位于竖直平面内,Oa、dO′段由轴承支撑 沿水平放置.整个弯杆置于匀强磁场中,磁场方向竖直向上,磁感应强度 为B.今在导体杆中沿abcd通以大小为I的电流,此时导体杆受到的安培 力对OO′轴的力矩大小等于 . 19.右图中abcd为一边长为l、具有质量的刚性导线框,位于水平面内,bc边中 串接有电阻R,导线的电阻不计.虚线表示一匀强磁场区域的边界,它与 线框的ab边平行.磁场区域的宽度为2l,磁感应强度为B,方向竖直向下. 线框在一垂直于ab边的水平恒定拉力作用下,沿光滑水平面运动,直到 通过磁场区域.已知ab边刚进入磁场时,线框便变为匀速运动,此时通过 电阻R的电流的大小为i0,试在右图的i-x坐标上定性画出:从导线框刚进 入磁场到完全离开磁场的过程中,流过电阻R的电流i的大小随ab边的位 置坐标x变化的曲线. 20.如图所示,倔强系数为k1的轻质弹簧两端分别与质量为m1、m2的物块1、 2拴接,倔强系数为k2的轻质弹簧上端与物块2拴接,下端压在桌面上(不 拴接),整个系统处于平衡状态.现施力将物块1缓慢竖直上提,直到下面 那个弹簧的下端刚脱离桌面.在此过程中,物块2的重力势能增加了 , 物块1的重力势能增加了 . 21.在光滑水平面上有一静止的物体.现以水平恒力甲推这一物体,作用一段 时间后,换成相反方向的水平恒力乙推这一物体.当恒力乙作用时间与 恒力甲作用时间相同时,物体恰好回到原处,此时物体的动能为32焦,则 在整个过程中,恒力甲做的功等于 焦,恒力乙做的功等于 焦. 五、本题共5小题,45分.解答应写出必要的文字说明、方程式和重要演算步骤. 只写出最后答案的不能得分.有数值计算的题,答案中必须明确写出数值和单 位. 22.(5分)一物块从倾角为θ、长为s的斜面的项端由静止开始下滑,物块与斜 面的滑动摩擦系数为μ,求物块滑到斜面底端所需的时间. 23.(8分)在折射率为n、厚度为d的玻璃平板上方的空气中有一点光源S,从S 发出的光线SA以角度θ入射到玻璃板上表面,经过玻璃板后从下表面射出,如 右图所示.若沿此光线传播的光从光源到玻璃板上表面的传播时间与在玻璃板 中的传播时间相等,点光源S到玻璃上表面的垂直距离l应是多少? 24.(8分)一质量为M的长木板静止在光滑水平桌面上.一质量为m的小滑块 以水平速度v0从长木板的一端开始在木板上滑动,直到离开木板.滑块 25.(12分)如图所示,有一个直立的气缸,气缸底到气缸口的距离为L0厘米, 用一厚度和质量均可忽略不计的刚性活塞A,把一定质量的空气封在气缸内,活 塞与气缸间的摩擦可忽略.平衡时活塞上表面与气缸口的距离很小(计算时可忽 略不计),周围大气的压强为H0厘米水银柱.现把盛有水银的一个瓶子放在活塞 上(瓶子的质量可忽略),平衡时活塞到气缸底的距离为L厘米.若不是把这瓶水 银放在活塞上,而是把瓶内水银缓缓不断地倒在活塞上方,这时活塞向下移,压 缩气体,直到活塞不再下移.求此时活塞在气缸内可能的位置以及与之相对应的 条件(即题中给出量之间应满足的关系).设气体的温度不变. 26.(12分)设在地面上方的真空室内存在匀强电场和匀强磁场.已知电场强 度和磁感应强度的方向是相同的,电场强度的大小E=4.0伏/米,磁感应强度的大 小B=0.15特.今有一个带负电的质点以v=20米/秒的速度在此区域内沿垂直场强 方向做匀速直线运动,求此带电质点的电量与质量之比q/m以及磁场的所有可 能方向(角度可用反三角函数表示). 参考解答 一、1.B 2.A 3.B 4.B 5.D 6.D 7.C 8.C 二、9.A、D 10.B、C 11.A 12.A、C 13.A、C 14.A、B 三、15.7.62,7.56 16.B、E ①导电纸 ②复写纸 ③白纸 指零,指零 17.舍去不合理点的直线,如下图,2.4×103 四、18.IBl1l2 19.x (0→l) i=i0 (l→2l) i=0 (2l→3l)如图,且x=3l处i≥I0 21.8,24 五、22.设物块质量为m,加速度为a,物块受力情况如下图所示, mgsinθ-f=ma, N-mgcosθ=0, f=μN, 解得 a=gsinθ-μgcosθ, 23.设光线在玻璃中的折射角为r,则光线从S到玻璃板上表面的传播 由折射定律 sinθ=nsinr, 24.设第一次滑块离开时木板速度为v,由系统的动量守恒,有 设滑块与木板间摩擦力为f,木板长L,滑行距离s,如右图,由动能定理 25.设整瓶水银放在活塞上后,使气缸内气体增加的压强为h厘米水银柱,由 玻意耳-马略特定律H0L0=(H0+h)L, (1) h的大小反映了水银质量的大小. 当水银注入后,活塞不再下移时,设活塞上水银的深度为△H厘米,活塞下 移的距离为△x厘米,则由玻意耳-马略特定律 H0L0=(H0+△H)(L0-△x), (3) 可能发生两种情况: 1.水银比较少,瓶内水银全部注入后,尚未灌满或刚好灌满活塞上方的气 缸,这时 △H=h, (5) △H≤△x, (6) 由(2)、(4)、(5)三式,得 △x=L0-L, (7) 活塞到气缸底的距离 L′=L0-△x=L, (8) 由(4)、(6)、(7)三式,得 L≥H0, (9) 即若L≥H0,则L′=L. 2.瓶内水银比较多,当活塞上方的气缸灌满水银时,瓶内还剩有一定量的 水银,这时 △H=△x, (10) △H2a (D)a'=2a 4.(1)、(2)两电路中,当a、b两端与e、f两端分别加上220伏的交流电压时,测得c、 d间与g、h间的电压均为110伏.若分别在c、d两端与g、h两端加上110伏的交 流电压,则a、b间与e、f间的电压分别为 (A)220伏,220伏 (B)220伏,110伏 (C)110伏,110伏 (D)220伏,0 5.在双缝干涉实验中,以白光为光源,在屏幕上观察到了彩色干涉条纹,若在双 缝中的一缝前放一红色滤光片(只能透过红光),另一缝前放一绿色滤光片(只 能透过绿光),这时 (A)只有红色和绿色的双缝干涉条纹,其它颜色的双缝干涉条纹消失 (B)红色和绿色的双缝干涉条纹消失,其它颜色的双缝干涉条纹依然存在 (C)任何颜色的双缝干涉条纹都不存在,但屏上仍有光亮 (D)屏上无任何光亮 二.本题共9小题;每小题5分,共45分.在每小题给出的四个选项中,有的小题只有 一个选项正确,有的小题有多个选项正确.全部选对的得5分,选不全的得2分, 有选错或不答的得0分. 6.在下列核反应方程中,x代表质子的方程是 7.光线在玻璃和空气的分界面上发生全反射的条件是 (A)光从玻璃射到分界面上,入射角足够小 (B)光从玻璃射到分界面上,入射角足够大 (C)光从空气射到分界面上,入射角足够小 (D)光从空气射到分界面上,入射角足够大 8.在下列叙述中,正确的是 (A)物体的温度越高,分子热运动越剧烈,分子平均动能越大 (B)布朗运动就是液体分子的热运动 (C)对一定质量的气体加热,其内能一定增加 (D)分子间的距离r存在某一值r0,当rr0时,斥力小于 引力 9.图中重物的质量为m,轻细线AO和BO的A、B端是固定的.平衡时AO是水平 的,BO与水平面的夹角为θ.AO的拉力F1和BO的拉力F2的大小是 10.为了增大LC振荡电路的固有频率,下列办法中可采取的是 (A)增大电容器两极板的正对面积并在线圈中放入铁芯 (B)减小电容器两极板的距离并增加线圈的匝数 (C)减小电容器两极板的距离并在线圈中放入铁芯 (D)减小电容器两极板的正对面积并减少线圈的匝数 11.简谐横波某时刻的波形图线如图所示.由此图可知 (A)若质点a向下运动,则波是从左向右传播的 (B)若质点b向上运动,则波是从左向右传播的 (C)若波从右向左传播,则质点c向下运动 (D)若波从右向左传播,则质点d向上运动 12.如图所示的电路中,电源的电动势恒定,要想使灯泡变暗,可以 (A)增大R1 (B)减小R1 (C)增大R2 (D)减小R2 13.如图所示的电路中,A1和A2是完全相同的灯泡,线圈L的电阻可以忽略.下列 说法中正确的是 (A)合上开关K接通电路时,A2先亮,A1后亮,最后一样亮 (B)合上开关K接通电路时,A1和A2始终一样亮 (C)断开开关K切断电路时,A2立刻熄灭,A1过一会儿才熄灭 (D)断开开关K切断电路时,A1和A2都要过一会儿才熄灭 14.在图示电路的三根导线中,有一根是断的,电源、电阻器R1、R2及另外两根导 线都是好的.为了查出断导线,某学生想先将万用表的红表笔连接在电源的 正极a,再将黑表笔分别连接在电阻器R1的b端和R2的c端,并观察万用表指针 的示数.在下列选挡中,符合操作规程的是 (A)直流10V挡 (B)直流0.5A挡 (C)直流2.5V挡 (D)欧姆挡 第Ⅱ卷 (非选择题 共90分) 注意事项: 1.第Ⅱ卷共8页,用钢笔或圆珠笔直接答在试题卷中(除题目有特殊规定外). 2.答卷前将密封线内的项目填写清楚. 三.本题共3小题;其中第15题5分,其余的每题6分,共17分.把答案填在题中的横 线上或按题目要求作图. 15.一游标卡尺的主尺最小分度为1毫米,游标上有10个小等分间隔,现用此卡尺 来测量工件的直径,如图所示.该工件的直径为_____________毫米. 16.下列给出的器材中,哪些是"验证玻一马定律实验"所必需的,把这些器材前面 的字母填在横线上. A.带有刻度的注射器 B.刻度尺 C.弹簧秤 D.钩码若干个 答:_____________. 实验读数过程中,不能用手握住注射器,这是为了________________. 用橡皮帽封住注射器小孔,这是为了___________________________. 17.某电压表的内阻在20千欧~50千欧之间,现要测量其内阻,实验室提供下列 可选用的器材: 待测电压表V(量程3V) 电流表A1(量程200μA ) 电流表A2(量程5mA) 电流表A3(量程0.6A) 滑动变阻器R(最大阻值1KΩ) 电源ε(电动势4V) 电键K. (1)所提供的电流表中,应选用_______________________(填写字母代号). (2)为了尽量减小误差,要求测多组数据.试在方框中画出符合要求的实验电 路图(其中电源和电键及其连线已画出). 四.本题共4小题;每小题5分,共20分.把答案填在题中横线上. 18.如图,在x轴的上方(y≥0)存在着垂直于纸面向外的匀强磁场,磁感应强度为B. 在原点O有一个离子源向x轴上方的各个方向发射出质量为m、电量为q的正 离子,速率都为v.对那些在xy平面内运动的离子,在磁场中可能到达的最大 x=________________,最大y=________________. 19.质量为m、电量为q的质点,在静电力作用下以恒定速率v沿圆弧从A点运动到 B点,其速度方向改变的角度为θ(弧度),AB弧长为s.则A,B两点间的电势差 UA-UB=________________,AB弧中点的场强大小E=________________. 20.已知地球半径约为6.4×106米,又知月球绕地球的运动可近似看作匀速圆周 运动,则可估算出月球到地心的距离约为________________米.(结果只保留 一位有效数字) 21.一内壁光滑的环形细圆管,位于竖直平面内,环的半径为R(比细管的半径大 得多).在圆管中有两个直径与细管内径相同的小球(可视为质点).A球的质量 为m1,B球的质量为m2.它们沿环形圆管顺时针运动,经过最低点时的速度都 为v0.设A球运动到最低点时,B球恰好运动到最高点,若要此时两球作用于圆 管 的 合 力 为 零 , 那 么 m1,m2,R 与 v0 应 满 足 的 关 系 式 是 _________________________. 五.本题共5小题,53分.解答应写出必要的文字说明、方程式和重要演算步骤.只 写出最后答案的不能得分.有数值计算的题,答案中必须明确写出数值和单 位. 22.(9分)有一个焦距为36厘米的凸透镜,在主轴上垂直放置一支蜡烛,得到一个 放大率为4的虚像.如果想得到放大率为4的实像,蜡烛应向哪个方向移动?移动 多少? 23.(9分)图中竖直圆筒是固定不动的,粗筒横截面积是细筒的4倍,细筒足够长. 粗筒中A、B两轻质活塞间封有空气,气柱长l=20厘米.活塞A上方的水银深H=10 厘米,两活塞与筒壁间的摩擦不计.用外力向上托住活塞B,使之处于平衡状态, 水银面与粗筒上端相平.现使活塞B缓慢上移,直至水银的一半被推入细筒中,求 活塞B上移的距离.设在整个过程中气柱的温度不变,大气压强p0相当于75厘米 高的水银柱产生的压强. 24.(11分)在方向水平的匀强电场中,一不可伸长的不导电细线的一端连着一个 质量为m的带电小球,另一端固定于O点.把小球拉起直至细线与场强平行,然后 无初速释放.已知小球摆到最低点的另一侧,线与竖直方向的最大夹角为θ(如 图).求小球经过最低点时细线对小球的拉力. 25.(12分)质量为m的钢板与直立轻弹簧的上端连接,弹簧下端固定在地上.平衡 时,弹簧的压缩量为x0,如图所示.一物块从钢板正上方距离为3x0的A处自由落 下,打在钢板上并立刻与钢板一起向下运动,但不粘连.它们到达最低点后又向 上运动.已知物块质量也为m时,它们恰能回到O点.若物块质量为2m,仍从A处自 由落下,则物块与钢板回到O点时,还具有向上的速度.求物块向上运动到达的最 高点与O点的距离. 26.(12分)如图1所示,真空室中电极K发出的电子(初速不计)经过U0=1000伏的 加速电场后,由小孔S沿两水平金属板A、B间的中心线射入.A、B板长l=0.20米, 相距d=0.020米,加在A、B两板间的电压u随时间t变化的u-t图线如图2所示.设A、 B间的电场可看作是均匀的,且两板外无电场.在每个电子通过电场区域的极短 时间内,电场可视作恒定的.两板右侧放一记录圆筒,筒在左侧边缘与极板右端 距离b=0.15米,筒绕其竖直轴匀速转动,周期T=0.20秒,筒的周长s=0.20米,筒能接 收到通过A、B板的全部电子. (1)以t=0时(见图2,此时u=0)电子打到圆筒记录纸上的点作为xy坐标系的原 点,并取y轴竖直向上.试计算电子打到记录纸上的最高点的y坐标和x坐 标.(不计重力作用) (2)在给出的坐标纸(图3)上定量地画出电子打到记录纸上的点形成的图线. 图1 图2 图3 1997年普通高等学校招生全国统一考试 物理试题答案及评分标准 说明: (1)定出评分标准是为了使全国各地尽可能在统一标准下评定成绩.试题的 参考解答是用来说明评分标准的.考生如按其它方法或步骤解答,正确的,同样 给分;有错的,根据错误的性质,参照评分标准中相应的规定评分. (2)第一、二、三、四题只要求写出答案,不要求说明理由或列出算式,只根 据答案评分. (3)第五大题,只有最后后答案而无演算过程的,不给分;只写出一般公式但 未能与试题所给的具体条件联系的,不给分. 一.答案及评分标准:全题15分,每小题3分.答错的或不答的,都给0分. 1.A 2.D 3.C 4.B 5.C 二.答案及评分标准:全题45分,每小题5分.每小题全选对的给5分,选不全的给2 分,有选错的给0分,不答的给0分. 6.B、C 7.B 8.A、D 9.B、D 10.D 11.B、D 12.A、D 13.A、D 14.A 三.答案及评分标准:全题17分,其中15题5分,其余的每题6分.答案正确的,按下 列答案后面括号内的分数给分;答错的,不答的,都给0分. 15.29.80 (5分,答29.8的同样给5分) 16.A,B,C,D (2分.选不全的给0分) 保持气体的温度恒定 (2分) 保持气体的质量不变 (2分) 17.A1 (2分), 如右图 (4分,线路有错就不给这4分) 四.答案及评分标准:全题20分,每小题5分,答案正确的,按下列答案后面括号内 的分数给分;答错的,不答的,都给0分. 20.4×108 (5分.只要数量级对,就给5分) 五.参考解答及评分标准: 22.解:先求蜡烛的原位置 由放大率公式 得 v1=4u1 ① 由透镜成像公式 ② 解得 再求蜡烛移动后的位置,由放大率公式得 v2=4u2③ 由透镜成像公式 ④ 解得 所以蜡烛应向远离透镜的方向移动,移动的距离为 评分标准:本题9分. ①式2分,②式1分,③式2分,④式1分,⑤式2分. 物体移动方向正确的给1分. 23.解: 在以下的计算中,都以1厘米汞柱产生的压强作为压强的单位. 设气体初态的压强为p1,则有 p1=p0+H ① 设S为粗圆筒的横截面积,气体初态的体积V1=Sl. 设气体末态的压强为P2,有 ② 设末态气柱的长度为l',气体体积为V2=Sl' 由玻意耳定律得 P1V1=P2V2③ 活塞B上移的距离d为 ④ 代入数据解得 d=8厘米 ⑤ 评分标准:本题9分. ①式1分,②式2分,③式1分,④式3分,⑤式2分. 24.解:设细线长为l,球的电量为q,场强为E.若电量q为正,则场强方向在题图中向 右,反之向左.从释放点到左侧最高点,重力势能的减少等于电势能的增加, mglcosθ=qEl(1+sinθ)① 若小球运动到最低点时的速度为v,此时线的拉力为T,由能量关系得 ② 由牛顿第二定律得 ③ 由以上各式解得 ④ 评分标准:本题11分. ①、②式各3分,③式2分,④式3分. 25.解:物块与钢板碰撞时的速度 ① 设v1表示质量为m的物块与钢板碰撞后一起开始向下运动的速度,因碰撞 时间极短,动量守恒, mv0=2mv1 ② 刚碰完时弹簧的弹性势能为EP.当它们一起回到O点时,弹簧无形变,弹性势 能为零,根据题给条件,这时物块与钢板的速度为零,由机械能守恒, ③ 设v2表示质量为2m的物块与钢板碰撞后开始一起向下运动的速度,则有 2mv0=3mv2 ④ 仍继续向上运动,设此时速度为v,则有 ⑤ 在以上两种情况中,弹簧的初始压缩量都是x0,故有 ⑥ 当质量为2m的物块与钢板一起回到O点时,弹簧的弹力为零,物块与钢板只 受到重力作用,加速度为g.一过O点,钢板受到弹簧向下的拉力作用,加速度大于 g.由于物块与钢板不粘连,物块不可能受到钢板的拉力,其加速度仍为g.故在O 点物块与钢板分离,分离后,物块以速度v竖直上升,则由以上各式解得,物块向上 运动所到最高点与O点的距离为 ⑦ 评分标准:本题12分. ①、②、③、④式各1分,⑤式2分,⑥式3分,得出⑦式再给3分. 26.解:(1)计算电子打到记录纸上的最高点的坐标 设v0为电子沿A、B板的中心线射入电场时的初速度,则 ① 电子在中心线方向的运动为匀速运动,设电子穿过A、B板的时间为t0,则 l=v0t0 ② 图(1) 电子在垂直A、B板方向的运动为匀加速直线运动.对于恰能穿过A、B板的 电子,在它通过时加在两板间的电压uc应满足 ③ 联立①、②、③式解得 此电子从A、B板射出时沿y方向的分速度为 ④ 此后,此电子作匀速直线运动,它打在记录纸上的点最高,设纵坐标为y,由图 (1)可得 ⑤ 由以上各式解得 ⑥ 从题给的ut图线可知,加于两板电压u的周期T0=0.10秒,u的最大值um=100 伏,因为uC7,AG>0 D.在 25℃时,若溶液呈中性,则 AG=2(7-pH) 10.常温下,将甲酸和氢氧化钠溶液混合,所得溶液 pH=7,则此溶液中 A. [HCOO—]>[Na+] B. [HCOO—]<[Na+] C. [HCOO—]=[Na+] D. 无法确定[HCOO—]和[Na+]的关系 11.两种元素原子的核外电子层数之比与最外层电子数之比相等,则在周期表的前 10 号元素中, 满足上述关系的元素共有 A. 1 对 B. 2 对 C. 3 对 D. 4 对 12.用足量的 co 还原 32.0g 某种氧化物,将生成的气体通入足量澄清石灰水中,得到 60g 沉淀, 则该氧化物是 A. FeO B. Fe2O3 C. CuO D. Cu2O 13.化合价为 n 的某种元素的硝酸盐的式量为 x ,其氢氧化物的式量为 y ,则 n 的值是 A. 45 yx  B. 45 xy  C. 79 yx  D. 79 xy  14. L 多巴是一种有机物,它可用于帕金森综合症的治疗,其结构简式如下: OH CHCOOHCH 2 2NH HO 这种药物的研制是基于获得 2000 年诺贝尔生理学或医学奖和获得 2001 年诺贝尔化学奖 的研究成果。下列关于 L 多巴酸碱性的叙述正确的是 A. 既没有酸性,又没有碱性 B. 既具有酸性,又具有碱性 C. 只有酸性,没有碱性 D. 只有碱性,没有酸性 15.目前普通认为,质子和中子都是由被称为u 夸克和 d 夸克的两类夸克组成。u 夸克带电量为 e3 2 , d 夸克带电量为 e3 1 , e 为基元电荷。下列论断可能正确的是 A. 质子由 1 个u 夸克和 1 个 d 夸克组成,中子由 1 个u 夸克和 2 个 d 夸克组成 B. 质子由 2 个u 夸克和 1 个 d 夸克组成,中子由 1 个u 夸克和 2 个 d 夸克组成 C. 质子由 1 个u 夸克和 2 个 d 夸克组成,中子由 2 个u 夸克和 1 个 d 夸克组成 D. 质子由 2 个u 夸克和 1 个 d 夸克组成,中子由 1 个u 夸克和 1 个 d 夸克组成 16.在光滑水平地面上有两个相同的弹性小球 A、B,质量都为 m。现 B 球静止,A 球向 B 球运 动,发生正碰。已知碰撞过程中总机械能守恒,两球压缩最紧时的弹性势能为 Ep,,则碰前 A 球的速度等于 A. m E p B. m E p2 C. 2 m E p D. 2 m E p2 17.图中 EF、GH 为平行的金属导轨,其电阻可不计,R 为电阻器,G 为电容器,AB 为可在 EF 和 GH 上滑动的导体横杆。有均匀磁场垂直与导轨平面。若用 I1 和 I2 分别表示图中该处导线 中的电流,则当横杆 AB A. 匀速滑动时,I1=0,I2=0 B. 匀速滑动时,I1≠0,I2≠0 C. 加速滑动时,I1=0,I2=0 D. 加速滑动时,I1≠0,I2≠0 18.质点所受的力 F 随时间变化的规律如图所示,力的方向 始终在一直线上。已知 t=0 时质点的速度为零。在图示 t1、 t2、t3 和 t4 各时刻中,哪一时刻质点的动能最大? A.t1 B.t2 C.t3 D.t4 19.为了观察门外情况,有人在门上开一小圆孔,将一块圆柱形玻璃嵌入其中,圆柱体轴线与门 面垂直。从圆柱底面中心看出去,可以看到的门外人射光线与轴线间的最大夹角称做视场角。 已知该玻璃的折射率为 n ,圆柱长为l ,底面半径为 r, 则视场角是 A. 22 arcsin lr nl  B. 22 arcsin lr nr  C. 22 arcsin lrn r  D. 22 arcsin lrn l  20.在如图所示的电路中,R1、R2、R3 和 R4 皆为定值电阻,R5 为可变电阻,电源的电动势为。, 内阻为 r。设电流表 A 的读数为 I,电压表 V 的读数为 U.当 R5 的滑动触点向图中 a 端移动 时, A.I 变大,U 变小 B.I 变大,U 变大 C.I 变小,U 变大 D.I 变小,U 变小 2002 年普通高等学校招生全国统一考试(全国卷) 理科综合能力测试 本试卷分第Ⅰ卷(选择题)和第Ⅱ卷(非选择题)两部分,第Ⅰ卷 1 至 5 页,第Ⅱ卷 6 至 15 页,满分 300 分。考试用时 150 分钟。 第Ⅱ卷 (非选择题 共 10 题 共 180 分) 注意事项: 1.用钢笔或圆珠笔直接答在试题卷中(除题Ⅱ有特殊规定外) 2.答卷前将密封线内的项目填写清楚。 题 号 21 22 23 24 25 26 27 28 29 30 总 分Ⅰ Ⅱ Ⅰ Ⅱ 分 数 21.(15 分)为验证“镁是植物生活的必需元素”,三位同学进行了实验设计, 下列是实验的基本思路。请分别指出三个实验思路能否达到实验目的, 得分 评卷人 28 29 Ⅰ Ⅱ Ⅰ Ⅱ 为 什么?再写出你的设计思路。 (1)实验一:取生长状况一致的大豆幼苗,用符合实验要求的容器进行培养。对照组容器内只 盛有蒸馏水,实验组盛有用蒸馏水配制的镁盐溶液。两组置于相同的适宜条件下培养,并 对溶液通气,观察比较两组植物的生长发育情况。 答: 得 分 (2)实验二:取生长状况一致的大豆幼苗,栽培在盛有砂性土壤的容器中(砂性上壤肥力均匀, 容器符合实验要求),对照组浇以蒸馏水,实验组浇以蒸馏水配制的镁盐溶液,两组置于 相同的适宜条件下培养,观察比较两组植物的生长发育情况。 答: 得 分 (3)实验三:取生长状况一致的大豆幼苗,栽培在盛有砂性土壤的容器中(破性土壤肥力均匀, 容器符合实验要求),对照组浇以含有植物必需的各种元素的完全营养液,实验组浇以不含 镁离子的完全营养液,两组置于相同的适宜条件下培养,观察比较两组植物的生长发育情 况。 答: 得 分 (4)下面由你进行实验设计,请写出你的设计思路。 答: 得 分 22.(9 分)科学家应用生物技术培育出了一种抗虫棉,它能产生毒素,杀 死 害虫,目前正在大面积推广种植。科学家还研究了害虫的邀传基础,发 现不抗毒素对抗毒素为显性(此外分别用 B 和 b 表示)。据此 回答: (1)种植抗虫棉,有利于生态环境保护,这是因为_____________________________。 得分 评卷人 得 分 (2)棉田不抗霉渍害虫的基因型为___________;抗毒素害虫的基因型为________________。 得 分 (3)不抗毒案害虫与抗毒素害虫杂交,则子代的基因型为_____________________________。 得 分 23.(12 分)如图所示:淀粉水解可产生某有机化合物 A 人在不同的氧化剂 作用下,可以生成 B(C6H14O6)或 C(C6H10O8),B 和 C 都不能发生银 镜反 应。A、B 和 C 都可以被强还原剂还原成为 D(C6H14O6)。B 脱水可得到 五元环的酯类化合物 E 或六元环的酯类化合物 F。 已知,相关物质被氧化的难易次序是: RCHO 最易,R-CH2OH 次之, 最难。 请在下列空格中填写 A、B、C、D、E、F 的结构简式。 A:______________ B:______________ C:_______________ D:______________ E:______________ F:_______________ 得 分 24.(14 分)在 25℃,101KPa 条件下,将 15L O2 通人 10L CO 和 H2 的混合 气中,使其完全燃烧,于燥后,恢复至原来的温度和压强。 (1)若剩余气体的体积是 15 L,则原 CO 和 H2 的混合气中 V(CO)=_________L,V(H2)=___________ L. 得 分 (2)若剩余气体的体积为 a L,则原 CO 和 H2 的混合气中 得分 评卷人 得分 评卷人 V(CO):V(H2)=______。 得 分 (3) 若剩余气体的体积为 aL,则 a 的取值范围是_____________。 25.(12 分)已知: ①A、B、C、D 四种物质均含元素 X,有的还可能合有元素 Y、Z。元素 Y、 X、Z 的原子序数依次递增。 ②X 在 A、B、C、D 中都不呈现它的最高化合价。 ③室温下单质 A 与某种常见一元强碱溶液反应,可得到 B 和 C. ④化合物 D 受热催化分解,可制得元素 Y 的单质。 (1)元素 X 是___________________________,Z 是__________________________。 得 分 (2)写出③中反应的化学方程式:_______________________________________。 得 分 (3)写出④中反应的化学方程式:_______________________________________。 得 分 26.(20 分)蹦床是运动员在一张绷紧的弹性网上蹦跳、翻滚并做各种空中 动 作的运动项目。一个质量为 60 kg 的运动员,从离水平网面 3.2m 高处自 由下落,着网后沿竖直方向蹦回到离水平网面 5.0m 高处。已知运动员 与 网接触的时间为 1.2s。若把在这段时间内网对运动员的作用力当作恒力处理,求此力的大小。 (g=10m/s2) 得分 评卷人 得分 评卷人 得 分 27.(20 分)电视机的显像管中,电子束的们转是用磁偏转技术实现的。电 子 束经过电压为 U 的加速电场后,进人一圆形匀强磁场区,如图所示。磁 场方向垂直于圆面。磁场区的中心为 O,半径为 r。当不加磁场时,电子 束将通过 O 点而打到屏幕的中心 M 点。为了让电子束射到屏幕边缘 P,需要加进场,使电 于束们转一己知角度 ,此时磁场的磁感应强度 B 应为多少? 得 分 28.(14 分)昆虫能分泌信息素。下列是一种信息素的结构简式; CH3(CH2)5CH=CH(CH2)9CHO Ⅰ.(8 分)指出该物质中的任意..一种官能团,为该官能团提供一种简单的 鉴 别方法并简述实验现象,写出与鉴别方法有关反应的化学方程式并指出 反应类型,完成下表。 得分 评卷人 得分 评卷人 官能团 鉴别方法及 实验现象 有关反应的化学方程式 反应类别 得 分 Ⅱ.(6 分)昆虫的性外激素属于信息素,根据其化学结构可以合成性引诱剂,性引诱剂可以用 于 防治害虫,请写出如何将性引诱剂用于害虫的防治。 ①__________________________________________________________________。 得 分 ②__________________________________________________________________。 29.(37 分)大气压强对许多物理实验和化学实验有着重要影响。 Ⅰ.(17 分)现用“验证玻意耳定律”的仪 器来测量大气压强 p0。注射器针筒已被 固定在竖直方向上,针筒上所标刻度是 注射器的容积,量大刻度 Vm=10ml。注射器活塞已装上钩码 框架,如图所示。此外,还有一架托盘天平、若干钩码、一把 米尺、一个针孔橡皮冒和少许润滑油。 (1)下面是实验步骤,试填写所缺的②和⑤。 ①用米尺测出注射器针筒上全部刻度的长度 L。 ②_____________________________________________________________。 ③把适量的润滑油抹在注射器的活塞上,将活塞插入针筒中,上下拉动活塞,使活塞 与 针筒的间隙内均匀地涂上润滑油。 得分 评卷人 ④将活塞插到适当的位置。 ⑤_____________________________________________________________。 ⑥在钩码框架两侧挂上钩码,记下挂上的钩码质量 m1。小达到平衡后,记下注射 器中空气柱的体积 V1。在这个过程中不要用手接触注射器以保证空气柱温度不变。 ⑦增加钩码的个数,使钩码的质量增大为 m2,达到平衡后,记下空气柱的体积 V2。 得 分 (2)求出计算大气压强 p0 的公式。(用已给的和测得的物理量表示) 得 分 Ⅱ.(20 分)制取氨气并完成喷泉实验(图中夹持装置均已略去)。 (1)写出实验室制取氨气的化学方程式: ____________________________________________________________________。 得 分 (2)收集氨气应使用______________-法,要得到干燥的氮气可选用_______________________ 做干燥剂。 得 分 (3)用图 1 装置进行喷泉实验,上部烧瓶已装满干燥氨气,引发水上喷泉的操作是 _________________________________________________________________________________。 该实验的原理是____________________________________________________________________ 得分 评卷人 __________________________________________________________________________________ __________________________________________________________________________________ _。 得 分 (4)如果只提供如图 2 的装置,请说明引发喷泉的方法。 答: 30.(27 分)有三根长度皆为 l=1.00m 的不可伸长的绝缘轻线,其中两根的 一 端固定在天花板上的 O 点,另一端分别拴有质量皆为 m=1.00×10-2kg 的 带电小球 A 和 B,它们的电量分别为-q 和+q,q=1.00×10-7C。A、B 之 间用第三根线连接起来。空间中存在大小为 E=1.00×106N/C 的匀强电场,场强方向沿水平方向 右,平衡时 A、B 球的位置如图所示。现将 O、B 之间的线烧断,由于有空气阻力,A、B 球最 后会达到新的平衡位置。求最后两球的机械能与电势能总和与烧断前相比改变了多少。(不计两 带电小球间相互作用的静电力) 得 分 2003 年普通高等学校招生全国统一考试(全国卷) 理科综合能力测试 得分 评卷人 第 I 卷(选择题 共 22 题 每题 6 分 共 132 分) 在下列各题的四个选项中,只有一个选项是符合题目要求的。 以下数据可供解题时参考: 原子量:H 1 Li 7 Be 9 C 12 O 16 Na 23 Mg 24 P 31 Cl 35.5 K 39 Ca 40 1.取适量干重相等的 4 份种子进行不同处理:(甲)风干,(乙)消毒后浸水萌发,(丙) 浸水后萌发,(丁)浸水萌发后煮熟冷却、消毒。然后分别放入 4 个保温瓶中。一段时间后,种 子堆内温度最高的是: A 甲 B 乙 C 丙 D 丁 2.植物叶片从幼到老的整个生命活动过程中 A 有机物输出也输入,矿质元素只输入 B 有机物只输出,矿质元素只输入 C 有机物只输出,矿质元素输入也输出 D 有机物与矿质元素都既输入,又输出 3.下列关于叶绿素合成和功能的叙述,错误的是 A 光是叶绿素合成的必要条件 B 低温抑制叶绿素的合成 C 矿质元素影响叶绿素的合成 D 提取的叶绿素溶液,给予适宜的温度、光照和 CO2,可进行光合作用 4.一只成年雄狗仍然保持幼年的体态,且精神萎靡、反应迟钝、行动呆笨,无求偶行为, 其原因是 A 睾丸发育不全 B 甲状腺功能低下 C 生长激素分泌不足 D 生长激素分泌不足、睾丸发育不全 5.据图判断,下列叙述不符合 生 态 学原理的是 A 物质经过多级利用,实现了 良 性 循环 B 每一级生产环节都获得产 品,提 高了生态经济效益 C 由于食物链延长,能量逐级 损耗, 系统总能量利用效率降低 D 由于各级产物都可以利用,减少了废物和污染 6.人类探测月球发现,在月球的土壤中含有较丰富的质量数为 3 的氦,它可以作为未来核 聚变的重要原料之一。氦的该种同位素应表示为 A 4 3 He B 3 2 He C 4 2 He D 3 3 e 7.在两个容积相同的容器中,一个盛有 HCl 气体,另一个盛有 H2 和 Cl2 的混合气体。在 同温同压下,两容器内的气体一定具有相同的 A 原子数 B 密度 C 质量 D 质子数 8.某无色混合气体可能由 CH4、NH3、H2、CO、CO2 和 HCl 中的某几种气体组成。在恒 温恒压条件下,将此混合气体通过浓 H2SO4 时,总体积基本不变;通过过量的澄清石灰水,未 见变浑浊,但混合气体的总体积减小,把剩余气体导出后,在 O2 中能够点燃,燃烧产物不能使 CuSO4 粉末变色。则原混合气体的成份是 A HCl 和 CO B HCl、H2 和 CO C CH4 和 NH3 D HCl、CO 和 CO2 9.将 20mL 0.4mol/L 硝酸铵溶液跟 50mL 0.1mol/L 氢氧化钡溶液混合,则混合溶液中各离 子浓度的大小顺序是 A [NO3 -]>[OH-]>[NH4 +]>[Ba2+] B [NO3 -]>[Ba2+]>[OH-]>[NH4 +] C [Ba2+]>[NO3 -]>[OH-]>[NH4 +] D [NO3 -]>[Ba2+]>[NH4 +]>[OH-] 10.下列反应的离子方程式正确的是 A 铝片跟氢氧化钠溶液反应:Al+2OH-=AlO2 -+H2↑ B 硫酸镁溶液跟氢氧化钡溶液反应:SO42-+Ba2+=BaSO4↓ C 碳酸钙跟醋酸反应:CaCO3+2CH3COOH=Ca2++2CH3COO-+H2O+CO2↑ D 铜片跟稀硝酸反应:Cu+NO3 -+4H+=Cu2++NO↑+2H2O 11.某温度下,在一容积可变的容器中,反应 2A(g)+B(g) 2C(g)达到平衡时,A、B 和 C 的物质的量分别为 4mol、2mol 和 4mol。保持温度和压强不变,对平衡混合物中三者的物 质的量做如下调整,可使平衡右移的是 A 均减半 B 均加倍 C 均增加 1mol D 均减少 1mol 12.某温度下向 100g 澄清的饱和石灰水中加入 5.6g 生石灰,充分反应后恢复到原来的温 度。下列叙述正确的是 A 沉淀物的质量为 5.6g B 沉淀物的质量为 7.4g C 饱和石灰水的质量大于 98.2g D 饱和石灰水的质量小于 98.2g 13.用 0.01mol/L NaOH 溶液完全中和 pH=3 的下列溶液各 100mL。需 NaOH 溶液体积最 大的是 A 盐酸 B 硫酸 C 高氯酸 D 醋酸 14.根据中学化学教材所附元素周期表判断,下列叙述不正确的是 A K 层电子为奇数的所有元素所在族的序数与该元素原子的 K 层电子数相等 B L 层电子为奇数的所有元素所在族的序数与该元素原子的 L 层电子数相等 C L 层电子为偶数的所有主族元素所在族的序数与该元素 原子的 L 层电子数相等 D M 层电子为奇数的所有主族元素所在族的序数与该元 素原子 的 M 层电子数相等 15.如图所示,三个完全相同的金属小球 a、b、c 位于等边 三角形 的三个顶点上。a 和 c 带正电,b 带负电,a 所带电量的大小比 b 的小。 已知 c 受到 a 和 b 的静电力的合力可用图中四条有向线段中的一 条来表 示,它应是 A F1 B F2 C F3 D F4 16.下面列出的是一些核反应方程 30 15 P→ 30 14 Si+X 9 4 Be+ 2 1 H→ 10 5 B+Y 4 2 He+ 4 2 He→ 7 3 Li+Z 其中 A X 是质子,Y 是中子,Z 是正电子 B X 是正电子,Y 是质子,Z 是中子 C X 是中子,Y 是正电子,Z 是质子 D X 是正电子,Y 是中子,Z 是质子 17.一束单色光从空气射入玻璃中,则其 A.频率不变,波长变长 B 频率变大,彼长不变 C 频率不变,波长变短 D 频率变小,波长不变 18.简谐机械波在给定的煤质中传播时,下列说法中正确的是 A 振幅越大,则波传播的速度越快 B 振幅超大,则波传播的速度越慢 C 在一个周期内,振动质元走过的路程等于一个波长 D 振动的频率超高,则波传播一个波长的距离所用的时间越短 19.如图所示,一个半球形的碗放在桌面上,碗口水 平,O 点为其球心,碗的内表面及碗口是光滑的。一根细线跨在 碗 口 上,线的两端分别系有质量为 m1 和 m2 的小球,当它们处 于 平 衡状态时,质量为 m1 的小球与 O 点的连线与水平线的夹 角 为 α=60°。两小球的质量比 m2/m1 为 A 3 /3 B 2 /3 C 3 /2 D 2 /2 20.如图所示,固定容器及可动活塞 P 都是绝热的,中 间 有 一导热的固定隔板 B,B 的两边分别盛有气体甲和乙。现将活塞 P 缓慢地向 B 移动一段距离, 已知气体的温度随其内能的增加而升高,则在移动 P 的过程中, A 外力对乙做功;甲的内能不变 B 外力对乙做功;乙的内能不变 C 乙传递热量给甲;乙的内能增加 D 乙的内能增加;甲的内能不变 21.图中虚线所示为静电场中的等势面 1、2、3、4,相邻的 等 势 面之间的电势差相等,其中等势面 3 的电势为 0。一带正电的点电 荷 在 静电力的作用下运动,经过 a、b 点时的动能分别为 26eV 和 5eV。 当 这 一点电荷运动到某一位置,其电势能变为-8eV 时,它的动 能 应 为 A 8eV B 13eV C 20eV 34eV 22.K-介子衰变的方程为:K-→π-+π0,其中 K-介 子和π -介子带负的基元电荷,π0 介子不带电。一个 K-介子沿垂直 于磁场 的方向射入匀强磁场中,其轨迹为圆弧 AP,衰变后产生的π - 介 子 的轨迹为圆弧 PB,两轨迹在 P 点相切,它们的半径 RK-与 R π -之比 为 2︰1。π0 介子的轨迹未画出。由此可知π-的动量大小与 π 0 的 动量大小之比为 A 1︰1 B 1︰2 C 1︰3 D 1︰6 第Ⅱ卷(非选择题 共 12 题 共 168 分) 23.(15 分)用伏安法测量电阻阻值 R,并求出电阻率ρ。 给定电压表(内阻约为 50kΩ)、电流表(内阻约为 40Ω)、滑线变阻器、电源、电键、待 测电阻(约为 250Ω)及导线若干。 (1)画出测量 R 的电路图。 (2)图 1 中的 6 个点表示实验中测得的 6 组电流 I、电压 U 的值,试写出根据此图求 R 值 的步骤: 。求出的电阻值 R = 。(保留 3 位有效数字) (3)待测电阻是一均匀材料制成的圆柱体,用游标为 50 分度的卡尺测量其长度与直径, 结果分别如图 2、图 3 所示。由图可知其长度为 ,直径为 。 (4)由以上数据可求出ρ= 。(保留 3 位有效数字) 24.(15 分)中子星是恒星演化过程的一种可能结果,它的密度很大。现有一中子星,观 测到它的自转周期为 T=1/30s。向该中子星的最小密度应是多少才能维持该星体的稳定,不致 因自转而瓦解。计等时星体可视为均匀球体。(引力常数 G=6.67×10-11m3/kg·s2) 25.(20 分)曾经流行过一种向自行车车头灯供电的小型交流发电机,图 1 为其结构示意 图。图中 N、S 是一对固定的磁极,abcd 为固定在转轴上的矩形线框,转轴过 bc 边中点、与 ab 边平行,它的一端有一半径 r0=1.0cm 的摩擦小轮,小轮与自行车车轮的边缘相接触,如图 2 所示。当车轮转动时,因摩擦而带动小轮转动,从而使线框在磁极间转动。设线框由 N=800 匝导线圈组成,每匝线圈的面积 S=20cm2,磁极间的磁场可视作匀强磁场,磁感强度 B=0.010T, 自行车车轮的半径 R1=35cm,小齿轮的半径 R2=4.cm,大齿轮的半径 R3=10.0cm(见图 2)。 现从静止开始使大齿轮加速转动,问大齿轮的角速度为多大才能使发电机输出电压的有效值 U =3.2V?(假定摩擦小轮与自行车轮之间无相对滑动) 26.(14 分)麦品种是纯合体,生产上用种子繁殖,现要选育矮杆(aa)、抗病(BB)的小 麦新品种;马铃薯品种是杂合体(有一对基因杂合即可称为杂合体),生产上通常用块茎繁殖, 现要选育黄肉(Yy)、抗病(Rr)的马铃薯新品种。请分别设计小麦品种间杂交育种程序,以 及马铃薯品种间杂交育种程序。要求用遗传图解表示并加以简要说明。(写出包括亲本在内的前 三代即可)。 27.(10 分)将青蛙脑破坏,保留脊髓,在脊柱下部打开脊椎骨,剥离出脊髓一侧邻近的 两对脊神经根(一对脊神经根包含一个背根和一个腹根,见图)。分别电刺激每对脊神经根的背 根与腹根均可引起蛙同侧后肢发生运动反应。然后进行下实验: ①在第一对脊神经根的背根中央处剪断,电刺激背根向中段,蛙后肢发生运动反应;电刺 激背根外周段,蛙后肢不发生反应。 ②在第二对脊神经根腹根中央处剪 断, 电刺激腹根向中段,蛙后肢不发生反应: 电 刺激腹根外周段,蛙后肢发生运动反应。 试分析回答: (1)根据实验①判断背根的功能 是 。因为 (2)根据实验②判断腹根的功能 是 。因为 生物体内葡萄糖分解代谢过程的图解 如 下 脊髓横切面及其一侧的一对脊神经根示意图 28.(6 分)据上面的图解回答: (1)反应①②③④中可在人体细胞中进行的是: 。 (2)粮食贮藏进程中有时会发生粮堆湿度增大现象,这是因为 。 (3)在反应①②③④中,必须在有氧条件下进行的是: 。 29.(7 分) (1)1mol 丙酮(CH3 COOH)在镍催化剂作用下加 1mol 氢气转变成乳酸,乳酸的结构简 式是 。 (2)与乳酸具有相同官能团的乳酸的同分异构体 A 在酸性条件下,加热失水生成 B,由 A 生成 B 的化学反应方程式是 。 (3)B 的甲酯可以聚合,聚合物的结构简式是 。 30.(13 分)根据图示填空 (1)化合物 A 含有的官能团是 。 (2)1mol A 与 2mo H2 反应生成 1moE,其反应方程式是 。 (3)与 A 具有相同官能团的 A 的同分异构体的结构简式是 。 (4)B 在酸性条件下与 Br2 反应得到 D,D 的结构简式是 。 (5)F 的结构简式是 。由 E 生成 F 的反应类型是 。 31.(13 分)A、B、C 是短周期ⅠA 和ⅡA 族元素的碳酸盐,它们的质量分别为 mA、mB、 mC,与足量盐酸完全反应,消耗盐酸的物质的量分别为 nA(HCl)、nB(HCl)、nC(HCl)。 已知:。mA=mB+mC,nA(HCl)=nB(HCl)+ nC(HCl)。请填空: (1)写出短周期ⅠA 和ⅡA 族元素形成的所有碳酸盐的名称: (2)若以 MA、MB 和 MC 分别表示 A、B、C 的相对分子质量,试写出 MA、MB 和 MC 三 者的相互关系式 。 (3)A 的正确选择有 种,其化学式为: 。 (4)若 A 和 B 为ⅡA 族元素的碳酸盐,C 为ⅠA 族元素的碳酸盐,则 A、B、C 的化学式 依次是 ,mB︰mC=1︰ 。(保留 2 位小数) 32.(11 分)X、Y、Z 是短周期元素的三种常见氧化物。X 跟水反应后可生成一种具有还 原性的不稳定的二元酸,该酸的化学式是 ;Y 和 X 的组成元素相同,Y 的化学式 是 ;1mol Z 在加热时跟水反应的产物需要用 6mol 的氢氧化钠才能完全中和,Z 的化学 式是 ,其中和产物的化学式是 。在一定条件下,Y 可以跟非金属 单质 A 反应生成 X 和 Z,其反应的化学方程式是 。 33.(22 分)用下面两种方法可以制得白色的 Fe(OH)2 沉淀。 方法一:用不含 Fe3+的 FeSO4 溶液与用不含 O2 的蒸馏水配制的 NaOH 溶液反应制备。 (1)用硫酸亚铁晶体配制上述 FeSO4 溶液时还需加入 。 (2)除去蒸馏水中溶解的 O2 常采用 的方法。 (3)生成白色 Fe(OH)2 沉淀的操作是用长滴管吸取不含 O2 的 NaOH 溶液,插入 FeSO4 溶 液液面下,再挤出 NaOH 溶液。这样操作的理由是 。 方法二:在如图装置中,用 NaOH 溶液、铁屑、稀 H2SO4 等试 剂制备。 (1)在试管Ⅰ里加入的试剂是 。 (2)在试管Ⅱ里加入的试剂是 。 (3)为了制得白色 Fe(OH)2 沉淀,在试管Ⅰ和Ⅱ中加入试剂, 打 开 止 水夹,塞紧塞子后的实验步骤是 。 (4)这样生成的 Fe(OH)2 沉淀能较长时间保持白色,其理由 是 。 34.(22 分)一传送带装置示意如图,其中传送带经过 AB 区域 时 是 水 平的,经过 BC 区域时变为圆弧形(圆弧由光滑模板形成,未画出), 经 过 CD 区域时是倾斜的,AB 和 CD 都与 BC 相切。现将大量的质量均 为 m 的 小货箱一个一个在 A 处放到传送带上,放置时初速为零,经传送带 运 送 到 D 处,D 和 A 的高度差为 h。稳定工作时传送带速度不变,CD 段上各箱等距排列,相邻两箱的 距离为 L。每个箱子在 A 处投放后, 在 到 达 B 之前已经相对于传送带静止,且 以 后 也不再滑动(忽略经 BC 段时的微小 滑 动)。已知在一段相当长的时间 T 内, 共 运 送小货箱的数目为 N。这装置由电动 机 带 动,传送带与轮子间无相对滑动,不 计 轮 轴处的摩擦。求电动机的平均抽出功 率 P 。 参考答案 Ⅰ卷包括 22 小题,每题 6 分,共 132 分。 1.C 2.D 3.D 4.B 5.C 6.B 7.A 8.A 9.B 10.C 11.C 12.D 13.D 14.C 15.B 16.D 17.C 18.D 19.A 20.C 21.C 22.C Ⅱ卷包括 12 个小题,共 168 分。 23.(15 分) (1) 或 (2)①作 U-I 直线,舍去左起第 2 点,其余 5 个点尽量靠近直线且均匀分布在直线两侧。 ②求该直线的斜 K,则 R=K。 229Ω(221~237Ω均为正确)。 (3)0.800cm 0.194cm (4)8.46×10-2Ω·m(8.16×10-2~8.76×10-2Ω·m 均为正确) 24.(15 分)参考解答: 考虑中子星赤道处一小块物质,只有当它受到的万有引力大于或等于它随星体一起旋转所 需的向心力时,中子星才不会瓦解。 设中子星的密度为ρ,质量为 M,半径为 R,自转角速度为ω,位于赤道处的小块物质质 量为 m,则有 GMm/R2=mω2R ω=2π/T M=4/3πρR3 由以上各式得 ρ=3π/GT2 代人数据解得 ρ=1.27×1014kg/m3 25.(20 分)参考解答: 当自行车车轮转动时,通过摩擦小轮使发电机的线框在匀强磁场内转动,线框中产生一正 弦交流电动势,其最大值 ε=ω0BSN 式中ω0 为线框转动的角速度,即摩擦小轮转动的角速度。 发电机两端电压的有效值 U= 2 /2εm 设自行车车轮转动的角速度为ω1,由于自行车车轮与摩擦小轮之间无相对滑动,有 R1ω1=R0ω0 小齿轮转动的角速度与自行车轮转动的角速度相同,也为ω1。设大齿轮转动的角速度为ω, 有 R3ω=R2ω1 由以上各式解得 ω=( 9 4 U/BSN)(R2r0/R3r1) 代入数据得 ω=3.2s-1 26.(14 分) 小麦 第一代 AABB×aabb 亲本杂交 ↓ 第二代 F1 AaBb 种植 F1 代,自交 ↓○× 第三代 F2 A_B_,A_bb,aaB_,aabb 种植 F2 代,选矮杆、抗病(aaB_), 继续自交,期望下代获得纯合体 (注:A_B_,A_bb,aaB_,aabb 表示 F2 出现的九种基因型和四种表现型。) 马铃薯 第一代 yyRr×Yyrr 亲本杂交 ↓ 第二代 YyRr,yyRr,YyRR,yyrr 种值,选黄肉,抗病(YyRr) 第三代 YyRr 用块茎繁殖 27.(10 分) (1)传入功能。电刺激背根向中段可以产生后肢运动反应,表明神经兴奋进入脊髓中枢并 通过传出神经引发肌肉收缩,因此是传入功能。电刺激背根外周段不发生反应,表明背根无传 出功能。 (2)传出功能。因为电刺激腹根向中段不发生反应,而电刺激腹根外周段可以产生后肢运 动反应,表明神经兴奋治传出神经引发肌肉收缩,因此是传出功能。 28.(6 分) (1)①、②、④ (2)呼吸作用产生水 (3)② 29.(7 分) (1) (2)HOCH2CH2COOH   ,H CH2=CHCOOH+H2O (3) 30.(13 分) (1)碳碳双键,醛基、羧基 (2)OHC-CH=CH-COOH+2H2   ,Ni HO-CH2-CH-CH2-COOH (3) (4) (5) 或 酯化反应 31.(13 分) (1)碳酸锂,碳酸钠,碳酸铍,碳酸镁 (2)mA/MA=mB/MB+mC/MC(或 MB<MA<MC;MC<MA<MB 以及其它合理答案) (3)2 种 Li2CO3 MgCO3 (4)MgCO3,BeCO3,Na2CO3 1.05 32.(11 分) H2SO3,SO3,P2O5 Na3PO4、H2O 2P+5SO3=5SO2+P2O5 33.(22 分) 方法一 (1)稀 H2SO4 铁屑 (2)煮沸 (3)避免生成的 Fe(OH)2 沉淀接触 O2 方法二 (1)稀 H2SO4 铁屑 (2)NaOH 溶液 (3)检验试管Ⅱ出口处排出的氢气的纯度。当排出的 H2 纯净时,再夹紧止水夹。 (4)试管Ⅰ中反应生成的 H2 充满了试管Ⅰ和试管Ⅱ,且外界空气不容易进入。 34.(22 分)参考解答: 以地面为参考系(下同),设传送带的运动速度为 v0,在水平段运输的过程中,小货箱先在 滑动摩擦力作用下做匀加速运动,设这段路程为 s,所用时间为 t,加速度为 a,则对小箱有 s=1/2at2 ① v0=at ② 在这段时间内,传送带运动的路程为 s0=v0t ③ 由以上可得 s0=2s ④ 用 f 表示小箱与传送带之间的滑动摩擦力,则传送带对小箱做功为 A=fs=1/2mv02 ⑤ 传送带克服小箱对它的摩擦力做功 A0=fs0=2·1/2mv02 ⑥ 两者之差就是克服摩擦力做功发出的热量 Q=1/2mv02 ⑦ 可见,在小箱加速运动过程中,小箱获得的动能与发热量相等。 T 时间内,电动机输出的功为 W= P T ⑧ 此功用于增加小箱的动能、势能以及克服摩擦力发热,即 W=1/2Nmv02+Nmgh+NQ ⑨ 已知相邻两小箱的距离为 L,所以 v0T=NL ⑩ 联立⑦⑧⑨⑩,得 P = T Nm [ 2 22 T LN +gh] 2003 年上海高考物理试题 一、选择题。(共 8 小题,每小题 5 分,共 40 分。每小题给出的四个答案中,至少有一个是正 确的。) 1.在核反应方程 (X)ONHe 17 8 14 7 4 2  的括弧中,X 所代表的粒子是 (A) A. H1 1 B. H2 1 C. e0 1 D. n1 0 2.关于机械波,下列说法正确的是 (ABC) A.在传播过程中能传递能量 B.频率由波源决定 C.能产生干涉、衍射现象 D.能在真空中传播 3.爱因斯坦由光电效应的实验规律,猜测光具有粒子性,从而提出光子说。从科学研究的方 法来说,这属于 (C) A.等效替代 B.控制变量 C.科学假说 D.数学归纳 4.一个质量为 0.3kg 的弹性小球,在光滑水平面上以 6m/s 的速度垂直撞到墙上,碰撞后小 球沿相反方向运动,反弹后的速度大小与碰撞前相同。则碰撞前后小球速度变化量的大小Δv 和碰撞过程中墙对小球做功的大小 W 为 (BC) A.Δv=0 B.Δv=12m/s C.W=0 D.W=10.8J 5.一负电荷仅受电场力作用,从电场中的 A 点运动到 B 点。在此过程中该电荷做初速度为 零的匀加速直线运动,则 A、B 两点电场强度 EA、EB 及该电荷在 A、B 两点的电势能εA、εB 之间的关系为 (AD) A.EA=EB B.EAεB 6.粗细均匀的电阻丝围成的正方形线框置于有界匀强磁场中,磁场方向垂直于线框平面,其 边界与正方形线框的边平行。现使线框以同样大小的速度沿四个不同方向平移出磁场,如图所 示,则在移出过程中线框一边 a、b 两点间的电势差绝对值最大的是 (B) A. B. C. D. 7.质量不计的直角形支架两端分别连接质量为 m 和 2m 的小球 A 和 B。支架的两直角边长度 分别为 2l 和 l,支架可绕固定轴 O 在竖直平面内无摩擦转动,如图所示。开始时 OA 边处于水 平位置,由静止释放,则 (BCD) A.A 球的最大速度为 2 gl2 B.A 球的速度最大时,两小球的总重力势能最小 C.A 球的速度最大时,两直角边与竖直方向的夹角为 45° D.A、B 两球的最大速度之比 v1∶v2=2∶1 8.劈尖干涉是一种薄膜干涉,其装置如图 1 所示。将一块平 板玻璃放置在另一平板玻璃之上,在一端夹入两张纸片,从而 在两玻璃表面之间形成一个劈形空气薄膜。当光垂直入射后, 从上往下看到的干涉条纹如图 2 所示。干涉条纹有如下特点: ⑴任意一条明条纹或暗条纹所在位置下面的薄膜厚度相等;⑵ 任意相邻明条纹或暗条纹所对应的薄膜厚度差恒定。现若在图 1 装置中抽去一张纸片,则当光垂直入射到新的劈形空气薄膜后, 从 上 往 下 观 察 到 的 干 涉 条 纹 (A) A.变疏 B.变密 C.不变 D.消失 二、填空题。(每小题 4 分,共 12 分) 9.卢瑟福通过___________实验,发现了原子中间有一个很小的核, 并由此提出了原子的核式结构模型。右面平面示意图中的四条线表示 α粒子运动的可能轨迹,在图中完成中间两条α粒子的运动轨迹。 [α粒子散射,图略] 10.细绳的一端在外力 作用下从 t=0 时刻开始做 简谐运动,激发出一列简 谐横波。在细绳上选取 15 个点,图 1 为 t=0 时刻各点 所处的位置,图 2 为 t=T/4 时刻的波形图(T 为波的周 a b v a b v a b v a b v B A m 2m 2l l O 原子核 图 1 t=0 图 2 t=T/4 图 3 t=3T/4 两 张 纸 片 图 1(俯视图) 图 2 期)。在图 3 中画出 t=3T/4 时刻的波形图。 [图略,传到 10 号点,7 号点在最高点] 11.有质量的物体周围存在着引力场。万有引力和库仑力有类似的规律,因此我们可以用定 义静电场强度的方法来定义引力场的场强。由此可得,与质量为 M 的质点相距 r 处的引力场场 强的表达式为 EG=____________(万有引力恒量用 G 表示)。 [GM/r2] 12.若氢原子的核外电子绕核作半径为 r 的匀速圆周运动,则其角速度ω=__________;电子 绕核的运动可等效为环形电流,则电子运动的等效电流 I=__________。(已知电子的质量为 m, 电量为 e,静电力恒量用 k 表示) [ mr k m e mr k m e 2, 2 ] 13.某登山爱好者在攀登珠穆朗玛峰的过程中,发现他携带的手表表面玻璃发生了爆裂。这 种手表是密封的,出厂时给出的参数为:27℃时表内气体压强为 1×105Pa;在内外压强差超过 6×104Pa 时,手表表面玻璃可能爆裂。已知当时手表处的气温为-13℃,则手表表面玻璃爆裂时 表内气体压强的大小为__________Pa;已知外界大气压强随高度变化而变化,高度每上升 12m, 大气压强降低 133Pa。设海平面大气压为 1×105Pa,则登山运动员此时的海拔高度约为 _________m。 [8.7×104,6613(数值在 6550 到 6650 范围内均可)] 三、实验题。(共 30 分) 14.(5 分)如图所示,在研究平抛运动时,小球 A 沿轨道 滑下,离开轨道末端(末端水平)时撞开轻质接触式开关 S, 被电磁铁吸住的小球 B 同时自由下落。改变整个装置的高度做 同样的实验,发现位于同一高度的 A、B 两球总是同时落地。该 实验现象说明了 A 球在离开轨道后 (C) A.水平方向的分运动是匀速直线运动 B.水平方向的分运动是匀加速直线运动 C.竖直方向的分运动是自由落体运动 D.竖直方向的分运动是匀速直线运动 15.(5 分)在右图所示的光电管的实验中,发现用一定频率的 A 单色光照射光电管式,电 流表指针会发生偏转,而用另一频率的 B 单色光照射时不发生光电效 应,那么 (AC) A.A 光的频率大于 B 光的频率 B.B 光的频率大于 A 光的频率 C.用 A 光照射光电管时流过电流表 G 的电流方向是 a 流向 b D.用 A 光照射光电管时流过电流表 G 的电流方向是 b 流向 a 16.(6 分,力矩盘) 17.(7 分,玻意尔) 18.(7 分)图 1 为某一热敏电阻(电阻值随温度的改变而改变,且对温度很敏感)的 I-U 关系 曲线图。 ⑴为了通过测量得到图 1 所示 I-U 关系的完 整曲线,在图 2 和图 3 两个电路中应选择的是图 ________;简要说明理由:____________。(电源 电动势为 9V,内阻不计,滑线变阻器的阻值为 0-100Ω)。 [2;电压可从 0V 调到所需电压,调节范围较 大。] A B S H 光电管 电源 G a b I/mA U/V1 2 3 4 5 6 7 50 40 30 20 10 O 图 1V A V A A R1 R2 热 敏 电 阻 9V ⑵在图 4 电路中,电源电压恒为 9V,电流表读数为 70mA,定值电阻 R1=250Ω。由热敏电 阻的 I-U 关系曲线可知,热敏电阻两端的电压为________V;电阻 R2 的阻值为______Ω。 [5.2;111.8(111.6—112.0 均给分)] ⑶举出一个可以应用热敏电阻的例子:______________________________________。 [热敏温度计(提出其它实例,只要合理均给分)] 四、(60 分)计算题。 19.(10 分)(气态方程) 20.(10 分)如图所示,一高度为 h=0.2m 的水平面在 A 点处与一倾角为θ=30°的斜面连接, 一小球以 v0=5m/s 的速度在平面上向右运动。求小球从 A 点运动到地面所需的时间(平面与斜 面均光滑,取 g=10m/s2)。某同学对此题的解法为: 小球沿斜面运动,则 tgtvh   sin2 1 sin 0 ,由此可求得落地时间 t。 问:你同意上述解法吗?若同意,求出所需时间; 若不同意则说明理由并求出你认为正确的结果。 [不同意。小球应在 A 点离开平面做平抛运动,而 不是沿斜面下滑,正确做法为: 落地点与 A 点的水平距离 m12 00  g hvtvs ,而斜面底宽 l=hcotθ=0.35m,s>l,小球离 开 A 点后不会落到斜面,因此落地时间即为平抛运动时间, g ht 2 =0.2s ] 21.(12 分)质量为 m 的飞机以水平速度 v0 飞离跑道后逐 渐上升,若飞机在此过程中水平速度保持不变,同时受到重力 和竖直向上的恒定升力(该升力由其它力的合力提供,不含升 力)。今测得当飞机在水平方向的位移为 l 时,它的上升高度为 h。求:⑴飞机受到的升力大小;⑵从起飞到上升至 h 高度的 过程中升力所做的功及在高度 h 处飞机的动能。 [⑴飞机水平速度不变 l=v0t,y 方向加速度恒定 h=at2/2, 消去 t 即得 a=2hv02/l2,由牛顿第二定律:F=mg+ma=mg(1+2hv02/gl2) ⑵升力做功 W=Fh= mgh(1+2hv02/gl2),在 h 处 vt=at=2hv0/l,故        2 2 2 0 412 1 l hmvEK ] 22.(14 分)如图所示,OACO 为置于水平面内的光滑闭合金属导轨,O、C 处分别接有短 电阻丝(图中用粗线表示),R1=4Ω、R2=8Ω(导轨 其 它 部 分 电 阻 不 计 )。 导 轨 OAC 的 形 状 满 足      xy 3sin2  (单位:m)。磁感应强度 B=0.2T 的匀 强磁场方向垂直于导轨平面。一足够长的金属棒在水 平外力 F 作用下,以恒定的速率 v=5.0m/s 水平向右 在导轨上从 O 点滑动到 C 点,棒与导轨接触良好且 始终保持与 OC 导轨垂直,不计棒的电阻。求:⑴外力 F 的最大值;⑵金属棒在导轨上运动时 电阻丝 R1 上消耗的最大功率;⑶在滑动过程中通过金属棒的电流 I 与时间 t 的关系。 [⑴金属棒匀速运动,F 外=F 安,E=BLv,I=E/R 总,F 外=BIL=B2L2v/R 总,Lmax=2sin90°=2m, Ah v0 θ y x l h o y x R1 R2 A o C v R 总=8/3Ω,故 Fmax=0.3N ⑵P1=E2/R1=1W ⑶金属棒与导轨接触点间的长度随时间变化      xy 3sin2  ,且 x=vt,E=BLv, 故      tR EI 3 5sin4 3  总 ] 23.(14 分)为研究静电除尘,有人设计了一个盒状容器,容器侧面是绝缘的透明有机玻璃, 它的上下底面是面积 A=0.04m2 的金属板,间距 L=0.05m,当 连接到 U=2500V 的高压电源正负两极时,能在两金属板间产 生一个匀强电场,如图所示。现把一定量均匀分布的烟尘颗 粒密闭在容器内,每立方米有烟尘颗粒 1013 个,假设这些颗 粒都处于静止状态,每个颗粒带电量为 q=+1.0×10-17C,质量 为 m=2.0×10-15kg,不考虑烟尘颗粒之间的相互作用和空气 阻力,并忽略烟尘颗粒所受重力。求合上电键后:⑴经过多 长时间烟尘颗粒可以被全部吸附?⑵除尘过程中电场对烟尘 颗粒共做了多少功?⑶经过多长时间容器中烟尘颗粒的总动 能达到最大? [⑴当最靠近上表面的烟尘颗粒被吸附到下板时,烟尘就被全部吸附。烟尘颗粒受到的电场 力 F=qU/L,L=at2/2=qUt2/2mL,故 t=0.02s ⑵W=NALqU/2=2.5×10-4J ⑶设烟尘颗粒下落距离为 x,则当时所有烟尘颗粒的总动能 EK=NA(L-x) mv2/2= NA(L-x) qUx/L,当 x=L/2 时 EK 达最大,而 x=at12/2,故 t1=0.014s ] 2004 年普通高等学校招生全国统一考试(湖南、湖北卷) 理科综合能力测试 本试卷分第Ⅰ卷(选择题)和第Ⅱ卷(非选择题)两部分,第Ⅰ卷 1 至 4 页,第Ⅰ卷 5 至 10 页,满分 300 分,考试用时 150 分钟,考试结束后,将本试卷和答题卡一并交回。 第Ⅰ卷 注意事项: 1. 答卷前,考生务必将自己的姓名、准考证号写在试题卷和等题卡上、并将准考证号条形 码粘贴在答题卡上指定位置。 2. 每小题选出答案后,用 2B 铅笔把答题卡上对应题目的答案标号涂黑,如需改动,用橡 皮擦干净后,再选涂其它答案标号, 3. 本卷共 21 题,每题 6 分,共 126 分,在每题给出的四个选项中,只有一项是符合题目 要求的。 以下数据可提供解题时参考: 原子量:C 12 N 14 O 16 Na 23 Mg 24 P 31 Cl 35.5 K 39 Ca 40 Fe 56 1. 下列关于光合作用强度的叙述正确的是 A. 叶片从幼到老光合作用强度不变 B. 森林或农田中植株上部叶片和下部叶片光合作用强度有差异 C. 光合作用强度是由基因决定的,因此是固定不变的 D. 在相同光照条件下,各种植物的光合作用强度相同 U S 接地 - + L 2. 某生物的体细胞染色体数为 2n。该生物减数分裂的第二次分裂与有丝分裂相同之处是 A. 分裂开始前,都进行染色体的复制 B. 分裂开始时,每个细胞中的染色体数都是 2n C. 分裂过程中,每条染色体的着丝点都分裂成两个 D. 分裂结束后,每个子细胞的染色体数都是 n 3. 用一定量的甲状腺激素连续饲喂正常成年小白鼠 4 周,与对照组比较,实验组小白鼠表现为 A. 耗氧量增加、神经系统的兴奋性降低 B. 耗氧量增加、神经系统的兴奋性增强 C. 耗氧量减少、神经系统的兴奋性降低 D. 耗氧量减少、神经系统的兴奋性增强 4. 下列属于生态系统食物网特征的是 A. 一种生物只能被另一种生物捕食 B. 食物链的环节数是无限的 C. 一种生物可能属于不同的营养级 D. 食物网上的生物之间都是捕食关系 5. 用动物细胞工程技术获取单克隆抗体,下列实验步骤中错误..的是 A. 将抗原注入小白鼠体内,获得能产生抗体的 B 淋巴细胞 B. 用纤维素酶处理 B 淋巴细胞与小白鼠骨髓瘤细胞 C. 用聚乙二醇作诱导剂,促使能产生抗体的 B 淋巴细胞与小白鼠骨髓瘤细胞融会 D. 筛选杂交瘤细胞,并从中选出能产生所需抗体的细胞群,培养后提取单克隆抗体 6. 在 pH=1 含 Ba2+离子的溶液中,还能大量存在的离子是 A. AlO  2 B. ClO— C. Cl— D. SO 2 4 7. 物质的量浓度相同的下列溶液中,符合按 pH 由小到大顺序排列的是 A. Na2CO3 NaHCO3 NaCl NH4Cl B. Na2CO3 NaHCO NH4Cl NaCl C. (NH4)2SO4 NH4Cl NaNO3 Na2S D. NH4Cl (NH4)2SO4 Na2S NaNO3 8. 已知 ⑴ H2(g)+ 2 1 O2(g)==H2O(g) △H1= a kJ·mol-1 ⑵ 2H2(g)+ O2(g)==2H2O(g) △H2= b kJ·mol-1 ⑶ H2(g)+ 2 1 O2(g)==H2O(l) △H3= c kJ·mol-1 ⑷ 2H2(g)+ O2(g)==2H2O(l) △H4= d kJ·mol-1 下列关系式正确的是 A. ad>0 C. 2a=b<0 D. 2c=d>0 9. 将 0.1 mol·L-1 醋酸溶液加水稀释,下列说法正确的是体 A. 溶液中 c(H+)和 c(OH—)都减小 B. 溶液中 c(H+)增大 C. 醋酸电离平衡向左移动 D. 溶液的 PH 值增大 10. 下列叙述正确的是 A. 同温同压下,相同体积的物质,它们的物质的量必相等 B. 任何条件下,等物质的量的乙烯和一氧化碳所含分子数必相等 C. 1L 一氧化碳气体一定比 1L 氧气的质量小 D. 等体积、等物质的量浓度的强酸中所含的 H+数一定相同 11. 若 1 mol 某气态烃 CxHy,完全燃烧,需用 3 mol O2,则 a b xo A. x=2,y=2 B. x=2,y=4 C. x=3,y=6 D. x=3,y=8 12. 下列分子中,所有原子不可能...共处在同一平面上的是 A. C2H2 B. C2S C. NH3 D. C6H6 13. 常温下,下列各组物质不能用同一种试剂通过化学反应区别的是 A. MnO2 CuO FeO B. (NH4)2SO4 K2SO4 NH4Cl C. AgNO3 KNO3 Na2CO3 D. Na2CO3 NaHCO3 KNO3 14. 现有 1200 个氢原子被激发到量子数为 4 的能级上,若这些受激氢原子最后都回到基态,则 在此过程中发出的光子总数是多少?假定处在量子数为 n 的激发态的氢原子跃迁到各较低 能级的原子数都是处在该激发态能级上的原子总数的 1 1 n 。 A. 2200 B. 2000 C. 1200 D. 2400 15. 下面是四种与光有关的事实: ① 用光导纤维传播信号 ② 用透明的标准样板和单色光检查平面的平整度 ③ 一束白光通过三棱镜形成彩色光带 ④ 水面上的油膜呈现彩色 其中,与光的干涉有关的是 A. ①④ B. ②④ C. ①③ D. ②③ 16. 一定量的气体吸收热量,体积膨胀并对外做功,则此过程的末态与初态相比 A. 气体内能一定增加 B. 气体内能一定减小 C. 气体内能一定不变 D. 气体内能是增是减不能确定 17. 如图,一简谐横波在 x 轴上传播,轴上 a、b 两点相距 12m。t=0 时 a 点为波峰,b 点为波谷; t=0.5s 时,a 点为波谷,b 点为波峰。则下列判断正确的是 A. 波一定沿 x 轴正方向传播 B. 波长可能是 8m C. 周期可能是 0.5s D. 波速一定是 24m/s 18. 如图所示,四个完全相同的弹簧都处于水平位置,它们的右端受到大小皆为 F 的拉力作用, 而左端的情况各不相同:①中弹簧的左端固定在墙上,②中弹簧的左端受大小也为 F 的拉 力作用,③中弹簧的左端拴一小物块,物块在光滑的桌面上滑动,④中弹簧的左端拴一小 物块,物块在有摩擦的桌面上滑动。若认为弹簧的质量都为零,以 l1、l2、l3、l4 依次表示 四个弹簧的伸长量,则有 ① FF ② F ③ ④F F A. l2>l1 B. l4>l3 C. l1>l3 D. l2=l4 19. 一直升飞机停在南半球的地磁极上空。该处地磁场的方向竖直向上,磁感应强度为 B。直升 飞机螺旋桨叶片的长度为 l,螺旋桨转动的频率为 f,顺着地磁场的方向看螺旋桨,螺旋桨按 顺时针方向转动。螺旋桨叶片的近轴端为 a,远轴端为 b,如图所示。忽略 a 到转轴中心线 的距离,用ε表示每个叶片中的感应电动势,则 A. ε=πfl2B,且 a 点电势低于 b 点电势 B. ε=2πfl2B,且 a 点电势低于 b 点电势 C. ε=πfl2B,且 a 点电势高于 b 点电势 D. ε=2πfl2B,且 a 点电势高于 b 点电势 20. 如图一绝缘细杆的两端各固定着一个小球,两小球带有等量异号的电荷,处于匀强电场中, 电场方向如图中箭头所示。开始时,细杆与电场方向垂直,即 在图中Ⅰ所示的位置;接着使细杆绕其中心转过 90°,到达图 中Ⅱ所示的位置;最后使细杆移到图中Ⅲ所示的位置。以 W1 表示细杆由位置Ⅰ到位置Ⅱ过程中电场力对两小球所做的功, W2 表示细杆由位置Ⅱ到位置Ⅲ过程中电场力对两小球所做的 功,则有 A. W1=0,W2≠0 B. W1=0,W2≠0 C. W1≠0,W2=0 D. W1≠0,W2≠0 21. 放在水平地面上的一物块,受到方向不变的水平推力 F 的 作用,F 的大小与时间 t 的关系和物块速度 v 与时间 t 的关 系如图所示。取重力加速度 g=10m/s2。由此两图线可以求 得物块的质量 m 和物块与地面之间的动摩擦因数μ分别为 A. m=0.5kg,μ=0.4 B. m=1.5kg,μ= 15 2 C. m=0.5kg,μ=0.2 D. m=1kg,μ=0.2 2004 年普通高等学校招生全国统一考试(湖南、湖北卷) 理科综合能力测试 第Ⅱ卷 (非选择题 共 10 题 共 174 分) 注意事项: 1. 用钢笔或圆珠笔直接答在试题卷中(除题目有特殊规定外)。 2. 答卷前将密封线内的项目填写清楚。 题号 22 23 24 25 26 27 28 29 30 31 32 总分 分数 22.(18 分)用以下器材测量一待测电阻 Rx 的阻值(900~1000Ω): 电源 E,具有一定内阻,电动势约为 9.0V; 电压表 V1,量程为 1.5V,内阻 r1=750Ω; 电压表 V2,量程为 5V,内阻 r2=2500Ω; 滑线变阻器 R,最大阻值约为 100Ω; 单刀单掷开关 K,导线若干。 ⑴测量中要求电压表的读数不小于其量程的 3 1 ,试画 出测量电阻 Rx 的一种实验电路原理图(原理图中的元件要 用题图中相应的英文字母标注)。 ⑵根据你所画的电路原理图在题给的实物图上画出 联线。 ⑶若电压表 V1 的读数用 U1 表示,电压表 V2 的读数用 U2 表示,则由已知量和测得量表示 Rx 的公式为 Rx=________。 23.(16 分)一水平放置的水管,距地面高 h=1.8m,管内截面积 S=2.0cm2。有水从管口以不变 的速度 v=2.0m/s 源源不断地沿水平方向射出,设水流稳定后在空中有多少立方米的水。 24.(18 分) 如图所示,在 y>0 的空间中存在匀强电场,场强沿 y 轴负 方向;在 y<0 的空间中,存在匀强磁场,磁场方向垂直 xy 平面 (纸面)向外。一电量为 q、质量为 m 的带正电的运动粒子,经 过 y 轴上 y=h 处的点 P1 时速率为 v0,方向沿 x 轴正方向;然后, 经过 x 轴上 y=2h 处的 P2 点进入磁场,并经过 y 轴上 y=-2h 处的 P3 点。不计重力。求 ⑴电场强度的大小。 ⑵粒子到达 P2 时速度的大小和方向。 ⑶磁感应强度的大小。 25.(20 分) 柴油打桩机的重锤由气缸、活塞等若干部件组成,气缸与活塞 间有柴油与空气的混合物。在重锤与桩碰撞的过程中,通过压缩使 混合物燃烧,产生高温高压气体,从而使桩向下运动,锤向上运动。 现把柴油打桩机和打桩过程简化如下: 柴油打桩机重锤的质量为 m,锤在桩帽以上高度为 h 处(如图 1)从静止开始沿竖直轨道 自由落下,打在质量为 M(包括桩帽)的钢筋混凝土桩子上。同时,柴油燃烧,产生猛烈的推 力,锤和桩分离,这一过程的时间极短。随后,桩在泥土中向下移动一距离 l。已知锤反跳后到 达最高点时,锤与已停下的桩帽之间的距离也为 h(如图 2)。已知 m=1.0×103kg,M=2.0×103kg, h=2.0m,l=0.20m,重力加速度 g=10m/s2,混合物的质量不计。设桩向下移动的过程泥土对桩的 作用力 F 是恒力,求此力的大小。 26.(16 分) 粉末状试样 A 是由等物质的量的 MgO 和 Fe2O3 组成的混合物。进行如下实验: ①取适量 A 进行铝热反应,产物中有单质 B 生成; ②另取 20g A 全部溶于 0.15L 6.0mol·L-1 盐酸中,得溶液 C; ③将①中得到的单质 B 和溶液 C 反应,放出 1.12L(标况)气体,同时生成溶液 D,还残 留有固体物质 B; ④用 KSCN 溶液检验时,溶液 D 不变色。 请填空: ⑴①中引发铝热反应的实验操作是______________,产物中的单质 B 是_____________。 ⑵②中所发生的各反应的化学方程式是_______________________。 ⑶③中所发生的各反应的离子方程式是_______________________。 ⑷若溶液 D 的体积仍视为 0.15L,则该溶液中 c(Mg2+)为____________。 c(Fe2+)为____________。 27.(14 分) 科学家发现某药物 M 能治疗心血管疾病是因为它在人体内能释放出一种“信使分子”D, 并阐明了 D 在人体内的作用原理,为此他们荣获了 1998 年诺贝尔生理学或医学奖。 请回答下列问题: ⑴已知 M 的分子量为 227,由 C、H、O、N 四种元素组成,C、H、N 的质量分数依次为 15.86%、2.20%和 18.50%。则 M 的分子式是__________________。D 是双原子分子,分子量为 30,则 D 的分子式为_________________。 ⑵油脂 A 经下列途径可得到 M。 反应①的化学方程式是_________________________。 反应②的化学方程式是_________________________。 ⑶C 是 B 和乙酸在一定条件下反应生成是化合物,分子量为 134,写出 C 所有可能的结构 简式______________________________________________________________________。 ⑷若将 0.1mol B 与足量的金属反应,则需消耗______________g 金属钠。 28.(14 分) 在玻璃圆筒盛有两种无色的互不相溶的中性液体。上层液体插入两 根石墨电极,圆筒内还放有一根下端弯成环状的玻璃搅棒,可以上下搅 动液体,装置如右图。接通电源,阳极周围的液体呈现棕色,且颜色由 浅变深,阴极上有气泡生成。停止通电,取出电极,用搅棒上下剧烈搅 动。静置后液体又分成两层,下层液体呈紫红色,上层液体几乎无色。 根据上述实验回答: ⑴阳极上的电极反应式为__________________________________。 ⑵阴极上的电极反应式为__________________________________。 ⑶原上层液体是__________________________________。 ⑷原下层液体是__________________________________。 ⑸ 搅 拌 后 两 层 液 体 颜 色 发 生 变 化 的 原 因 是 ______________________________________ __________________________________________________________________________。 ⑹要检验上层液体中含有的金属离子,其方法是________________________________, 现象是____________________________________________________________________。 29.(16 分) O || 抗击“非典”期间,过氧乙酸(CH3C——O——OH)是广为使用的消毒剂。它可由 H2O 和冰醋酸反应制取,所以在过氧乙酸中常含有残留的 H2O。测定产品中过氧乙酸浓度 c0 涉及下 列反应: ⑴ □ MnO  4 + □H2O2 +□ H+ === □ Mn2+ + □ O2 + □ H2O ⑵ H2O2 + 2I- + 2 H+ === I2 + 2 H2O O || ⑶ CH3C——O——OH + 2I- + 2 H+ ===CH3COOH + I2 + 2 H2O ⑷ I2 + 2S2O 2 3 ==== S4O 2 6 + 2I- 请回答以下问题: ⑴配平反应①的离子方程式(配平系数填入以下方框内): □ MnO  4 + □H2O2 +□ H+ === □ Mn2+ + □ O2 + □ H2O ⑵用 Na2S2O3 标准溶液滴定 I2 时(反应④)选用的指示剂是_____________________。 ⑶取 b0 mL 待测液,用硫酸使溶液酸化,在用浓度为 a1 mol·L-1 的 KMnO4 标准溶液滴定其 中的 H2O2,耗用的 KMnO4 体积为 b1 mL(反应①,滴定过程中 KMnO4 不与过氧乙酸反应) 另取 b0 mL 待测液,加入过量的 KI,并用硫酸使溶液酸化,此时过氧乙酸和残留的 H2O2 都能跟 KI 反应生成 I2(反应②和③)。再用浓度为 a2 mol·L-1 的 Na2S2O3 标准溶液滴定生成的 I2,耗用 Na2S2O3 溶液体积为 b2 mL。 请根据上述实验数据计算过氧乙酸的浓度(用含 a1、a2、b0、b1、b2 的代数示表示)。 c0=______________________。 ⑷为计算待测液中过氧乙酸的浓度 c0,加入的 KI 的质量已过量但没有准确称量,是否影响 测定结果_______________(填是或否)。 30.(22 分) 试回答下列⑴——⑵题。 ⑴在一些性状的遗传中,具有某种基因型的合子不能完成胚胎发育,导致后代中不存在该 基因型的个体,从而使性状的分离比例发生变化。小鼠毛色的遗传就是一个例子。 一个研究小组,经大量重复实验,在小鼠毛色的遗传的研究中发现: A. 黑色鼠与黑色鼠杂交,后代全部为黑色鼠。 B. 黄色鼠与黄色鼠杂交,后代中黄色鼠与黑色鼠发比例为 2:1 C. 黄色鼠与黄色鼠杂交,后代中黄色鼠与黑色鼠发比例为 1:1 根据上述实验结果,回答下列问题:(控制毛色的显性基因用 A 表示,隐性基因用 a 表示) ①黄色鼠的基因型是_____________,黑色鼠发基因型是_________________。 ②推测不能完成胚胎发育的合子的基因型是____________________。 ③写出上述 B、C 两个杂交组合的遗传图解。 ⑵回答下列问题: ①真核生物基因的编码区中能够编码蛋白质的序列称为__________,不能够编码蛋白质的 序列称为_____________。 ②一般来说,如果知道了某真核生物的一条多肽链的氨基酸序列,你能否确定其基因编码 区的 DNA 序列?为什么? 31.(22 分) 胰高血糖素对小白鼠和人具有相同的生理作用。为了验证“胰高血糖素具有升温血糖的生 理作用”,请以小白鼠为实验对象设计实验步骤,预测和解释实验应出现的结果,并写出实验结 论。 (一)实验材料和用具: 正常实验小白鼠 2 只,生理盐水,用生理盐水配制的适宜浓度的胰高血糖素溶液,班氏糖 定性试剂,注射器,试管,烧杯等。 (二)实验步骤: (实验提示:采用腹腔注射药,给药剂量不作实验设计要求;给药 1 小时后,用注射器在 小白鼠膀胱处穿刺取尿液。) (三)实验结果的预测、解释和结论: 参考答案 1—10 B C B C B C C C D B 11—21 B C D A B D B D A C A 22.(18 分)(1) (2) (1) Rx= 2112 211 rUrU rrU  或 Rx= 1 1 12 )( rU UU  23.(16 分) 以 t 表示水由喷口处到落地所用的时间,有 h= 2 1 gt2 ① 单位时间内喷出的水量为 Q=Sυ ② 空中水的总量应为 V=Qt ③ 由以上各式得 V=S·V· g h2 ④ 代入数值得 V=2.4×10-4 m3 ⑤ 24.(18 分) (1)粒子在电场、磁场中运动的轨迹如图所示。设粒子从 P1 到 P2 的时间为 t,电场度的大小为 E,粒子在电场中的加速度为 a,由牛顿第二定律及运动学公式有 qE=ma ① υ0t=2A ② 2 1 at2=h ③ 由①、②、③式解得 E= qh mp 2 2 0 ④ (2)粒子到达 P2 时速度沿 x 方向的分量仍为υ0,以υ1 表示速度沿 y 方向分量的大小,υ表示 速度的大小,θ表示速度和 x 轴的夹角,则有 υ 2 1 =2ah ⑤ υ= 2 0 2 1   ⑥ tgθ= 0 1   ⑦ 由②、③、⑤式得 υ1=υ0 ⑧ 由⑥、⑦、 ⑧式得 υ= 02 ⑨ θ=45° ⑩ (3)设磁场的磁感应强度为 B,在洛仑兹力作用下粒子做匀速圆周运动,由牛顿第二定律 qυB=m r 2 ○11 r 是圆周的半径。此圆周与 x 轴和 y 轴的交点分别为 P2、P3。因为 OP2=OP3,θ=45°,由几何 关系可知,连线 P2P3 为圆轨道的直径,由此可求得 r= h2 ○12 由⑨、○11 、○12 可得 B= qh m 0 ○13 25.(20 分) 锤自由下落,碰桩前速度υ1 向下, υ1= gh2 ① 碰后,已知锤上升高度为(h-l),故刚碰后向上的速度为 υ2= )(2 lhg  ② 设碰后桩的速度为 V,方向向下,由动量守恒, mυ1=MV-mυ2 ③ 桩下降的过程中,根据功能关系, 2 1 MV2+Mgl=Fl ④ 由①、②、③、④或得 F=Mg+ ])(22)[( lhhlhM m l mg  ⑤ 代入数值,得 F=2.1×105N ⑥ 26.(16 分)(每空 2 分,共 16 分) (1)加少量 KClO3,插上 Mg 条并将其点燃 Fe (2)Fe2O3+6HCl====2FeCl3+3H2O MgO+2HCl====MgCl2+H2O (3)Fe+2H+====Fe2++H2 Fe+2Fe3+====3Fe2+ (4)c(Mg2+)====0.67 mol ·L-1 c(Fe2+)====2.3 mol·L-1 27。(14 分) (1)C3H5O9N3 NO 28。(14 分) (1)2I――2e―=====I2 (2 ) 2H++2e―====H2 (3 )KI(或 NaI 等)水溶液 (4 )CCl4(或 CHCl3 等) (5 )I2 在 CCl4 中的溶解度大于在水中的溶解度,所以绝大部分 I2 都转移到 CCl4 中 (6 )焰色反应 透过蓝色钴玻璃观察火焰呈柴紫色(其它合理答案同样给分。例如,若③中 答 NaI 水溶液,这里答火焰呈黄色。) 29。(16 分) (1)2 5 6 2 5 8 (2 )淀粉溶液 (3 ) ob baba 2 5 2222  (4 )否 30。(22 分) (1) ①Aa aa ②AA ③B : A a × A a 黄色 黄色 IAA : 2Aa : Iaa 不存活 黄色 黑色 C: A a × aa 黄色 黑色 IAa : Iaa 黄色 黑色 (2) ①外显子、内含子 ②不能。 首先,一种氨基酸可以有多种密码子; 其次,一般地说真核生物的基因具有内 含子。 31。(20 分) (二)实验步骤: (1)确定 1 只鼠为实验鼠,腹腔注射胰高血糖素溶液;另一只鼠为对照鼠,腹腔注射等容量生 理盐水。 (2)将两支试管分别编号为 1 号和 2 号,各加入等量的班氏糖定性试剂。 (3)给药 1 小时后,对两只小白鼠采尿液,实验鼠尿液放入 1 号试管内,对照鼠尿液放入 2 号 试管内。 (4)将两支试管摇匀后,放入盛有开水的烧杯内加热煮沸,待冷却后,观察两支试管溶液颜色 的变化。 (三)实验结果的预测、解释和结论: 1 号试管中应该出现砖红色沉淀,表明实验鼠尿液中有葡萄糖;2 号试管中仍为蓝色溶液,表明 对照鼠尿液中无葡萄糖。 实验结论:实验鼠血糖升高,超过一定数值而出现糖尿,是胰高血糖素具有升高血糖的生理 作用所引起的。 2004 年普通高等学校招生全国考试统一考试 全国卷Ⅰ (物理部分) 第Ⅰ卷(选择题 共 126 分) 本卷共 21 题,每题 6 分,共 126 分。 14.本题中用大写字母代表原子核。E 经α衰变成为 F,再经β衰变成为 G,再经α衰变成为 H。 上述系列衰变可记为下式: 另一系列衰变如下: 已知 P 是 F 的同位素,则 A.Q 是 G 的同位素,R 是 H 的同位素 B.R 是 E 的同位素,S 是 F 的同位素 C.R 是 G 的同位素,S 是 H 的同位素 D.Q 是 E 的同位素,R 是 F 的同位素 15.如图所示,ad、bd、cd 是竖直面内三根固定的光滑细杆, a、b、c、 d 位于同一圆周上,a 点为圆周的最高点,d 点为最低点。 每 根 杆 上都套着一个小滑环(图中未画出),三个滑环分别从 a、 b、c 处 释放(初速为 0),用 t1、、、t2、、t3 依次表示各滑环到达 d 所 用 的 时间,则 A.t1 、t2、>t3 C.t3 > t1、>t2、 D.t1=、t2、=t3 16.若以μ表示水的摩尔质量,υ表示在标准状态下水蒸气 的 摩 尔 体积,ρ为在标准状态下水蒸气的密度,NA 为阿佛加德罗常数,m、Δ分别表示每个水分子 的质量和体积,下面是四个关系式: ① NA= υρ m ② ρ= μ NAΔ ③ m = μ NA ④ Δ= υ NA 其中 A. ①和②都是正确的 B.①和③都是正确的 C.③和④都是正确的 D. ①和④都是正确的 17.一列简谐横波沿 x 轴负方向传播,图 1 是 t =1s 时的波形图,图 2 是波中某振动质元位移随 时间变化的振动图线(两图用同一时间起点),则图 2 可能是图 1 中哪个质元的振动图线? A.x=0 处的质元 B.x=1m 处的质元 C.x=2m 处的质元 D. x=3m 处的质元 a b c d β α β β α α F E G H S R Q P t/s 0 1 2 3 4 5 6 y/m 图 2 x/m 0 1 2 3 4 5 6 y/m 图 1 18.图中电阻 R1、R2、R3 的阻值相等,电池的内阻不计。开关 K 接通后流过 R2 的电流是 K 接 通前的 A.1 2 B.2 3 C.1 3 D. 1 4 19.下表给出了一些金属材料的逸出功。 材料 铯 钙 镁 铍 钛 逸出功(10—19J) 3.0 4.3 5.9 6.2 6.6 现用波长为 400 nm 的单色光照射上述材料,能产生光电效应的材料最多有几种?(普朗克 常量 h=6.6×10-34J·s , 光速 c=3.0×10 8m/s) A.2 种 B.3 种 C.4 种 D.5 种 20.下列哪个说法是正确的? A.游泳运动员仰卧在水面静止不动时处于失重状态 B.蹦床运动员在空中上升和下落过程中都处于失重状态 C.举重运动员在举起杠铃后不动的那段时间内处于超重状态 D.体操运动员双手握住单杠吊在空中不动时处于失重状态 21.发出白光的细线光源 ab,长度为 l0 ,竖直放置,上端 a 恰好在水面以下,如图。现考虑线 光源 ab 发出的靠近水面法线(图中的虚线)的细光束 经 水 面 折 射 后所成的像,由于水对光有色散作用,若以 l1 表示红光 成 的 像 的 长 度,l2 表示蓝光成的像的长度,则 A. l1< l2< l0 B. l1> l2> l0 C. l2 > l1> l0 D. l2 < l1< l0 第Ⅱ卷(非选择题) 本卷共 10 题,共 174 分。 22.(18 分) ⑴图中给出的是用螺旋测微器测量一金属薄板厚度时的示数,此读数应为 mm ⑵实验室内有一电压表mV ,量程为 150mV 内阻约为 150Ω 。现要将其改装成量程为 10mA 的 电流表,并进行校准。为此,实验室提供如下器材:干电池 E(电动势为 1.5V ),电阻箱 R, 滑线变阻器 Rˊ,电流表 A (有 1.5mA , 15mA 与 150mA 三个量程)及开关 K。 (a)对电流表改装时必须知道电压表的内阻。可用图示的电路测量电压表mV 的内阻。在既不 R1 R2 R3 K mV Rˊ K A 0 5 15 10 a b 水 损坏仪器又能使精确度尽可能高的条件下,电路中的电流表 A 应选用的量程是 。 若合上 K,调节滑线变阻器后测得电压表的读数为 150mV,电流表 A 的读数为 1.05mA,则电 压表的内阻 RmV 为 。(取三位有效数字) (b)在对改装成的电流表进行校准时,把 A 作为标准电流表,画出对改装成的电流表进行校 准的电路原理图(滑线变阻器作限流使用),图中各元件要用题中给出的符号或字母标注。图中 电 阻 箱 的 取 值 是 ( 取 三 位 有 效 数 字 ), 电 流 表 A 应 选 的 量 程 是 。 23.(16 分)在勇气号火星探测器着陆的最后阶段,着陆器降落到火星表面上,再经过多次弹跳 才停下来。假设着陆器第一次落到火星表面弹起后,到达最高点时高度为 h ,速度方向是水平 的,速度大小为 v0 ,求它第二次落到火星表面时速度的大小,计算时不计火星大气阻力。已 知火星的一个卫星的圆轨道的半径为 r ,周期为 T, 火星可视 为半径为 r0 的均匀球体。 24.(18 分)图中 a1b1c1d1 和 a2b2c2d2 为在同一竖直平 面内的金 属导轨,处在磁感强度 B 的匀强磁场中,磁场方向垂 直导轨所 在平面(纸面)向里。导轨的 a1b1 段与 a2b2 段是竖直 的,距离 为 l1;c1d1 段与 c2d2 段也是竖直的,距离为 l2。x1y1 与 x2y2 为 两 根用不可伸长的绝缘轻线相连的金属细杆,质量分别 为 m1 、 m2,它们都垂直于导轨并与导轨保持光滑接触。两杆 与导轨构 成的回路的总电阻为 R。F 为作用与金属杆 x1y1 上竖 直向上的 恒力。已知两杆运动到图示位置时,已匀速向上运动, 求此时作 用于两杆的重力的功率的大小和回路电阻上的热功 率。 25.(20 分)一小圆盘静止在桌布上,位于 一 方桌的水平面的中央。桌布的一边与桌的 AB 边重合,如图。已知盘与桌布间的动摩擦因 数 为μ1 ,盘与桌面间的动摩擦因数为μ2 。 现 突然以恒定的加速度 a 将桌布抽离桌面,加 速 度的方向是水平的且垂直于 AB 边。若圆盘 最 后未从桌面掉下,则加速度 a 满足的条件是 什 么?(以 g 表示重力加速度) 参考答案 14.B、15.D、16.B、17.A、18.B、19.A、20.B、21.D 22.⑴6.124 ⑵(a)1.5mA 、143Ω (b)如图所示 、16.8Ω、15mA 23.答案: υ= 8π2h T2 r3 r02 +υ02 、 24.答案:P= F-(m1+m2)g B2(l2-l1)2 R(m1+m2)g Q=[F-(m1+m2)g B(l2-l1) ]2R 25.答案:a ≥ μ1+2μ2 μ2 μ1g A a B b1 a1 a2 x2 b2 c2 d2 y2 x1 c1 y1 d1 F A mV E K R/ R 2004 年普通高等学校招生全国统一考试 全国卷Ⅳ(理综) 本试卷分第Ⅰ卷(选择题)和第Ⅱ卷(非选择题)两部分。 注意事项: 1.答第Ⅰ卷前,考生务必将自己的姓名、准考证号、考试科目涂写在答题卡上。 2.每小题选出答案后,用铅笔把答题卡上对应题目的答案标号涂黑。如需改动,用橡皮擦 干净后,再选涂其他答案标号,不能答在试题卷上。 3.本卷共 21 小题,每小题 6 分,共 126 分。在每小题给出的四个选项中,只有一项是最 符合题目要求的。 以下数据可供解题时参考: 原子量:H—1 C—12 O—16 S—32 Fe—56 1.某种病毒已侵入人体细胞内,机体免疫系统对该靶细胞发挥的免疫作用是 ( ) A.体液免疫 B.细胞免疫 C.自身免疫 D.非特异性免疫 2.下列关于光合作用和呼吸作用的叙述,错误的是 ( ) A.光合作用和呼吸作用都包括一系列氧化还原反应 B.光合作用和呼吸作用必须在有水的条件下进行 C.光合作用的全部反应是呼吸作用全部反应的逆转 D.光合作用和呼吸作用都是能量转化过程 3.下列关于实验的描述,正确的是 ( ) A.将在蔗糖溶液中已发生质壁分离的洋葱表皮细胞转到更高浓度的蔗糖溶液中,则发生 质壁分离复原 B.将斐林试剂加入到蔗糖溶液中,加热后出现砖红色沉淀 C.将肝脏研磨液煮沸冷却后,加入到过氧化氢溶液中立即出现大量气泡 D.将双缩脲试剂加入到蛋清稀释液中,溶液变成紫色 4.肺炎双球菌中的 S 型具有多糖类荚膜,R 型则不具有。下列叙述错误..的是 ( ) A.培养 R 型活细菌时加 S 型细菌的多糖类物质,能产生一些具荚膜的细菌 B.培养 R 型活细菌时加 S 型细菌 DNA 的完全水解产物,不能产生具荚膜的细菌 C.培养 R 型活细菌时加 S 型细菌 DNA,能产生具荚膜的细菌 D.培养 R 型活细菌时加 S 型细菌的蛋白质,不能产生具荚膜的细菌 5.一个池塘有生产者(浮游植物)、初级消费者(植食性鱼类)、次级消费者(肉食性鱼类)和 分解者(微生物)。其中生产者固定的全部能量为 a,流入初级消费者、次级消费者和分解 者的能量依次为 b、c、d,下列表述正确的是 ( ) A.a=b+d B.a>b+d C.aQ2 17.图中 M 是竖直放置的平面镜,镜离地面的距离可调节。甲、乙二人站在镜前,乙离镜的距 离为甲离镜的距离的 2 倍,如图所示。二人略错开,以便甲能看到乙的像。以 l 表示镜的 长度,h 表示乙的身高,为使甲能看到镜中乙的全身像,l 的最小值为 ( ) A. h3 1 B. h2 1 C. h4 3 D.h 18.已知:一简谐横波在某一时刻的波形图如图 所示,图中位于 a、b 两处的质元经过四分之 一周期后分别运动到 a 、b 处。某人据此做 出如下判断:①可知波的周期,②可知波的 传播速度,③可知的波的传播方向,④可知 波的波长。其中正确的是( ) A.①和④ B.②和④ C.③和④ D.②和③ 19.如图,在倾角为 的固定光滑斜面上,有一用绳子拴着的长木板,木板上站着一只猫。 已知木板的质量是猫的质量的 2 倍。当绳子突然断开时,猫立即沿着板向上跑,以保持 其相对斜面的位置不变。则此时木板沿斜面下滑的加速度为 ( ) A. sin2 g B. sing C. sin2 3 g D.2 sing 20.如图所示,轻杆的一端有一个小球,另一端有光滑的固定轴 O。现给球一初速度,使球和 杆一起绕 O 轴在竖直面内转动,不计空气阻力,用 F 表 示球到达最高点时杆对小球的作用力,则 F( ) A.一定是拉力 B.一定是推力 C.一定等于 0 D.可能是拉力,可能是推力,也可能等于 0 21.一平行板电容器的电容为 C,两板间的距离为 d,上板带正电,电量为 Q,下板带负电,电 量也为 Q,它们产生的电场在很远处的电势为零。两个带异号电荷的小球用一绝缘刚性杆 相连,小球的电量都为 q,杆长为 l,且 lmp+mn D.mpF2。试求在两个物块运动过程中轻线的拉力 T。 24.(22 分) 空间中存在方向垂直于纸面向里的匀强磁场,磁感应强度为 B,一带电量为+q、质量为 m 的粒子,在 p 点以某一初速开始运动,初速方向在图中纸面内如图中 P 点箭头所示。该粒子运 动到图中 Q 点时速度方向与 P 点时速度方向垂直,如图中 Q 点箭头所示。已知 P、Q 间的距离 为 l。若保持粒子在 P 点时的速度不变,而将匀强磁场换成匀强电场,电场方向与纸面平行且与 粒子在 P 点时速度方向垂直,在此电场作用下粒子也由 P 点运动到 Q 点。不计重力。求: (1)电场强度的大小。 (2)两种情况中粒子由 P 运动到 Q 点所经历的时间之差。 25.(22 分) 如图所示,在一光滑的水平面上有两块相同的木板 B 和 C。重物(A 视质点)位于 B 的右 端,A、B、C 的质量相等。现 A 和 B 以同一速度滑向静止的 C,B 与 C 发生慧碰。碰后 B 和 C 粘在一起运动,A 在 C 上滑行,A 与 C 有摩擦力。已知 A 滑到 C 的右端面未掉下。试问:从 B、C 发生正碰到 A 刚移动到 C 右端期间,C 所走过的距离是 C 板长度的多少倍? 26.(18 分) 下面图表示制备无水盐 E 的主要步骤: 已知 B 是石灰石,D 盐含有约 49%的结晶水,无水盐 E 可用作干燥剂。取少量 E 溶于水, 向其中滴加硝酸后,再滴加硝酸银溶液,有白色沉淀 F 生成。 填写以下空白(写出化学式或名称): A ,C ,D , E ,F 。 27.(18 分) 芳香化合物 A、B 互为同分异构体,B 的结构筒式是 。A 经①、② 两步反应得 C、D 和 E。B 经①、②两步反应得 E、F 和 H。上述反应过程、产物性质及相互关 系如图所示。 (提示: (1)写出 E 的结构简式 。 (2)A 有 2 种可能的结构,写出相应的结构简式 。 (3)F 和小粒金属钠反应的化学方程式是 , 实验现象是 ,反应类型是 。 (4)写出 F 在浓 H2SO4 作用下在 170℃发生反应的化学方程式: 实验现象是 ,反应类型是 。 (5)写出 F 与 H 在加热和浓 H2SO4 催化作用下发生反应的化学方程式 , 实验现象是 ,反应类型是 。 (6)在 B、C、D、F、G、I 化合物中,互为同系物的是 。 28.(14 分) 根据下图及描述,回答下列问题: (1)关闭图 A 装置中的止水夹 a 后,从长颈漏斗向试管中注入一定量的水,静置后如图所示。 试判断:A 装置是否漏气?(填“漏气”、“不漏气”或“无法确定”) , 判断理由: 。 (2)关闭图 B 装置中的止水夹 a 后,开启活塞 b,水不断往下滴,直至企部流入烧瓶。试判断: B 装置是否漏气?(填“漏气”、“不漏气”或“无法确定”) , 判断理由: 。 29.(16 分) A、B、C、D、E 分别代表 5 种微粒,每种微粒中都含有 18 个电子,其中 A 和 C 都是由单 原子形成的阴离子,B、D 和 E 都是分子;又知在水溶液中 A 跟 B 反应可生成 C 和 D;E 具有强氧化性。请回答: (1)用化学符号表示上述 5 种微粒: A ,B ,C ,D ,E 。 (2)在水溶液中 A 跟 B 反应的离子方程式是: 。 30.(13 分) 右图示意的是人体内物质代谢与能量代谢的关系(部分)。 试按图中标号填写有关内容。 ① ② ③ ④ ⑤ ⑥ ⑦ ⑧ ⑨ ⑩ ○11 ○12 ○13 注:①⑤⑥⑦⑨表示过程, ②③④⑧⑩表示物质, ○11 表示能量, ○12 和○13 为高能化合物。 31.(17 分) 在一个透明的容器中加入适量 NaHCO3 稀溶液,将杨树叶片迅速封入其中,装置如图所示, 摇动容器,使容器内空气中的 CO2 和溶液中的 CO2 达到动态平衡,在保持温度不变的条件 下,进行如下实验,试根据实验回答下列问题: (1)光照几分钟后,容器内溶液的 pH (增大、减小),其原因是 。 (2)随着光照时间的延长,溶液 pH 的变化速度 趋于变(快、慢),其原因是 。 (3)若将装置置于暗室中,一段时间后,溶液的 pH (增大、减小),其原因是 。 (4)该装置可以用来研究植物的 和 。 2004 年普通高等学校招生全国统一考试 全国卷Ⅲ(理综) (老课程)答案 1—10 ABCDBCAADB 11—21 CCCCDDDBADB Ⅱ卷包括 10 小题,共 174 分。 22.(20 分) (1)54.14 (2)①将 K2 闭合,K2 断开,记下电压表读数 U1。 ②K1、K2 均闭合,记下电压表读数 U2。 结果:电源电动势 E=U1 内阻 r= RU UU 2 21  23.设两物块一起运动的加速度为 a,则有 F1-F2=(m1+m2)a ① 根据牛顿第二定律,对质量为 m1 的物块有 F1-T=m1a ② 由①、②两式得 21 1221 mm FmFmT   ③ 24.(22 分) (1)粒子在磁场中做匀速圆周运动,以 v0 表示粒子在 P 点的初速度,R 表示圆周的半径, 则有 qv0B=m R v 2 0 ① 由于粒子在 Q 点的速度垂直它在 p 点时的速度,可知粒子由 P 点到 Q 点的轨迹为 4 1 圆周, 故有 2 lR  ② 以 E 表示电场强度的大小,a 表示粒子在电场中加速度的大小,tE 表示粒子在电场中由 p 点运动到 Q 点经过的时间,则有 qE=ma ③ 2 2 1 EatR  ④ R=v0tE ⑤ 由以上各式,得 m qlBE 2 2 ⑥ (2)因粒子在磁场中由 P 点运动到 Q 点的轨迹为 4 1 圆周,故运动经历的时间 tE 为圆周运 动周期 T 的 4 1 ,即有 tE= 4 1 T ⑦ 而 0 2 v RT  ⑧ 由⑦⑧和①式得 qB mtE 2  ⑨ 由①⑤ 两式得 qB mtE  ⑩ qB mtt ER )12(   ○11 25.(22 分) 设 A、B、C 的质量均为 m。碰撞前,A 与 B 的共同速度为 v0,碰撞后 B 与 C 的共同速度 为 v1。对 B、C,由动量守恒定律得 mv0=2mv1 ① 设 A 滑至 C 的右端时,三者的共同速度为 v2。对 A、B、C,由动量守恒定律得 2mv0=3mv2 ② 设 A 与 C 的动摩擦因数为μ,从发生碰撞到 A 移至 C 的右端时 C 所走过的距离为 s,对 B、 C 由功能关系 2 1 2 2 )2(2 1)2(2 1 vmvmmgs  ③ 设 C 的长度为 l,对 A,由功能关系 2 2 2 0 2 1 2 1)( mvmvlsmg  ④ 由以上各式解得 3 7 l s ⑤ 26.(18 分) A. HCl(盐酸), C.Ca(OH)2(石灰乳) D.CaCl2·6H2O, E.CaCl2, F.AgCl。 27.(18 分) 28.(14 分) (1)不漏气 由于不漏气,加水后试管内气体体积减小,导致压强增大,长颈漏斗内的水面高出试管内 的水面。 (2)无法确定 由于分液漏斗和烧瓶间有橡皮管相连,使分液漏斗中液面上方和烧瓶中液面上方的压强相 同,无论装置是否漏气,都不影响分液漏斗中的液体滴入烧瓶。 29.(16 分) (1)S2-, HCl, Cl-, H2S, F2 (2)S2-+2H+=H2S↑ 30.(13 分) ①消化(酶分解) ②蛋白质 ③肝糖元 ④肌糖元 ⑤脱氨基 ⑥⑨有氧呼吸 ⑦无氧呼吸 ⑧尿素 ⑩脂肪 ○11 热能 ○12 ATP ○13 磷酸肌酸 31.(17 分) (1)增大、植物光合作用消耗 CO2 引起溶液中的碳酸氢根离子减少,使得氢离子浓度降低, 溶液 pH 增大。 (2)慢、随着光照时间的延长,光合作用使溶液中的碳酸氢根离子大量消耗,导致碳酸氢 根离子转化为 CO2 的量减少,氢离子浓度降低的速度减慢,溶液 pH 的变化速度转慢。 (3)减小,在暗室叶片进行呼吸作用,释放 CO2,容器中 CO2 浓度增加,使溶液中碳酸增 加,pH 减小。 (4)光合作用 呼吸作用。 2005 年普通高等学校招生全国统一考试(全国卷Ⅱ) 理科综合能力测试 选择题:共 21 小题,每小题 6 分,共 126 分。 以下数据可供解题时参考: 相对原子质量(原子量):H 1 C 12 N 14 O 16 Mg 24 Cu 64 一、选择题(本题包括 13 小题。每小题只有一个....选项符合题意) 1.糖尿病患者容易出现 A.细胞内液增多 B.组织液增多 C.体液增多 D.尿量增多 2.当抗原刺激体产生细胞免疫反映时,效应 T 细胞发挥的作用是 A.产生抗体使细胞裂解 B.激活靶细胞内的溶酶体酶使靶细胞裂解 C.产生组织胺增强 B 细胞的功能 D.促进 B 细胞产生淋巴因子 3.图中三条曲线分别代表了动物物种多样性程度、动物数 量易变程度 及冬眠动物比例在不同类型生态系统中的变化趋势。代表 动物物种多 样性程度和数量易变程度的曲线依次是 A.①② B.②③ C.①③ D.②① 4.当人处于炎热环境时,会引起 A.冷觉感受器兴奋 B.温觉感受器抑制 C.甲状腺激素分泌量增加 D.下丘脑体温调解中枢兴奋 5.下列实验中所用试剂错误..的是 A.在观察植物细胞有丝分裂实验中,使用醋酸洋红溶液使染色体着色 B.在提取叶绿体色素实验中,使用丙酮提取色素 C.在 DNA 的粗提取与鉴定实验中,使用氯化钠溶液析出 DNA D.在蛋白质的鉴定实验中,使用苏丹Ⅲ染液鉴定蛋白质 6.分析发现,某陨石中有半衰期极短的镁的一种放射性同位素 28 Mg,该同位素的原子核内的中 子数是 A.12 B.14 C.16 D.18 7.下列说法正确的是 A.常温常压下,只有一种元素的单质呈液态 B.周期表中所有元素都是从自然界中发现的 C.过渡元素不全是金属元素 D. 常温常压下,气态单质的分子都是由非金属元素的原子形成的 8.下列单质中,最容易跟氢气发生反映的是 A. O2 B. N2 C. F2 D. Cl2 9.NA 代表阿伏加德罗常数,下列说法正确的是 A.在同温同压时,相同体积的任何气体单质所含的原子数目相同 B.2g 氢气所原子数目为 NA C.在常温常压下,11.2L 氮气所含的原子数目为 NA D.17g 氮气所含电子数目为 10NA 10.相同体积的 pH=3 的强酸溶液和弱酸溶液分别跟足量的镁完全反应,下列说法正确的是 A.弱酸溶液产生较多的氢气 B.强酸溶液产生较多的氢气 C.两者产生等量的氢气 D.无法比较两者产生氢气的量 11.以知反应 A2(g)+ 2B2 2AB2(g)的△H<0,下列说法正确的是 A.升高温度,正向反应速率增加,逆向反应速率减小 B.升高温度有利于反应速率增加,从而缩短达到平衡的时间 C.达到平衡后,升高温度或增大压强都有利于该反应平衡正向移动 D.达到平衡后,降低温度或减小压强都有利于该反应平衡正向移动 12.某酒精厂由于管理不善,酒精滴漏到某种化学品上而酿成火灾。该化学品可能是 A.KMnO4 B.NaCl C. (NH4)2SO4 D. CH3COOH 13.等量的 CuO 和 MgO 粉末分别溶于相同体积的硝酸中,得到的 Cu(NO3)2 和 Mg(NO3)2 溶液的 浓度分别为 amol·L-1 和 b mol·L-1。则 a 与 b 的关系为 A.a=b B. a=2b C. 2a=b D. a=5b 二、选择题(本题包括 8 小题。每小题给出四个选项中,有的只有一个选项正确,有的有多个 选项正确,全部选对的得 6 分,选对但不全的得 3 分,有选错的得 0 分) 14.如图所示,位于光滑固定斜面上的小物块 P 受到水平向右 的推力 F 是 作用。已知物块 P 沿斜面加速下滑。现保持 F 的方向不变, 使其减小, 则加速度 A.一定变小 B.一定变大 C.一定不变 D.可能变小,可能变大,也可能不变 15.一束复色光由空气射向玻璃,发生折射而分为 a、b 两束单 色光,其传 播方向如图所示。设玻璃对 a、b 的折射率分别为 na 和 nb,a、 b 在玻璃中 的传播速度分别为 va 和 vb,则 A. na>nb B. na< nb C. va> vb D. va < vb 16.对于定量气体,可能发生的过程是 A.等压压缩,温度降低 B.等温吸热,体积不变 C.发出热量,内能增加 D.绝热压缩,内能不变 17.图中画出了氢原子的 4 个能级,并注明相应的能量 E。 处在 n=4 的能 级是一群氢原子向低能级跃迁时,能够发出若干种不同 频率的光波。 已知金属钾的逸出功为 2.22eV。在这些刚帮光波中,能 够从金属钾的 表面打出光电子的总共有 A.二种 B.三种 C.四种 D.五种 18.已知引力常量 G、月球到地球中心的距离 R 和月球绕 地球运行的周 期 T。仅利用这三个数据,可以估算出的物理量有 A.月球的质量 C.地球的质量 C.地区的半径 D.月球绕地球运行速度的大小 19.一简谐横波沿 X 轴正方向传播,某时刻其波形 如 图 所 示。下面说法正确的是 A.由波形图可知该波的波长 B. 由波形 图可知该 波的周期 C.经 4 1 周期后质元 P 运动到 Q 点 D. 经 4 1 周 期后质元 R 的速度变为零 20.处在匀强磁场中的矩形线圈 abcd,以恒定的角速度绕 ab 边转动,磁 场方向平行于纸面并与 ab 垂直。在 t=0 时刻,线圈平面与 纸面重合(如 图),线圈的 cd 边离开纸面向外运动。若规定由 a→b→c →d → a 方 向 的感应电流为正,则能反映线圈中感应电流 I 随时间 t 变 化的图象是 21.图中 a、b 是两个点电荷,它们的电量分别为 Q1、Q2, MN 是 ab 连线的中垂线,P 是中垂线上的一点。下列哪种情况能使 P 点的场强 方向指向 MN 的左侧 A. Q1、Q2 都是正电荷,且 Q1< Q2 B. Q1 是正电荷,Q2 是负电荷,且 Q1>| Q2| C. Q1 是负电荷,Q2 是正电荷,且|Q1|< Q2 D. Q1、Q2 都是负电荷,且|Q1|>| Q2| 非选择题:共 10 题,共 174 分。 22.(17 分) (1)用游标为 50 分度的卡尺(测量值可准确到 0.02mm)测定某圆柱的直径时,卡尺上的读 数如图。可读出圆柱的直径为________mm。 (2)利用图 1 所示的电路测量电流表 的内阻 RA。图中 R1、R2 为电阻,K1、K2 为电键, B 是电源(内阻可忽略)。 ①根据图 1 所给出的电路原理图,在图 2 的实物图上连线。 ②已知 R1=140Ω,R2=60Ω。当电键 K1 闭合、K2 断开时,电流表读数为 6.4mA;当 K1、K2 均闭合时,电流表读数为 8.5mA。由此可以求出=_______。(保留 2 位有效数字) 23.(16 分)如图所示,在水平桌面的边角处有一轻质光滑的定 滑轮 K,一 条不可伸长的轻绳绕过 K 分别与物块 A、B 相连,A、B 的质量 分 别 为 mA、mB。开始时系统。现用一水平恒力 F 拉物块 A,使物块 B 上升。已知 当 B 上升距离为 h 时,B 的速度为 v,求此过程中物块 A 克服摩 擦 力 所 做 的功。重力加速度为 g。 24.(19 分)在同时存在匀强电场和匀强磁场的空间中取正交 坐标系 Oxyz (z 轴正方向竖直向上),如图所示。已知电场方向沿 z 轴再正 方向,场强 大小为 E;磁场方向沿 y 轴正方向,磁场、磁感应强度的大小 为 B;重力加 速度为 g。问:一质量为 m、带电量为+q 的从原点出发的质点 能否在坐标 轴(x、y、z)上以速度 v 做匀速运动?若能,m、q、E、B、 v 及 g 应满足 怎样的关系?若不能,说明理由。 25.(20 分)质量为 M 的小物块 A 截止在离地面高 h 的水平桌面的边缘,质量为 m 的小物块 B 沿桌面向 A 运动并以速度 v0 与之发生正碰(碰撞时间极短)。碰后 A 离开桌面,其落地点离出 mA 发点的水平距离为 L。碰后 B 反向运动。求 B 后退的距离。以知 B 与桌面的动摩擦因数为μ。 重力加速度为 g。 26.(13 分)A、B、C、D、E 代表单质或化合物,它们之间的相互转换关系如下图所示。A 为 地壳中含量仅次于氧的非金属元素的单质,其晶体结构与金刚石相似。 A + E B D+H2O ++ C 加 碳 高 温 加氧化钙 高温 加氢氧化钠 加 热 请填空: (1) 形成单质 A 的原子的结构示意图为 ,它的最高化合价为 。 (2) B 的化学式(分子式)为 ,B 的晶体类型为 ,B 和碳反应生成 A 和 E 的化学方程式是 。 (3) C 的化学式(分子式)为 ,D 的化学式(分子式)为 。 27.(15 分)A、B、C、D、E 5 瓶透明溶液,分别是 HCl、BaCl2、NaHSO4、Na2CO3 和 AgNO3 中的一种。已知: ① A 与 B 反应有气体生成 ② B 与 C 反应有沉淀生成 ③ C 与 D 反应有沉淀生成 ④ D 与 E 反应有沉淀生成 ⑤ A 与 E 反应有气体生成 ⑥ 在②和③的反应中生成的沉淀是同一种物质 请填空: (1)在②和③的反应中,生成的沉淀物质的化学式(分子式)是 。 (2)A 是 ,B 是 ,C 是 ,D 是 ,E 是 , (3) A 与 E 反应的离子方程式是 。 28.(16 分):下图中的实验装置可用于制取乙炔。 请填空: (1) 图中,A 管的作用是 , 制取乙炔的化学方程式是 。 (2) 乙炔通入 KMnO4 酸性溶液中观察到的现象是 , 乙炔发生了 反应。 (3)乙炔通入溴的 CCl4 溶液中观察到的现是 , 乙炔发生了 反应。 (4)为了安全,点燃乙炔前应 ,乙炔燃烧时的实验现象 是 。 29.(16 分)某有机化合物 A 的相对分子质量(分子量)大于 110,小于 150。经分析得知,其 中碳和氢的质量分数之和为 52.24%,其余为氧。请回答: (1)该化合物分子中含有几个氧原子,为什么? (2)该化合物的相对分子质量(分子量)是 。 (3)该化合物的化学式(分子式)是 。 O || (4)该化合物分子中最多含 个 ─ C ─官能团。 30.(27 分)回答下列⑴、⑵小题。 ⑴植物叶片表皮分布有大量的气孔,气孔结构如图所示。当组成气孔的细胞(保卫细胞)吸 水后,会膨胀不变形,气孔开启;反之细胞失水收缩,气孔关闭。请以放置一小段时间的菠菜 为材料设计一个实验,证明气孔具有开启和关闭的功 能。要求 写出实验材料与主要用具、实验步骤、预测实验结果 并作出解 释。 实验材料与主要用具: 实验步骤: 预测实验结果并作出解释: ⑵某实验小组为了验证乙烯的生理作用,进行了下列 实验: 取 A、B 两箱尚未成熟的番茄(绿色),A 箱的番茄呈 ________ 色,B 箱番茄呈_______色。从这两箱番茄中取等量的果肉,分别研磨成匀浆,除去匀浆中的色 素,过滤。取无色的等量滤液分别加入 A、B 两支试管中,再各加入等量斐林试剂,加热后, 摇匀观察,发现 A 试管中呈砖红色,B 试管中也呈此颜色,但比 A 试管中的颜色_______(深 或浅),说明经乙烯利处理后的番茄中________含量增多了。 根据上述试验可证明乙烯具有_______的作用。 31.(15 分)已知果蝇中,灰身与黑身为一对相对性状(显性基因用 B 表示,隐性基因用 b 表 示);直毛与分叉毛为一对相对性状(显性基因用 F 表示,隐性基因用 f 表示)。两只亲代果蝇 杂交得到以下子代类型和比例: 灰身、直毛 灰身、分叉毛 黑身、直毛 黑身、分叉毛 雌蝇 4 3 0 4 1 0 雄蝇 8 3 8 3 8 1 8 1 请回答: ⑴控制灰身与黑身的基因位于_______;控制直毛与分叉毛的基因位于_______。 ⑵亲代果蝇的表现型为________、________。 ⑶亲代果蝇的基因型为________、________。 ⑷子代表现为灰身直毛的雌蝇中,纯合体与杂合体的比例为________。 ⑸子代雄蝇中,灰身分叉毛的基因型为_______、_______;黑身直毛的基因型为________。 理科综合能力测试参考答案 选择题共 21 小题,每小题 6 分,共 126 分。 一、 选择题:选择题对的给 6 分,选错或未选的给 0 分。 1.D 2.B 3.C 4.D 5.D 6.C 7.D 8.C 9.D 10.A 11.B 12.A 13.C 二、选择题:全部选对的得 6 分,选对但不全的得 3 分,有选错的得 0 分。 14.B 15.A D 16.A C 17.C 18.B D 19.A D 20.C 21.A C D 非选择题共 10 小题,共 173 分。 22.(17 分) (1)42.12(5 分) (2)①连线如图所示(6 分) ②43(6 分) 23.(16 分) 在此过程中,B 的重力势能的增量为 mBgh,A、B 动能的增量为 2 1 (mA+mB)v2,恒力 F 所做的功 为 Fh,用 w 表示 A 克服摩擦力所做的功,根据功能关系有 Fh-w= 2 1 (mA+mB)v2+mBgh 既 w= Fh- 2 1 (mA+mB)v2+mBgh 评分参考:最后答案正确的给 16 分。若最后答案有错,但知道 B 重力发改变量或 A、B 动能的 改变量或恒力的功,可给少量的分(总共不超过 16 分)。其他解法应拟定相应的评分标准。 24.(19 分) 已知带电质点受到的电场力为 qE,方向沿 z 轴正方向;质点受到的重力为 mg,沿 z 轴的 负方向。 假设质点在 x 轴上做匀速运动,则它受到的洛伦兹力必沿 z 轴正方向(当 v 沿 x 轴正方向) 或沿 z 轴负方向(当 v 沿 x 轴负方向),要质点做匀速运动必分别有 qvb+qE=mg ① 或 qE=qvb+mg ② 假设质点在 y 轴上做匀速运动,则无论沿 y 轴正方向还是负方向,洛伦兹力都为 0,要质 点做匀速运动必有 qE=mg ③ 假设质点在 z 轴上做匀速运动,则它受到的洛伦兹力 必平行于 x 轴,而电场力和重力都平 行于 z 轴,三力的合力不可能为 0,与假设矛盾,故质点不能在 z 轴上做匀速运动。 评分参考:①②③式各 5 分,说明不能在 z 轴上做匀速运动给 4 分。 25.(20 分) 设 t 为 A 从离开桌面至落地经历时间,V 表示刚碰后 A 的速度,有 h= 2 1 gt2 ① L=Vt ② 设 v 为刚碰后 B 的速度的大小,由动量守恒, mv0=MV-mv ③ 设 B 后退的距离为 l,由功能关系, μmgl= 2 1 mv2 ④ 由以上各式得 l= 2 022 1 )( vh g m ML g  ⑤ 评分参考:①②③④⑤式各 4 分。 26.(13 分) (1) (2)SiO2 原子晶体 SiO2 +2C===Si +2CO↑ (6 分) (3)CaSiO3 Na2SiO3 (4 分) 27.(15 分) (1)AgCl (2 分) (2)Na2CO3 HCl AgNO3 BaCl2 NaHSO4 (各 2 分,共 10 分) (3)CO 2 3 +2H+====H2O +CO2↑ (3 分) 28.(16 分) (1)调解水面高度以控制反应的发生和停止。 CaC2+2H2O====Ca(OH)2 +C2H2↑ (2)KMnO4 溶液褪色 氧化 (3)溴的四氯化碳溶液褪色 加成 (4)检查乙炔的纯度;火焰明亮并伴有浓烈的黑烟 (各 4 分,共 16 分) 29.(16 分) (1)4 个氧原子 因为 110nb B. na< nb C. va> vb D. va < vb 16.对于定量气体,可能发生的过程是 A.等压压缩,温度降低 B.等温吸热,体积不变 C.发出热量,内能增加 D.绝热压缩,内能不变 17.图中画出了氢原子的 4 个能级,并注明相应的能量 E。 处在 n=4 的能 级是一群氢原子向低能级跃迁时,能够发出若干种不同 频率的光波。 已知金属钾的逸出功为 2.22eV。在这些刚帮光波中,能 够从金属钾的 表面打出光电子的总共有 A.二种 B.三种 C.四种 D.五种 18.已知引力常量 G、月球到地球中心的距离 R 和月球绕 地球运行的周 期 T。仅利用这三个数据,可以估算出的物理量有 A.月球的质量 C.地球的质量 C.地区的半径 D.月球绕地球运行速度的大小 19.一简谐横波沿 X 轴正方向传播,某时刻其波形 如 图 所 示。下面说法正确的是 A.由波形图可知该波的波长 B. 由波形 图可知该 波的周期 C.经 4 1 周期后质元 P 运动到 Q 点 D. 经 4 1 周 期后质元 R 的速度变为零 20.处在匀强磁场中的矩形线圈 abcd,以恒定的角速度绕 ab 边转动,磁 场方向平行于纸面并与 ab 垂直。在 t=0 时刻,线圈平面与 纸面重合(如 图),线圈的 cd 边离开纸面向外运动。若规定由 a→b→c →d → a 方 向 的感应电流为正,则能反映线圈中感应电流 I 随时间 t 变 化的图象是 21.图中 a、b 是两个点电荷,它们的电量分别为 Q1、Q2, MN 是 ab 连线的中垂线,P 是中垂线上的一点。下列哪种情况能使 P 点的场强 方向指向 MN 的左侧 A. Q1、Q2 都是正电荷,且 Q1< Q2 B. Q1 是正电荷,Q2 是负电荷,且 Q1>| Q2| C. Q1 是负电荷,Q2 是正电荷,且|Q1|< Q2 D. Q1、Q2 都是负电荷,且|Q1|>| Q2| 非选择题:共 10 题,共 174 分。 22.(17 分) (1)用游标为 50 分度的卡尺(测量值可准确到 0.02mm)测定某圆柱的直径时,卡尺上的读 数如图。可读出圆柱的直径为________mm。 (2)利用图 1 所示的电路测量电流表 的内阻 RA。图中 R1、R2 为电阻,K1、K2 为电键, B 是电源(内阻可忽略)。 ①根据图 1 所给出的电路原理图,在图 2 的实物图上连线。 ②已知 R1=140Ω,R2=60Ω。当电键 K1 闭合、K2 断开时,电流表读数为 6.4mA;当 K1、K2 均闭合时,电流表读数为 8.5mA。由此可以求出=_______。(保留 2 位有效数字) mA 23.(16 分)如图所示,在水平桌面的边角处有一轻质光滑的定 滑轮 K,一 条不可伸长的轻绳绕过 K 分别与物块 A、B 相连,A、B 的质量 分 别 为 mA、mB。开始时系统。现用一水平恒力 F 拉物块 A,使物块 B 上升。已知 当 B 上升距离为 h 时,B 的速度为 v,求此过程中物块 A 克服摩 擦 力 所 做 的功。重力加速度为 g。 24.(19 分)在同时存在匀强电场和匀强磁场的空间中取正交 坐标系 Oxyz (z 轴正方向竖直向上),如图所示。已知电场方向沿 z 轴再正 方向,场强 大小为 E;磁场方向沿 y 轴正方向,磁场、磁感应强度的大小 为 B;重力加 速度为 g。问:一质量为 m、带电量为+q 的从原点出发的质点 能否在坐标 轴(x、y、z)上以速度 v 做匀速运动?若能,m、q、E、B、 v 及 g 应满足 怎样的关系?若不能,说明理由。 25.(20 分)质量为 M 的小物块 A 截止在离地面高 h 的水平桌面的边缘,质量为 m 的小物块 B 沿桌面向 A 运动并以速度 v0 与之发生正碰(碰撞时间极短)。碰后 A 离开桌面,其落地点离出 发点的水平距离为 L。碰后 B 反向运动。求 B 后退的距离。以知 B 与桌面的动摩擦因数为μ。 重力加速度为 g。 26.(13 分)A、B、C、D、E 代表单质或化合物,它们之间的相互转换关系如下图所示。A 为 地壳中含量仅次于氧的非金属元素的单质,其晶体结构与金刚石相似。 A + E B D+H2O ++ C 加 碳 高 温 加氧化钙 高温 加氢氧化钠 加 热 请填空: (2) 形成单质 A 的原子的结构示意图为 ,它的最高化合价为 。 (2) B 的化学式(分子式)为 ,B 的晶体类型为 ,B 和碳反应生成 A 和 E 的化学方程式是 。 (3) C 的化学式(分子式)为 ,D 的化学式(分子式)为 。 27.(15 分)A、B、C、D、E 5 瓶透明溶液,分别是 HCl、BaCl2、NaHSO4、Na2CO3 和 AgNO3 中的一种。已知: ① A 与 B 反应有气体生成 ② B 与 C 反应有沉淀生成 ③ C 与 D 反应有沉淀生成 ④ D 与 E 反应有沉淀生成 ⑤ A 与 E 反应有气体生成 ⑥ 在②和③的反应中生成的沉淀是同一种物质 请填空: (1)在②和③的反应中,生成的沉淀物质的化学式(分子式)是 。 (2)A 是 ,B 是 ,C 是 ,D 是 ,E 是 , (3) A 与 E 反应的离子方程式是 。 28.(16 分):下图中的实验装置可用于制取乙炔。 请填空: (3) 图中,A 管的作用是 , 制取乙炔的化学方程式是 。 (4) 乙炔通入 KMnO4 酸性溶液中观察到的现象是 , 乙炔发生了 反应。 (3)乙炔通入溴的 CCl4 溶液中观察到的现是 , 乙炔发生了 反应。 (4)为了安全,点燃乙炔前应 ,乙炔燃烧时的实验现象 是 。 29.(16 分)某有机化合物 A 的相对分子质量(分子量)大于 110,小于 150。经分析得知,其 中碳和氢的质量分数之和为 52.24%,其余为氧。请回答: (1)该化合物分子中含有几个氧原子,为什么? (2)该化合物的相对分子质量(分子量)是 。 (3)该化合物的化学式(分子式)是 。 O || (4)该化合物分子中最多含 个 ─ C ─官能团。 30.(27 分)回答下列⑴、⑵小题。 ⑴植物叶片表皮分布有大量的气孔,气孔结构如图所示。当组成气孔的细胞(保卫细胞)吸 水后,会膨胀不变形,气孔开启;反之细胞失水收缩,气孔关闭。请以放置一小段时间的菠菜 为材料设计一个实验,证明气孔具有开启和关闭的功 能。要求 写出实验材料与主要用具、实验步骤、预测实验结果 并作出解 释。 实验材料与主要用具: 实验步骤: 预测实验结果并作出解释: ⑵某实验小组为了验证乙烯的生理作用,进行了下列 实验: 取 A、B 两箱尚未成熟的番茄(绿色),A 箱的番茄呈 ________ 色,B 箱番茄呈_______色。从这两箱番茄中取等量的果肉,分别研磨成匀浆,除去匀浆中的色 素,过滤。取无色的等量滤液分别加入 A、B 两支试管中,再各加入等量斐林试剂,加热后, 摇匀观察,发现 A 试管中呈砖红色,B 试管中也呈此颜色,但比 A 试管中的颜色_______(深 或浅),说明经乙烯利处理后的番茄中________含量增多了。 根据上述试验可证明乙烯具有_______的作用。 31.(15 分)已知果蝇中,灰身与黑身为一对相对性状(显性基因用 B 表示,隐性基因用 b 表 示);直毛与分叉毛为一对相对性状(显性基因用 F 表示,隐性基因用 f 表示)。两只亲代果蝇 杂交得到以下子代类型和比例: 灰身、直毛 灰身、分叉毛 黑身、直毛 黑身、分叉毛 雌蝇 4 3 0 4 1 0 雄蝇 8 3 8 3 8 1 8 1 请回答: ⑴控制灰身与黑身的基因位于_______;控制直毛与分叉毛的基因位于_______。 ⑵亲代果蝇的表现型为________、________。 ⑶亲代果蝇的基因型为________、________。 ⑷子代表现为灰身直毛的雌蝇中,纯合体与杂合体的比例为________。 ⑸子代雄蝇中,灰身分叉毛的基因型为_______、_______;黑身直毛的基因型为________。 理科综合能力测试参考答案 选择题共 21 小题,每小题 6 分,共 126 分。 二、 选择题:选择题对的给 6 分,选错或未选的给 0 分。 1.D 2.B 3.C 4.D 5.D 6.C 7.D 8.C 9.D 10.A 11.B 12.A 13.C 二、选择题:全部选对的得 6 分,选对但不全的得 3 分,有选错的得 0 分。 14.B 15.A D 16.A C 17.C 18.B D 19.A D 20.C 21.A C D 非选择题共 10 小题,共 173 分。 22.(17 分) (1)42.12(5 分) (2)①连线如图所示(6 分) ②43(6 分) 23.(16 分) 在此过程中,B 的重力势能的增量为 mBgh,A、B 动能的增量为 2 1 (mA+mB)v2,恒力 F 所做的功 为 Fh,用 w 表示 A 克服摩擦力所做的功,根据功能关系有 Fh-w= 2 1 (mA+mB)v2+mBgh 既 w= Fh- 2 1 (mA+mB)v2+mBgh 评分参考:最后答案正确的给 16 分。若最后答案有错,但知道 B 重力发改变量或 A、B 动能的 改变量或恒力的功,可给少量的分(总共不超过 16 分)。其他解法应拟定相应的评分标准。 24.(19 分) 已知带电质点受到的电场力为 qE,方向沿 z 轴正方向;质点受到的重力为 mg,沿 z 轴的 负方向。 假设质点在 x 轴上做匀速运动,则它受到的洛伦兹力必沿 z 轴正方向(当 v 沿 x 轴正方向) 或沿 z 轴负方向(当 v 沿 x 轴负方向),要质点做匀速运动必分别有 qvb+qE=mg ① 或 qE=qvb+mg ② 假设质点在 y 轴上做匀速运动,则无论沿 y 轴正方向还是负方向,洛伦兹力都为 0,要质 点做匀速运动必有 qE=mg ③ 假设质点在 z 轴上做匀速运动,则它受到的洛伦兹力 必平行于 x 轴,而电场力和重力都平 行于 z 轴,三力的合力不可能为 0,与假设矛盾,故质点不能在 z 轴上做匀速运动。 评分参考:①②③式各 5 分,说明不能在 z 轴上做匀速运动给 4 分。 25.(20 分) 设 t 为 A 从离开桌面至落地经历时间,V 表示刚碰后 A 的速度,有 h= 2 1 gt2 ① L=Vt ② 设 v 为刚碰后 B 的速度的大小,由动量守恒, mv0=MV-mv ③ 设 B 后退的距离为 l,由功能关系, μmgl= 2 1 mv2 ④ 由以上各式得 l= 2 022 1 )( vh g m ML g  ⑤ 评分参考:①②③④⑤式各 4 分。 26.(13 分) (1) (2)SiO2 原子晶体 SiO2 +2C===Si +2CO↑ (6 分) (3)CaSiO3 Na2SiO3 (4 分) 27.(15 分) (1)AgCl (2 分) (2)Na2CO3 HCl AgNO3 BaCl2 NaHSO4 (各 2 分,共 10 分) (3)CO 2 3 +2H+====H2O +CO2↑ (3 分) 28.(16 分) (1)调解水面高度以控制反应的发生和停止。 CaC2+2H2O====Ca(OH)2 +C2H2↑ (2)KMnO4 溶液褪色 氧化 (3)溴的四氯化碳溶液褪色 加成 (4)检查乙炔的纯度;火焰明亮并伴有浓烈的黑烟 (各 4 分,共 16 分) 29.(16 分) (1)4 个氧原子 因为 110b B. 在滴有酚酞溶液的氨水中,加入 NH4CL 的溶液恰好无色,则此时溶液 PH< 7 C. 1.0×10-3mol/L 盐酸的 PH=3.0,1.0×10-8mol/L 盐酸 PH=8.0 D. 若 1mL PH=1 盐酸与 100mL MaOH 溶液混合后,溶液 PH=7,则 NaOH 溶液的 PH=11 10.右图是一种染料敏化太阳能电池的示意图。电池的一个由有机光敏染料(S)涂覆 TiO2 纳米 晶 体 表 面 制 成 , 另 一 电 极 由 导 电 玻 璃 镀 铂 李 自 成 , 电 池 中 发 生 的 反 应 为 : 下列关于该电池叙述错误的是 A.电池工作时, I  离子在镀铂导电玻璃电极上放电 B.电池工作时,是将太阳能转化为电能 C.电池的电解质溶液中 I  和 I  3 浓度不会减少 D.电池中镀铂导电玻璃为正极 11.下图表示 4-溴环已烯所发生的 4 个不同反应。其中,产物只含有一种官能团的反应是 A. ①② B.②③ C.③④ D.①④ 12.一定条件下磷与干燥氯气反应,若 0.25 g 磷消耗掉 314mL 氯气(标准状况),则产物中 PCL3 与 PCL5 的物质的量之比接近于 A. 3 : 1 B. 5 : 3 C. 2 : 3 D. 1 : 2 13.下面关于 2sio 晶体网状结构的叙述正确的是 A..最小的环上,有 3 个 si 原子和 3 个 o 原子 B.最小的环上, si 和 o 原子数之比为 1:2 C.最小的环上,有 6 个 si 原子和 6 个 o 原子 D.存在四面体结构单元, o 处于中心, si 处于 4 个顶角 二、选择题(本题共 8 小。在每小题给出的四个选项中,有的只有一个选项正确, 有的有多个选项正确,全部选对的得 6 分,选对但不全的得 3 分,有选错的得 0 分。) 14.原子核 238 92  经放射性衰变①变为原子 234 90 Th ,继而经放射性衰变②变为原子核 234 91 Pa ,再经 放射性衰变③变为原子核 234 92  。放射性衰变①、②和③依次为 A..  衰变、  衷变和  衰变 B.  衰变、 衷变和  衰变 C.  衰变、  衰变和 衰变 D.  衰变、  衰变和 衰变 15.如右图,轻弹簧上端与一质量为 m 的木块 1 相连,下端与另一质量为 M 的木块 2 相连,整 个系统置于水平放置的光滑木坂上,并处于静止状态。现将木板沿水平方向突然抽出,设抽 出后的瞬间,木块 1、2 的加速度大小分别为 1a 、 2a  重力加速度大小为 g 则有 A. 1 0a  , 2a g B. 1a g , 2a g C. 1 20, m Ma a gM   D. 1a g , 2 m Ma gM  16.关于静电场,下列结论普遍成立的是 A..电场强度大的地方电势高,电场强度小的地方电势低 B.电场中任意两点之间的电势差只与这两点的场强有关 C.在正电荷或负电荷产生的静电场中,场强方向都指向电势降低最快的方向 D.将正点电荷从场强为零的一点移动到场强为零的另一点,电场力做功这零 17.某地的地磁场磁感应强度的竖直分量方向向下,大小为 4.5×10-5T。一灵敏电压表连接在当 地入海河段的两岸,河宽 100m,该河段涨潮和落潮时有海水(视为导体)流过。设落潮时, 海水自西向东流,流速为 2m/s。下列说法正确的是 A.电压表记录的电压为 5mV B.电压表记录的电压为 9mV C.河南岸的电势较高 D.河北岩的电势较高 18.一水平抛出的小球落到一倾角为 的斜面上时, 其速度方向与斜面垂直,运动轨迹如右图中虚 线所示。小球在竖直方向下落的距离与在水平 方向通过的距离之比为 A. tan B. 2tan C. 1 tan D. 1 2tan 19.右图为两分子系统的势能 pE 与两分子间距离 r 的关系曲线。下列说法正确的是 A.当 r 大于 r 1 时,分子间的作用力表现为引力 B.当 r 小于 r 1 时,分子间的作用力表现为斥力 C.当 r 等于 r 2 时,分子间的作用力为零 D.当 r 由 r 1 变到 r 2 的过程中,分子间的作用力做负功 20.某人手持边长为 6cm 的正方形平面镜测量身后一棵树的高度。测量时保持镜面与地面垂直, 镜子与眼睛的距离为 0.4m。在某位置时,他在镜中恰好能够看到整棵树的像;然后他向前 走了 6.0m,发现用这个镜子长度的 5/6 就能看到整棵树的像。这棵树的高度约为 A.4.0m B.4.5m C.5.0m D.5.5m 21.一简谐振子沿 x 轴振动,平衡位置在坐标原点。t =0 时刻振子的位移 x =-0.1m;t = 4 3 s 时 刻 x =0.1m;t =4 s 时刻 x =0.1m。该振子的振幅和周期可能为 A.0.1m, 8 3 s B.0.1m,8 s C.0.2m, 8 3 s D.0.2m,8 s 2010 年普通高等学校招生全国统一考试 理科综合能力测试 第Ⅱ卷 注意事项: 1.答题前,考生先在答题卡上用直径 0.5 毫米的黑色墨水签字笔将自己的姓名,准考证号填 写清楚,然后贴好条形码。请认真核准条形码上的准考证号,姓名和科目。 2.第Ⅱ卷共 8 页,请用直径 0.5 毫米黑色墨水签字笔在答题卡上各题的答题区域内作答, 在试卷上作答无效........ 3.第Ⅱ卷共 13 题,共 174 分。 22.(6 分)(注意:在试题卷上作答无效............) 图 1 是利用激光测转速的原理示意图,图中圆盘可绕固定轴转动,盘边缘侧面上有一小 段涂有很薄的反光材料。当盘转到某一位置时,接收器可以接受到反光涂层所反射的激光束, 并将所收到的光信号转变成电信号,在示波器显示屏上显示出来(如图 2 所示)。 (1)若图 2 中示波器显示屏横向的每大格(5 小格)对应的事件为 25.00 10 s ,则圆盘的转 速 为 转/s。(保留 3 位有效数字) (2)若测得圆盘直径为10.20cm,则可求得圆盘侧面反光涂层的长度为 cm 。 (保留 3 位有效数字) 23.(12 分)(注意:在试题卷上作答无效............) 一电流表的量程标定不准确,某同学利用图 1 所示电路测量该电流表的实际量程 m I 所用器 材有: 量程不准的电流表 1A ,内阻 1r =10.0 ,量程标称为 5.0 mA; 标准电流表 A2,内阻 r2=45,量程为 1.0; 标准电阻 R,阻值 10; 滑动变阻器 R,总电阻约为 3000; 电源 E,电动势为 3, 内阻不计; 保护电阻 R;开关 S;导线。 回答下列问题: (1)在图 2 所示的实物图上画出连线。 (2)开关 S 闭合前,滑动变阻器的滑动端 C 应滑动至 端。 (3)开关 S 闭合后,调节滑动变阻器的滑动端,使电流表 A1 满偏;若此时电流表 A2 的读 数为 I2,则 A1 的量程 Im 为 。 (4)若测量时,A1 未调到满偏,两电流表的示数如图 3 所示,从图中读出 A1 的示数 I1= ,A2 的示数 I2= ;由读出的数据计算得 Im= 。(保留 3 位有效数 字) (5)写一条提高测量准确度的建议: 。 24.(15 分)(注意:在试题卷上作答无效............) 汽车由静止开始在平直的公路上行驶,0~60s 内汽车的加速度随时间变化的图线如右图所 示。 (1)画出汽车在 0~60s 内的 v-t 图线; (2)求这 60s 内汽车行驶的路程。 25.(18 分)(注意:在试卷题上作答无效.........) 如右图,质量分别为 m 和 M 的两个星球 A 和 B 在引力作用下都绕 O 点做匀速 圆周运动,星球 A 和 B 两者中心之间的距离为 L。已知 A、B 的中心和 O 三点始终 共线,A 和 B 分别在 O 的两侧。引力常数为 G。 (1)求两星球做圆周运动的周期: (2)在地月系统中,若忽略其他星球的影响,可以将月球和地球看成上述星球 A 和 B,月球绕其轨道中心运行的周期为 1T 。但在近似处理问题时,常常认为月球是绕地 心做圆周运动的,这样算得的运行周期记为 2T 。已知地球和月球的质量分别为 245.98 10 kg 和 227.35 10 kg 。求 2T 与 1T 两者平方之比。(结果保留 3 位小数) 26.(21 分)(注意:在试卷题上作答无效.........) 如下图,在0 3x a  区域内存在与 xy 平面垂直的匀强磁场,磁感应强度的大小为 B。在 t=0 时刻,一位于坐标原点的粒子源在 xy 平面内发射出大量同种带电粒子,所有粒子的 初速度大小相同,方向与 y 轴正方向夹角分布在 0~180°范围内。已知沿 y 轴正方向发射的粒 子在 t= 0t 时刻刚好从磁场边界上 P( 3a ,a)点离开磁场。求: (1)粒子在磁场中做圆周运动的半径 R 及粒子的比荷 q/m; (2)此时刻仍在磁场中的粒子的初速度方向与y轴正方向夹角的取值范围; (3)从粒子发射到全部粒子离开磁场所用的时间. 27.(15 分) (注意:在试卷题上作答无效.........) 在溶液中,反应 A+2B C 分别在三种不同实验条件下进行,它们的起始浓度均 为 c(A)=0.100mol/L、c(B)=0.200mol/L 及 c(C)=0mol/L。反应物 A 的浓度随时间的变化如下 图所示。 请回答下列问题: (1)与①比较,②和③分别仅改变一种反应条件。所改变的条件和判断的理由是: ②___________________________________________________________________ ③___________________________________________________________________ (2)实验②平衡时 B 的转化率为_______;实验③平衡时 C 的浓度为___________; (3)该反应的 H______0,其判断理由是___________________________________________ (4)该反应进行到 4.0min 时的平均反应速率: 实验②: Bv =_____________________________________________________________ 实验③: cv =______________________________________________________________ 28.(15 分)(注意:在试题卷上作答无效) 有 A,B,C,D,E 和 F 六瓶无色溶液,它们都是中学化学中常用的无机试剂。纯 E 为无色油状液 体;B,C,D 和 F 是盐溶液,且它们的阴离子均不同。现进行如下实验: 1 A 有刺激性气味,用沾有浓盐酸的玻璃棒接近 A 时产生白色烟雾; 2 将 A 分别加入其它物种溶液中,只有 D,F 中有沉淀生成;继续加入过量 A 时,D 中沉淀 无变化,F 中沉淀完全溶解; 3 将 B 分别加入 C,D,E,F 中,C,D,F 中产生沉淀,E 中有无色,无味气体溢出; 4 将 C 分别加入 D,E,F 中,均有沉淀生成,再加入稀 HNO3,沉淀均不溶。 根据上述实验信息,请回答下列问题: (1)能确定的溶液是(写出溶液标号与相应溶质的化学式): ____________________________________________________ ____________________________________________________ (2)不能确定的溶液,写出其标号,溶质可能的化学式及进一步鉴别的方法: ____________________________________________________ ____________________________________________________ 29.(15 分)(注意:在试题卷上作答无效) 请设计 CO2 在高温下与木炭反应生成 CO 的实验。 (1) 在下面方框中,A 表示由长颈漏斗和锥形瓶组成的气体发生器,请在 A 后完成该 反应的实验装置示意图(夹持装置、连接胶管及尾气处理部分不必画出,需要加 热的仪器下方用△标出),按气流方向在每件仪器下方标出字母 B,C……;其它可 选用的仪器(数量不限)简易表示如下: (2)根据方框中的装置图,填写下表 仪器标号 仪器中所加物质 作用 A 石灰石,稀盐酸 石灰石与盐酸作用产生 CO2 (3)有人对气体发生器作如下改进:在锥形瓶中放入一小试管,将长颈漏斗下端插入小试管中。 改进后的优点是____________________________________________________; (4)验证 CO 的方法是_____________________________________________________。 30.(15 分)(注意:在试题卷上作答无效) 有机化合物 A-H 的转换关系如下所示: 请回答下列问题: (1)链径 A 有支链且只有一个官能团,其相对分子质量在 65~75 之间,lnolA 完全燃烧消 耗 7mol 氧气,则 A 的结构简式是______,名称是___________________; (2)在特定催化剂作用下,A 与等物质的量的 H2 反应生成 E。由 E 转化为 F 的化学方程式 是_____________________________________________________________________; ( 3 ) G 与 金 属 钠 反 应 能 放 出 气 体 。 由 G 转 化 为 H 的 化 学 方 程 式 是 __________________________________________________________________; (4)①的反应类型是______________;③的反应类型是_____________________; (5)链径 B 是 A 的同分异构体,分子中的所有碳原子共平面,其催化氧化产物为正戊烷, 写出 B 所有可能的结构简式______________________________________________________; (6)C 也是 A 的一种同分异构体,它的一氯代物只有一种(不考虑立体异构)。则 C 的结构简 式为___________________________________________________。 31.(8 分)(注意:在试题卷上作答无效) 从某植物长势一致的黄化苗上切取等长幼茎段(无叶和侧芽),自茎段顶端向下对称纵切至约 3/4 处。将切开的茎段浸没在蒸馏水中。一段时间后,观察到半边茎向外弯曲生长,如图所示。 若上述黄化茎段中的生长素浓度是促进生长的,放入水中后半边茎内,外两侧细胞中生长素浓 度都不会升高。请仅根据生长素的作用特点分析半边茎向外弯曲生长这一现象,推测出现该现 象的两种可能原因。 原因 1 是_________________________________________________________________。 原因 2 是___________________________________________________________________。 32.(10 分)(注意:在试题卷上作答无效............) 用去除脑但保留脊椎的哇(称脊蛙)为材料,进行反射活动实验。请回答与此有关的问题: (1)用针刺激脊蛙左后肢的趾部,可观察到该后肢出现收缩活动。该反射活动的感受其位 于左后肢趾部的____________中,神经中枢位于_____________中。 (2)反射活动总是从感受器接受刺激开始到效应器产生反映结束,这一方向性是由 _______________________________________________所决定的。 (3)剪断支配脊蛙左后肢的传出神经(见右图),立即刺激 A 端____________(能、不能) 看到左后肢收缩活动。若刺激剪断处的某一段出现伸缩活动,该活动__________(能、不能) 称为反射活动,主要原因是__________________________________。 33.(12 分)(注意:在试题卷上作答无效............) 现有 4 个纯合南瓜品种,其中 2 个品种的果形表现为圆形(圆甲和圆乙),1 个表现为扁盘 形(扁盘),1 个表现为长形(长)。用这 4 个南瓜品种做了 3 个实验,结果如下 实验 1:圆甲×圆乙,F1 为扁盘,F2 中扁盘:圆:长=9:6:1 实验 2:扁盘×长,F1 为扁盘,F2 中扁盘:圆:长=9:6:1 实验 3:用长形品种植株的花粉分别对上述杂交组合的 P1 植株授粉,其后代中扁盘:圆: 长均等于 1:2:1。综合上述实验结果,请回答: (1)南瓜果形的遗传受_________对等位基因控制,且遵循________________定律。 (2)若果形由一对等位基因控制用 A、a 表示,若由两对等位基因控制用 A、a 和 B、b 表 示 , 以 此 类 推 , 则 圆 形 的 基 因 型 应 为 _____________________ , 扁 盘 的 基 因 型 为 ________________________,长形的基因型应为________________________。 (3)为了验证(1)中的结论,可用长形品种植株的花粉对实验 1 得到的 F2 植株授粉,单 株收获 F2 中扁盘果实的种子,每株的所有种子单独种植在一起可得到一株系。观察多个这样的 株系,则所有株系中,理论上有 1/9 的株系 F3 果形均表现为扁盘,有_________的株系 F3 果形 的表现型及其数量比为扁盘:圆=1:1,有____________的株系 F3 果形的表现型及其数量比为 ____________________________________________。 34.(12 分)(注意:在试题卷上作答无效.........) 下列是与微生物培养有关的问题,请回答: ⑴某细菌固体培养基的组成成分是 2 4KH PO 、、 2 4Na HPO 4MgSO 、葡萄糖、尿素、琼脂和蒸 馏水,其中凝固剂是 ,碳源是 ,氮源是 。已知只有能合成脲酶的 细菌才能在该培养基上生长,帮该培养基属于 培养基。按照化学成分分类,该培养基 属于 培养基。从同化作用类型看,用该培养的细菌属于 。 ⑵将少量细菌接种到一定体积的液体培养基中,适宜条件下培养,定时取样测定菌体数目,以 时间为横坐标,以菌体数目的对数为纵坐标,可以得到细菌的 曲线。该曲线中以菌 体数目的对数作为纵坐标的原因是 。实验室中,为了获得形态和生理特证一致的菌体,一般应 在 期取体;在生产中,常收集培养至 期的细菌用 于次生代谢产物的提取。 理科综合能力测试(陕西卷) 一、 选择题 本题共 13 小题,每小题 6 分,在每小题给出的四个选项中,只有一项是符合 题目要求的。 1.下列有关细胞的叙述,正确的是 (D) A.病毒是一类具有细胞结构的生物 B.海澡细胞具有细胞核且 DNA 分子呈环状 C.人体所有细胞的细胞周期持续时间相同 D.内质网膜和高尔基体膜都具有流动性 2.下列关于呼吸作用的叙述,正确的是 (D) A.无氧呼吸的终产物是丙酮酸 B.有氧呼吸产生的 在线粒体基质中与氧结合生成水 C.无氧呼吸不需要 的参与,该过程最终有 的积累 D.质量相同时,脂肪比糖原有氧氧化释放的能量多 3.若要在普通显微镜下观察到质壁分离、RNA 和脂肪,下列四组材料中应选择的一组是 (C) A.水稻胚乳和花生子叶 B.天竹葵叶和水稻胚乳 C. 紫色洋葱和花生子叶 D.天竺葵叶和紫色洋葱 4.水中氧含量随水温的升高而下降,生活在寒温带湖泊中的某动物,其血液中的血红蛋白含量 与其生活的水温有关。右图中能正确表示一定温度范围内动物血液中血红蛋白含量随水温变化 趋势的曲线是 【A】 A 甲 B。乙 C.丙 D.丁 5..将神经细胞置于相当于细胞外液的溶液(溶液 S)中,可测到静息电位。给予细胞一个适宜 的刺激,膜两侧出现一个暂时性的电位变化,这种膜电位变化称为动作电位。适当降低溶液 S 中 NA+浓度,测量该细胞的静息电位和动作电位,可观测到 【D】 A.静息电位值减小 B.静息电位值增大 C.动作电位峰值升高 D.动作电位峰值降低 6.在白花豌豆品种栽培园中,偶然发现了一株开红花的豌豆植株,推测该红花表现型的出现是 花色基因突变的结果。为了确定推测是否正确,应检测和比较红花植株与百花植株中 【B】 A 白花基因的碱基组成 B 花色基因的 DNA 序列 C.细胞的 DNA 含量 D.细胞的 RNA 含量 7.下列各项表达中正确的是 【C】 A. Na2O2 的电子式为Na Na B.106g 的乙醇和丙醇混合液完全燃烧生成的 CO2 为 112L(标准状态) C.在氮原子中,质子数为 7 而种子数不一定为 7 D.CL-的结构示意图为 8.分子式为 C3H6CL2 的同分异构体共有(不考虑例题异构) 【B】 A.3 中 B.4 种 C.5 种 D.6 种 9.下列各组的反应,属于统一反应类型的是 【D】 A.由溴丙烷睡解制丙醇:由丙烯和水反应制丙醇 B.由甲苯硝化制对硝基甲苯:由甲苯氧化制苯甲酸 C.由苯乙烷消去制环乙烯:由丙烯加溴制 1,2-二溴丙烷 D.由乙酸和乙醇制乙酸乙酯:由苯甲酸乙酯水解制苯甲酸和乙醇 10.把 500ml 含有 BaCl2 和 KCl 的混合溶液分成 5 等分,取一份加入含 a mol 硫酸钠的溶液,恰 好是钡离子完全沉淀:令取一份加入 b mol 硝酸银的溶液,恰好使卤离子完全沉淀,则该混合 溶液中钾离子浓度为 【D】 A.0.1(b-2a)mol·L-1 B.10(2a-b) mol·L-1 C.10(b-a) mol·L-1 D.10(b-2a) mol·L-1 11.已知:HCN(aq)与 NaOH(aq)反应的△H 等于 【C】 A.-67.7KJ·mol·L-1 B.-43.5 KJ·mol·L-1 C.+43.5 KJ·mol·L-1 D.+67.7 KJ·mol·L-1 12.根据右图,可判断出下列离子方程式中错误的是 (A) A.2Ag(s) + Cd2+(s) = 2Ag(s) + Cd(s) B. Co2+(aq)+ Cd(s) = Co(s)+ Cd2+(aq) C. 2Ag (aq) + Cd(s) = 2Ag(s) Cd2+(aq) D. 2Ag (aq) +Co(s)=2Ag(s)+Co2+(aq) 13.下表中评价合理的是 二.选择题:本体共 8 小题,每小题 6 分,在每小题给出的四个选项中,有的只有一个选项正 确,有的有多个选项正确,全部选对的得 6 分,选对但不全的得 3 分,有选错的得 0 分。 14.在电磁学发展过程中,许多科学 ia 作出了贡献,下列说法正确的是 【A】 A.奥斯特发现了电流磁效应;法拉第发现了电磁感应现象 B.麦克斯韦预言了地磁波;楞次用实验证实了地磁波的存在 C.昆仑发现了点电荷的相互作用规律;密立根发现了磁场对电流的作用规律 D.安培发现了磁场对运动电荷的作用规律;洛仑兹发现了磁场对电流的作用规律 15.一根轻质弹簧一端固定,用大小为 F1 的力压弹簧的另一端,平衡的长度为 l1;改用大小为 F2 的力拉弹簧;平衡的长度为 l2。弹簧的拉伸或压缩均在弹性限度内,该弹簧的劲度系数为 【C】 A. B. C. D. 16.如图所示,在外力作用下某质点运动的 v-t 图像为正泫曲线。从图像可以判断【A、D】 A.在 0~t1 时间内,外力作正功 B. 在 0~t1 时间内外力的功率逐渐增大 C.在 t2 时刻,外力的功率最大 D.在 t1~t3 时间内,外力做的总功为零 17.静电除尘器是目前普遍采用的一种高效除尘去。某除尘器模型的收衬板是很长的条形金属 板,图中直线 a、b 为该收尘器的横截面。工作时收尘器带正电,其左侧的电场分布如图所示, 粉尘带负电,在电场作用下向收尘版运动,左后落在收尘版上。 若用 粗黑曲线表示原来静止与点 P 的带电粉尘颗粒的运动轨迹,下 列 4 附图中可能正确的是(葫芦哦重力和空气阻力) 【A】 18.如图所示,一物体置于水平地面上。当用水平方向成 60°角的力 F1 时,物体做直线运动: 当改用与水平方向成 30°角的力 F2 拉物体时,物体仍做匀速直线运动。若 F1 和 F2 的大小 相等,则物体与地面的之间的摩擦因数为【B】 19.电源的效率η 定义为外电路电阻消耗的功率与电源的总功率之比。在测电源电动势和内电 阻的实验中得到的实验图线如图所示,图中 U 为路端电压,I 为干路电流,a,b 为图线上的 两点,相应状态下电源的功率分别为ηa, ηb 由图可知ηa, ηb 值分别为【D】 20.太阳系中的 8 大行星的轨道均可以通过可以近似看做圆轨道。下列 4 幅图是用来描述这些行 星运动所遵从的某一规律的图像。图中坐标系的横轴是 lg (T/T0) ,纵轴是 lg(R/R0);这里 T 和 R 分别是行星绕太阳运行周期和相应的圆轨道半径,T0 和 R0 分别是水星绕太阳运行的周期和相 应的圆轨道半径。下列 4 幅图中正确的是【B】 21.如图所示,两个端面半径为 R 的圆柱形铁芯同轴水平放置,相对端面之间有一缝隙,铁芯绕 上导线并与电源连接,在缝隙中形成一均匀磁场。一铜质细直棒 ab 水平放置于缝隙中,且与圆 柱轴线等高、垂直。让铜棒从静止开始自由下落,,铜棒下落距离为 0.2R 时铜棒中电动势能大 小为 E1.,下落距离为 0.8R 时电动势能为 E2。忽略涡流损耗和边缘效应。关于 E1、E2 的大小和 铜棒离开磁场前两端的极性,下列判断正确的是【D】 A. E1.>E2 ,a 端为正 B. E1.>E2 ,b 端为正 C. E1.